summaryrefslogtreecommitdiff
path: root/Master/texmf-dist/doc/support/latexindent/success/sampleAFTER.tex
blob: b1f37474fe051c3cf8286b8fa1ec92ac0eba1006 (plain)
1
2
3
4
5
6
7
8
9
10
11
12
13
14
15
16
17
18
19
20
21
22
23
24
25
26
27
28
29
30
31
32
33
34
35
36
37
38
39
40
41
42
43
44
45
46
47
48
49
50
51
52
53
54
55
56
57
58
59
60
61
62
63
64
65
66
67
68
69
70
71
72
73
74
75
76
77
78
79
80
81
82
83
84
85
86
87
88
89
90
91
92
93
94
95
96
97
98
99
100
101
102
103
104
105
106
107
108
109
110
111
112
113
114
115
116
117
118
119
120
121
122
123
124
125
126
127
128
129
130
131
132
133
134
135
136
137
138
139
140
141
142
143
144
145
146
147
148
149
150
151
152
153
154
155
156
157
158
159
160
161
162
163
164
165
166
167
168
169
170
171
172
173
174
175
176
177
178
179
180
181
182
183
184
185
186
187
188
189
190
191
192
193
194
195
196
197
198
199
200
201
202
203
204
205
206
207
208
209
210
211
212
213
214
215
216
217
218
219
220
221
222
223
224
225
226
227
228
229
230
231
232
233
234
235
236
237
238
239
240
241
242
243
244
245
246
247
248
249
250
251
252
253
254
255
256
257
258
259
260
261
262
263
264
265
266
267
268
269
270
271
272
273
274
275
276
277
278
279
280
281
282
283
284
285
286
287
288
289
290
291
292
293
294
295
296
297
298
299
300
301
302
303
304
305
306
307
308
309
310
311
312
313
314
315
316
317
318
319
320
321
322
323
324
325
326
327
328
329
330
331
332
333
334
335
336
337
338
339
340
341
342
343
344
345
346
347
348
349
350
351
352
353
354
355
356
357
358
359
360
361
362
363
364
365
366
367
368
369
370
371
372
373
374
375
376
377
378
379
380
381
382
383
384
385
386
387
388
389
390
391
392
393
394
395
396
397
398
399
400
401
402
403
404
405
406
407
408
409
410
411
412
413
414
415
416
417
418
419
420
421
422
423
424
425
426
427
428
429
430
431
432
433
434
435
436
437
438
439
440
441
442
443
444
445
446
447
448
449
450
451
452
453
454
455
456
457
458
459
460
461
462
463
464
465
466
467
468
469
470
471
472
473
474
475
476
477
478
479
480
481
482
483
484
485
486
487
488
489
490
491
492
493
494
495
496
497
498
499
500
501
502
503
504
505
506
507
508
509
510
511
512
513
514
515
516
517
518
519
520
521
522
523
524
525
526
527
528
529
530
531
532
533
534
535
536
537
538
539
540
541
542
543
544
545
546
547
548
549
550
551
552
553
554
555
556
557
558
559
560
561
562
563
564
565
566
567
568
569
570
571
572
573
574
575
576
577
578
579
580
581
582
583
584
585
586
587
588
589
590
591
592
593
594
595
596
597
598
599
600
601
602
603
604
605
606
607
608
609
610
611
612
613
614
615
616
617
618
619
620
621
622
623
624
625
626
627
628
629
630
631
632
633
634
635
636
637
638
639
640
641
642
643
644
645
646
647
648
649
650
651
652
653
654
655
656
657
658
659
660
661
662
663
664
665
666
667
668
669
670
671
672
673
674
675
676
677
678
679
680
681
682
683
684
685
686
687
688
689
690
691
692
693
694
695
696
697
698
699
700
701
702
703
704
705
706
707
708
709
710
711
712
713
714
715
716
717
718
719
720
721
722
723
724
725
726
727
728
729
730
731
732
733
734
735
736
737
738
739
740
741
742
743
744
745
746
747
748
749
750
751
752
753
754
755
756
757
758
759
760
761
762
763
764
765
766
767
768
769
770
771
772
773
774
775
776
777
778
779
780
781
782
783
784
785
786
787
788
789
790
791
792
793
794
795
796
797
798
799
800
801
802
803
804
805
806
807
808
809
810
811
812
813
814
815
816
817
818
819
820
821
822
823
824
825
826
827
828
829
830
831
832
833
834
835
836
837
838
839
840
841
842
843
844
845
846
847
848
849
850
851
852
853
854
855
856
857
858
859
860
861
862
863
864
865
866
867
868
869
870
871
872
873
874
875
876
877
878
879
880
881
882
883
884
885
886
887
888
889
890
891
892
893
894
895
896
897
898
899
900
901
902
903
904
905
906
907
908
909
910
911
912
913
914
915
916
917
918
919
920
921
922
923
924
925
926
927
928
929
930
931
932
933
934
935
936
937
938
939
940
941
942
943
944
945
946
947
948
949
950
951
952
953
954
955
956
957
958
959
960
961
962
963
964
965
966
967
968
969
970
971
972
973
974
975
976
977
978
979
980
981
982
983
984
985
986
987
988
989
990
991
992
993
994
995
996
997
998
999
1000
1001
1002
1003
1004
1005
1006
1007
1008
1009
1010
1011
1012
1013
1014
1015
1016
1017
1018
1019
1020
1021
1022
1023
1024
1025
1026
1027
1028
1029
1030
1031
1032
1033
1034
1035
1036
1037
1038
1039
1040
1041
1042
1043
1044
1045
1046
1047
1048
1049
1050
1051
1052
1053
1054
1055
1056
1057
1058
1059
1060
1061
1062
1063
1064
1065
1066
1067
1068
1069
1070
1071
1072
1073
1074
1075
1076
1077
1078
1079
1080
1081
1082
1083
1084
1085
1086
1087
1088
1089
1090
1091
1092
1093
1094
1095
1096
1097
1098
1099
1100
1101
1102
1103
1104
1105
1106
1107
1108
1109
1110
1111
1112
1113
1114
1115
1116
1117
1118
1119
1120
1121
1122
1123
1124
1125
1126
1127
1128
1129
1130
1131
1132
1133
1134
1135
1136
1137
1138
1139
1140
1141
1142
1143
1144
1145
1146
1147
1148
1149
1150
1151
1152
1153
1154
1155
1156
1157
1158
1159
1160
1161
1162
1163
1164
1165
1166
1167
1168
1169
1170
1171
1172
1173
1174
1175
1176
1177
1178
1179
1180
1181
1182
1183
1184
1185
1186
1187
1188
1189
1190
1191
1192
1193
1194
1195
1196
1197
1198
1199
1200
1201
1202
1203
1204
1205
1206
1207
1208
1209
1210
1211
1212
1213
1214
1215
1216
1217
1218
1219
1220
1221
1222
1223
1224
1225
1226
1227
1228
1229
1230
1231
1232
1233
1234
1235
1236
1237
1238
1239
1240
1241
1242
1243
1244
1245
1246
1247
1248
1249
1250
1251
1252
1253
1254
1255
1256
1257
1258
1259
1260
1261
1262
1263
1264
1265
1266
1267
1268
1269
1270
1271
1272
1273
1274
1275
1276
1277
1278
1279
1280
1281
1282
1283
1284
1285
1286
1287
1288
1289
1290
1291
1292
1293
1294
1295
1296
1297
1298
1299
1300
1301
1302
1303
1304
1305
1306
1307
1308
1309
1310
1311
1312
1313
1314
1315
1316
1317
1318
1319
1320
1321
1322
1323
1324
1325
1326
1327
1328
1329
1330
1331
1332
1333
1334
1335
1336
1337
1338
1339
1340
1341
1342
1343
1344
1345
1346
1347
1348
1349
1350
1351
1352
1353
1354
1355
1356
1357
1358
1359
1360
1361
1362
1363
1364
1365
1366
1367
1368
1369
1370
1371
1372
1373
1374
1375
1376
1377
1378
1379
1380
1381
1382
1383
1384
1385
1386
1387
1388
1389
1390
1391
1392
1393
1394
1395
1396
1397
1398
1399
1400
1401
1402
1403
1404
1405
1406
1407
1408
1409
1410
1411
1412
1413
1414
1415
1416
1417
1418
1419
1420
1421
1422
1423
1424
1425
1426
1427
1428
1429
1430
1431
1432
1433
1434
1435
1436
1437
1438
1439
1440
1441
1442
1443
1444
1445
1446
1447
1448
1449
1450
1451
1452
1453
1454
1455
1456
1457
1458
1459
1460
1461
1462
1463
1464
1465
1466
1467
1468
1469
1470
1471
1472
1473
1474
1475
1476
1477
1478
1479
1480
1481
1482
1483
1484
1485
1486
1487
1488
1489
1490
1491
1492
1493
1494
1495
1496
1497
1498
1499
1500
1501
1502
1503
1504
1505
1506
1507
1508
1509
1510
1511
1512
1513
1514
1515
1516
1517
1518
1519
1520
1521
1522
1523
1524
1525
1526
1527
1528
1529
1530
1531
1532
1533
1534
1535
1536
1537
1538
1539
1540
1541
1542
1543
1544
1545
1546
1547
1548
1549
1550
1551
1552
1553
1554
1555
1556
1557
1558
1559
1560
1561
1562
1563
1564
1565
1566
1567
1568
1569
1570
1571
1572
1573
1574
1575
1576
1577
1578
1579
1580
1581
1582
1583
1584
1585
1586
1587
1588
1589
1590
1591
1592
1593
1594
1595
1596
1597
1598
1599
1600
1601
1602
1603
1604
1605
1606
1607
1608
1609
1610
1611
1612
1613
1614
1615
1616
1617
1618
1619
1620
1621
1622
1623
1624
1625
1626
1627
1628
1629
1630
1631
1632
1633
1634
1635
1636
1637
1638
1639
1640
1641
1642
1643
1644
1645
1646
1647
1648
1649
1650
1651
1652
1653
1654
1655
1656
1657
1658
1659
1660
1661
1662
1663
1664
1665
1666
1667
1668
1669
1670
1671
1672
1673
1674
1675
1676
1677
1678
1679
1680
1681
1682
1683
1684
1685
1686
1687
1688
1689
1690
1691
1692
1693
1694
1695
1696
1697
1698
1699
1700
1701
1702
1703
1704
1705
1706
1707
1708
1709
1710
1711
1712
1713
1714
1715
1716
1717
1718
1719
1720
1721
1722
1723
1724
1725
1726
1727
1728
1729
1730
1731
1732
1733
1734
1735
1736
1737
1738
1739
1740
1741
1742
1743
1744
1745
1746
1747
1748
1749
1750
1751
1752
1753
1754
1755
1756
1757
1758
1759
1760
1761
1762
1763
1764
1765
1766
1767
1768
1769
1770
1771
1772
1773
1774
1775
1776
1777
1778
1779
1780
1781
1782
1783
1784
1785
1786
1787
1788
1789
1790
1791
1792
1793
1794
1795
1796
1797
1798
1799
1800
1801
1802
1803
1804
1805
1806
1807
1808
1809
1810
1811
1812
1813
1814
1815
1816
1817
1818
1819
1820
1821
1822
1823
1824
1825
1826
1827
1828
1829
1830
1831
1832
1833
1834
1835
1836
1837
1838
1839
1840
1841
1842
1843
1844
1845
1846
1847
1848
1849
1850
1851
1852
1853
1854
1855
1856
1857
1858
1859
1860
1861
1862
1863
1864
1865
1866
1867
1868
1869
1870
1871
1872
1873
1874
1875
1876
1877
1878
1879
1880
1881
1882
1883
1884
1885
1886
1887
1888
1889
1890
1891
1892
1893
1894
1895
1896
1897
1898
1899
1900
1901
1902
1903
1904
1905
1906
1907
1908
1909
1910
1911
1912
1913
1914
1915
1916
1917
1918
1919
1920
1921
1922
1923
1924
1925
1926
1927
1928
1929
1930
1931
1932
1933
1934
1935
1936
1937
1938
1939
1940
1941
1942
1943
1944
1945
1946
1947
1948
1949
1950
1951
1952
1953
1954
1955
1956
1957
1958
1959
1960
1961
1962
1963
1964
1965
1966
1967
1968
1969
1970
1971
1972
1973
1974
1975
1976
1977
1978
1979
1980
1981
1982
1983
1984
1985
1986
1987
1988
1989
1990
1991
1992
1993
1994
1995
1996
1997
1998
1999
2000
2001
2002
2003
2004
2005
2006
2007
2008
2009
2010
2011
2012
2013
2014
2015
2016
2017
2018
2019
2020
2021
2022
2023
2024
2025
2026
2027
2028
2029
2030
2031
2032
2033
2034
2035
2036
2037
2038
2039
2040
2041
2042
2043
2044
2045
2046
2047
2048
2049
2050
2051
2052
2053
2054
2055
2056
2057
2058
2059
2060
2061
2062
2063
2064
2065
2066
2067
2068
2069
2070
2071
2072
2073
2074
2075
2076
2077
2078
2079
2080
2081
2082
2083
2084
2085
2086
2087
2088
2089
2090
2091
2092
2093
2094
2095
2096
2097
2098
2099
2100
2101
2102
2103
2104
2105
2106
2107
2108
2109
2110
2111
2112
2113
2114
2115
2116
2117
2118
2119
2120
2121
2122
2123
2124
2125
2126
2127
2128
2129
2130
2131
2132
2133
2134
2135
2136
2137
2138
2139
2140
2141
2142
2143
2144
2145
2146
2147
2148
2149
2150
2151
2152
2153
2154
2155
2156
2157
2158
2159
2160
2161
2162
2163
2164
2165
2166
2167
2168
2169
2170
2171
2172
2173
2174
2175
2176
2177
2178
2179
2180
2181
2182
2183
2184
2185
2186
2187
2188
2189
2190
2191
2192
2193
2194
2195
2196
2197
2198
2199
2200
2201
2202
2203
2204
2205
2206
2207
2208
2209
2210
2211
2212
2213
2214
2215
2216
2217
2218
2219
2220
2221
2222
2223
2224
2225
2226
2227
2228
2229
2230
2231
2232
2233
2234
2235
2236
2237
2238
2239
2240
2241
2242
2243
2244
2245
2246
2247
2248
2249
2250
2251
2252
2253
2254
2255
2256
2257
2258
2259
2260
2261
2262
2263
2264
2265
2266
2267
2268
2269
2270
2271
2272
2273
2274
2275
2276
2277
2278
2279
2280
2281
2282
2283
2284
2285
2286
2287
2288
2289
2290
2291
2292
2293
2294
2295
2296
2297
2298
2299
2300
2301
2302
2303
2304
2305
2306
2307
2308
2309
2310
2311
2312
2313
2314
2315
2316
2317
2318
2319
2320
2321
2322
2323
2324
2325
2326
2327
2328
2329
2330
2331
2332
2333
2334
2335
2336
2337
2338
2339
2340
2341
2342
2343
2344
2345
2346
2347
2348
2349
2350
2351
2352
2353
2354
2355
2356
2357
2358
2359
2360
2361
2362
2363
2364
2365
2366
2367
2368
2369
2370
2371
2372
2373
2374
2375
2376
2377
2378
2379
2380
2381
2382
2383
2384
2385
2386
2387
2388
2389
2390
2391
2392
2393
2394
2395
2396
2397
2398
2399
2400
2401
2402
2403
2404
2405
2406
2407
2408
2409
2410
2411
2412
2413
2414
2415
2416
2417
2418
2419
2420
2421
2422
2423
2424
2425
2426
2427
2428
2429
2430
2431
2432
2433
2434
2435
2436
2437
2438
2439
2440
2441
2442
2443
2444
2445
2446
2447
2448
2449
2450
2451
2452
2453
2454
2455
2456
2457
2458
2459
2460
2461
2462
2463
2464
2465
2466
2467
2468
2469
2470
2471
2472
2473
2474
2475
2476
2477
2478
2479
2480
2481
2482
2483
2484
2485
2486
2487
2488
2489
2490
2491
2492
2493
2494
2495
2496
2497
2498
2499
2500
2501
2502
2503
2504
2505
2506
2507
2508
2509
2510
2511
2512
2513
2514
2515
2516
2517
2518
2519
2520
2521
2522
2523
2524
2525
2526
2527
2528
2529
2530
2531
2532
2533
2534
2535
2536
2537
2538
2539
2540
2541
2542
2543
2544
2545
2546
2547
2548
2549
2550
2551
2552
2553
2554
2555
2556
2557
2558
2559
2560
2561
2562
2563
2564
2565
2566
2567
2568
2569
2570
2571
2572
2573
2574
2575
2576
2577
2578
2579
2580
2581
2582
2583
2584
2585
2586
2587
2588
2589
2590
2591
2592
2593
2594
2595
2596
2597
2598
2599
2600
2601
2602
2603
2604
2605
2606
2607
2608
2609
2610
2611
2612
2613
2614
2615
2616
2617
2618
2619
2620
2621
2622
2623
2624
2625
2626
2627
2628
2629
2630
2631
2632
2633
2634
2635
2636
2637
2638
2639
2640
2641
2642
2643
2644
2645
2646
2647
2648
2649
2650
2651
2652
2653
2654
2655
2656
2657
2658
2659
2660
2661
2662
2663
2664
2665
2666
2667
2668
2669
2670
2671
2672
2673
2674
2675
2676
2677
2678
2679
2680
2681
2682
2683
2684
2685
2686
2687
2688
2689
2690
2691
2692
2693
2694
2695
2696
2697
2698
2699
2700
2701
2702
2703
2704
2705
2706
2707
2708
2709
2710
2711
2712
2713
2714
2715
2716
2717
2718
2719
2720
2721
2722
2723
2724
2725
2726
2727
2728
2729
2730
2731
2732
2733
2734
2735
2736
2737
2738
2739
2740
2741
2742
2743
2744
2745
2746
2747
2748
2749
2750
2751
2752
2753
2754
2755
2756
2757
2758
2759
2760
2761
2762
2763
2764
2765
2766
2767
2768
2769
2770
2771
2772
2773
2774
2775
2776
2777
2778
2779
2780
2781
2782
2783
2784
2785
2786
2787
2788
2789
2790
2791
2792
2793
2794
2795
2796
2797
2798
2799
2800
2801
2802
2803
2804
2805
2806
2807
2808
2809
2810
2811
2812
2813
2814
2815
2816
2817
2818
2819
2820
2821
2822
2823
2824
2825
2826
2827
2828
2829
2830
2831
2832
2833
2834
2835
2836
2837
2838
2839
2840
2841
2842
2843
2844
2845
2846
2847
2848
2849
2850
2851
2852
2853
2854
2855
2856
2857
2858
2859
2860
2861
2862
2863
2864
2865
2866
2867
2868
2869
2870
2871
2872
2873
2874
2875
2876
2877
2878
2879
2880
2881
2882
2883
2884
2885
2886
2887
2888
2889
2890
2891
2892
2893
2894
2895
2896
2897
2898
2899
2900
2901
2902
2903
2904
2905
2906
2907
2908
2909
2910
2911
2912
2913
2914
2915
2916
2917
2918
2919
2920
2921
2922
2923
2924
2925
2926
2927
2928
2929
2930
2931
2932
2933
2934
2935
2936
2937
2938
2939
2940
2941
2942
2943
2944
2945
2946
2947
2948
2949
2950
2951
2952
2953
2954
2955
2956
2957
2958
2959
2960
2961
2962
2963
2964
2965
2966
2967
2968
2969
2970
2971
2972
2973
2974
2975
2976
2977
2978
2979
2980
2981
2982
2983
2984
2985
2986
2987
2988
2989
2990
2991
2992
2993
2994
2995
2996
2997
2998
2999
3000
3001
3002
3003
3004
3005
3006
3007
3008
3009
3010
3011
3012
3013
3014
3015
3016
3017
3018
3019
3020
3021
3022
3023
3024
3025
3026
3027
3028
3029
3030
3031
3032
3033
3034
3035
3036
3037
3038
3039
3040
3041
3042
3043
3044
3045
3046
3047
3048
3049
3050
3051
3052
3053
3054
3055
3056
3057
3058
3059
3060
3061
3062
3063
3064
3065
3066
3067
3068
3069
3070
3071
3072
3073
3074
3075
3076
3077
3078
3079
3080
3081
3082
3083
3084
3085
3086
3087
3088
3089
3090
3091
3092
3093
3094
3095
3096
3097
3098
3099
3100
3101
3102
3103
3104
3105
3106
3107
3108
3109
3110
3111
3112
3113
3114
3115
3116
3117
3118
3119
3120
3121
3122
3123
3124
3125
3126
3127
3128
3129
3130
3131
3132
3133
3134
3135
3136
3137
3138
3139
3140
3141
3142
3143
3144
3145
3146
3147
3148
3149
3150
3151
3152
3153
3154
3155
3156
3157
3158
3159
3160
3161
3162
3163
3164
3165
3166
3167
3168
3169
3170
3171
3172
3173
3174
3175
3176
3177
3178
3179
3180
3181
3182
3183
3184
3185
3186
3187
3188
3189
3190
3191
3192
3193
3194
3195
3196
3197
3198
3199
3200
3201
3202
3203
3204
3205
3206
3207
3208
3209
3210
3211
3212
3213
3214
3215
3216
3217
3218
3219
3220
3221
3222
3223
3224
3225
3226
3227
3228
3229
3230
3231
3232
3233
3234
3235
3236
3237
3238
3239
3240
3241
3242
3243
3244
3245
3246
3247
3248
3249
3250
3251
3252
3253
3254
3255
3256
3257
3258
3259
3260
3261
3262
3263
3264
3265
3266
3267
3268
3269
3270
3271
3272
3273
3274
3275
3276
3277
3278
3279
3280
3281
3282
3283
3284
3285
3286
3287
3288
3289
3290
3291
3292
3293
3294
3295
3296
3297
3298
3299
3300
3301
3302
3303
3304
3305
3306
3307
3308
3309
3310
3311
3312
3313
3314
3315
3316
3317
3318
3319
3320
3321
3322
3323
3324
3325
3326
3327
3328
3329
3330
3331
3332
3333
3334
3335
3336
3337
3338
3339
3340
3341
3342
3343
3344
3345
3346
3347
3348
3349
3350
3351
3352
3353
3354
3355
3356
3357
3358
3359
3360
3361
3362
3363
3364
3365
3366
3367
3368
3369
3370
3371
3372
3373
3374
3375
3376
3377
3378
3379
3380
3381
3382
3383
3384
3385
3386
3387
3388
3389
3390
3391
3392
3393
3394
3395
3396
3397
3398
3399
3400
3401
3402
3403
3404
3405
3406
3407
3408
3409
3410
3411
3412
3413
3414
3415
3416
3417
3418
3419
3420
3421
3422
3423
3424
3425
3426
3427
3428
3429
3430
3431
3432
3433
3434
3435
3436
3437
3438
3439
3440
3441
3442
3443
3444
3445
3446
3447
3448
3449
3450
3451
3452
3453
3454
3455
3456
3457
3458
3459
3460
3461
3462
3463
3464
3465
3466
3467
3468
3469
3470
3471
3472
3473
3474
3475
3476
3477
3478
3479
3480
3481
3482
3483
3484
3485
3486
3487
3488
3489
3490
3491
3492
3493
3494
3495
3496
3497
3498
3499
3500
3501
3502
3503
3504
3505
3506
3507
3508
3509
3510
3511
3512
3513
3514
3515
3516
3517
3518
3519
3520
3521
3522
3523
3524
3525
3526
3527
3528
3529
3530
3531
3532
3533
3534
3535
3536
3537
3538
3539
3540
3541
3542
3543
3544
3545
3546
3547
3548
3549
3550
3551
3552
3553
3554
3555
3556
3557
3558
3559
3560
3561
3562
3563
3564
3565
3566
3567
3568
3569
3570
3571
3572
3573
3574
3575
3576
3577
3578
3579
3580
3581
3582
3583
3584
3585
3586
3587
3588
3589
3590
3591
3592
3593
3594
3595
3596
3597
3598
3599
3600
3601
3602
3603
3604
3605
3606
3607
3608
3609
3610
3611
3612
3613
3614
3615
3616
3617
3618
3619
3620
3621
3622
3623
3624
3625
3626
3627
3628
3629
3630
3631
3632
3633
3634
3635
3636
3637
3638
3639
3640
3641
3642
3643
3644
3645
3646
3647
3648
3649
3650
3651
3652
3653
3654
3655
3656
3657
3658
3659
3660
3661
3662
3663
3664
3665
3666
3667
3668
3669
3670
3671
3672
3673
3674
3675
3676
3677
3678
3679
3680
3681
3682
3683
3684
3685
3686
3687
3688
3689
3690
3691
3692
3693
3694
3695
3696
3697
3698
3699
3700
3701
3702
3703
3704
3705
3706
3707
3708
3709
3710
3711
3712
3713
3714
3715
3716
3717
3718
3719
3720
3721
3722
3723
3724
3725
3726
3727
3728
3729
3730
3731
3732
3733
3734
3735
3736
3737
3738
3739
3740
3741
3742
3743
3744
3745
3746
3747
3748
3749
3750
3751
3752
3753
3754
3755
3756
3757
3758
3759
3760
3761
3762
3763
3764
3765
3766
3767
3768
3769
3770
3771
3772
3773
3774
3775
3776
3777
3778
3779
3780
3781
3782
3783
3784
3785
3786
3787
3788
3789
3790
3791
3792
3793
3794
3795
3796
3797
3798
3799
3800
3801
3802
3803
3804
3805
3806
3807
3808
3809
3810
3811
3812
3813
3814
3815
3816
3817
3818
3819
3820
3821
3822
3823
3824
3825
3826
3827
3828
3829
3830
3831
3832
3833
3834
3835
3836
3837
3838
3839
3840
3841
3842
3843
3844
3845
3846
3847
3848
3849
3850
3851
3852
3853
3854
3855
3856
3857
3858
3859
3860
3861
3862
3863
3864
3865
3866
3867
3868
3869
3870
3871
3872
3873
3874
3875
3876
3877
3878
3879
3880
3881
3882
3883
3884
3885
3886
3887
3888
3889
3890
3891
3892
3893
3894
3895
3896
3897
3898
3899
3900
3901
3902
3903
3904
3905
3906
3907
3908
3909
3910
3911
3912
3913
3914
3915
3916
3917
3918
3919
3920
3921
3922
3923
3924
3925
3926
3927
3928
3929
3930
3931
3932
3933
3934
3935
3936
3937
3938
3939
3940
3941
3942
3943
3944
3945
3946
3947
3948
3949
3950
3951
3952
3953
3954
3955
3956
3957
3958
3959
3960
3961
3962
3963
3964
3965
3966
3967
3968
3969
3970
3971
3972
3973
3974
3975
3976
3977
3978
3979
3980
3981
3982
3983
3984
3985
3986
3987
3988
3989
3990
3991
3992
3993
3994
3995
3996
3997
3998
3999
4000
4001
4002
4003
4004
4005
4006
4007
4008
4009
4010
4011
4012
4013
4014
4015
4016
4017
4018
4019
4020
4021
4022
4023
4024
4025
4026
4027
4028
4029
4030
4031
4032
4033
4034
4035
4036
4037
4038
4039
4040
4041
4042
4043
4044
4045
4046
4047
4048
4049
4050
4051
4052
4053
4054
4055
4056
4057
4058
4059
4060
4061
4062
4063
4064
4065
4066
4067
4068
4069
4070
4071
4072
4073
4074
4075
4076
4077
4078
4079
4080
4081
4082
4083
4084
4085
4086
4087
4088
4089
4090
4091
4092
4093
4094
4095
4096
4097
4098
4099
4100
4101
4102
4103
4104
4105
4106
4107
4108
4109
4110
4111
4112
4113
4114
4115
4116
4117
4118
4119
4120
4121
4122
4123
4124
4125
4126
4127
4128
4129
4130
4131
4132
4133
4134
4135
4136
4137
4138
4139
4140
4141
4142
4143
4144
4145
4146
4147
4148
4149
4150
4151
4152
4153
4154
4155
4156
4157
4158
4159
4160
4161
4162
4163
4164
4165
4166
4167
4168
4169
4170
4171
4172
4173
4174
4175
4176
4177
4178
4179
4180
4181
4182
4183
4184
4185
4186
4187
4188
4189
4190
4191
4192
4193
4194
4195
4196
4197
4198
4199
4200
4201
4202
4203
4204
4205
4206
4207
4208
4209
4210
4211
4212
4213
4214
4215
4216
4217
4218
4219
4220
4221
4222
4223
4224
4225
4226
4227
4228
4229
4230
4231
4232
4233
4234
4235
4236
4237
4238
4239
4240
4241
4242
4243
4244
4245
4246
4247
4248
4249
4250
4251
4252
4253
4254
4255
4256
4257
4258
4259
4260
4261
4262
4263
4264
4265
4266
4267
4268
4269
4270
4271
4272
4273
4274
4275
4276
4277
4278
4279
4280
4281
4282
4283
4284
4285
4286
4287
4288
4289
4290
4291
4292
4293
4294
4295
4296
4297
4298
4299
4300
4301
4302
4303
4304
4305
4306
4307
4308
4309
4310
4311
4312
4313
4314
4315
4316
4317
4318
4319
4320
4321
4322
4323
4324
4325
4326
4327
4328
4329
4330
4331
4332
4333
4334
4335
4336
4337
4338
4339
4340
4341
4342
4343
4344
4345
4346
4347
4348
4349
4350
4351
4352
4353
4354
4355
4356
4357
4358
4359
4360
4361
4362
4363
4364
4365
4366
4367
4368
4369
4370
4371
4372
4373
4374
4375
4376
4377
4378
4379
4380
4381
4382
4383
4384
4385
4386
4387
4388
4389
4390
4391
4392
4393
4394
4395
4396
4397
4398
4399
4400
4401
4402
4403
4404
4405
4406
4407
4408
4409
4410
4411
4412
4413
4414
4415
4416
4417
4418
4419
4420
4421
4422
4423
4424
4425
4426
4427
4428
4429
4430
4431
4432
4433
4434
4435
4436
4437
4438
4439
4440
4441
4442
4443
4444
4445
4446
4447
4448
4449
4450
4451
4452
4453
4454
4455
4456
4457
4458
4459
4460
4461
4462
4463
4464
4465
4466
4467
4468
4469
4470
4471
4472
4473
4474
4475
4476
4477
4478
4479
4480
4481
4482
4483
4484
4485
4486
4487
4488
4489
4490
4491
4492
4493
4494
4495
4496
4497
4498
4499
4500
4501
4502
4503
4504
4505
4506
4507
4508
4509
4510
4511
4512
4513
4514
4515
4516
4517
4518
4519
4520
4521
4522
4523
4524
4525
4526
4527
4528
4529
4530
4531
4532
4533
4534
4535
4536
4537
4538
4539
4540
4541
4542
4543
4544
4545
4546
4547
4548
4549
4550
4551
4552
4553
4554
4555
4556
4557
4558
4559
4560
4561
4562
4563
4564
4565
4566
4567
4568
4569
4570
4571
4572
4573
4574
4575
4576
4577
4578
4579
4580
4581
4582
4583
4584
4585
4586
4587
4588
4589
4590
4591
4592
4593
4594
4595
4596
4597
4598
4599
4600
4601
4602
4603
4604
4605
4606
4607
4608
4609
4610
4611
4612
4613
4614
4615
4616
4617
4618
4619
4620
4621
4622
4623
4624
4625
4626
4627
4628
4629
4630
4631
4632
4633
4634
4635
4636
4637
4638
4639
4640
4641
4642
4643
4644
4645
4646
4647
4648
4649
4650
4651
4652
4653
4654
4655
4656
4657
4658
4659
4660
4661
4662
4663
4664
4665
4666
4667
4668
4669
4670
4671
4672
4673
4674
4675
4676
4677
4678
4679
4680
4681
4682
4683
4684
4685
4686
4687
4688
4689
4690
4691
4692
4693
4694
4695
4696
4697
4698
4699
4700
4701
4702
4703
4704
4705
4706
4707
4708
4709
4710
4711
4712
4713
4714
4715
4716
4717
4718
4719
4720
4721
4722
4723
4724
4725
4726
4727
4728
4729
4730
4731
4732
4733
4734
4735
4736
4737
4738
4739
4740
4741
4742
4743
4744
4745
4746
4747
4748
4749
4750
4751
4752
4753
4754
4755
4756
4757
4758
4759
4760
4761
4762
4763
4764
4765
4766
4767
4768
4769
4770
4771
4772
4773
4774
4775
4776
4777
4778
4779
4780
4781
4782
4783
4784
4785
4786
4787
4788
4789
4790
4791
4792
4793
4794
4795
4796
4797
4798
4799
4800
4801
4802
4803
4804
4805
4806
4807
4808
4809
4810
4811
4812
4813
4814
4815
4816
4817
4818
4819
4820
4821
4822
4823
4824
4825
4826
4827
4828
4829
4830
4831
4832
4833
4834
4835
4836
4837
4838
4839
4840
4841
4842
4843
4844
4845
4846
4847
4848
4849
4850
4851
4852
4853
4854
4855
4856
4857
4858
4859
4860
4861
4862
4863
4864
4865
4866
4867
4868
4869
4870
4871
4872
4873
4874
4875
4876
4877
4878
4879
4880
4881
4882
4883
4884
4885
4886
4887
4888
4889
4890
4891
4892
4893
4894
4895
4896
4897
4898
4899
4900
4901
4902
4903
4904
4905
4906
4907
4908
4909
4910
4911
4912
4913
4914
4915
4916
4917
4918
4919
4920
4921
4922
4923
4924
4925
4926
4927
4928
4929
4930
4931
4932
4933
4934
4935
4936
4937
4938
4939
4940
4941
4942
4943
4944
4945
4946
4947
4948
4949
4950
4951
4952
4953
4954
4955
4956
4957
4958
4959
4960
4961
4962
4963
4964
4965
4966
4967
4968
4969
4970
4971
4972
4973
4974
4975
4976
4977
4978
4979
4980
4981
4982
4983
4984
4985
4986
4987
4988
4989
4990
4991
4992
4993
4994
4995
4996
4997
4998
4999
5000
5001
5002
5003
5004
5005
5006
5007
5008
5009
5010
5011
5012
5013
5014
5015
5016
5017
5018
5019
5020
5021
5022
5023
5024
5025
5026
5027
5028
5029
5030
5031
5032
5033
5034
5035
5036
5037
5038
5039
5040
5041
5042
5043
5044
5045
5046
5047
5048
5049
5050
5051
5052
5053
5054
5055
5056
5057
5058
5059
5060
5061
5062
5063
5064
5065
5066
5067
5068
5069
5070
5071
5072
5073
5074
5075
5076
5077
5078
5079
5080
5081
5082
5083
5084
5085
5086
5087
5088
5089
5090
5091
5092
5093
5094
5095
5096
5097
5098
5099
5100
5101
5102
5103
5104
5105
5106
5107
5108
5109
5110
5111
5112
5113
5114
5115
5116
5117
5118
5119
5120
5121
5122
5123
5124
5125
5126
5127
5128
5129
5130
5131
5132
5133
5134
5135
5136
5137
5138
5139
5140
5141
5142
5143
5144
5145
5146
5147
5148
5149
5150
5151
5152
5153
5154
5155
5156
5157
5158
5159
5160
5161
5162
5163
5164
5165
5166
5167
5168
5169
5170
5171
5172
5173
5174
5175
5176
5177
5178
5179
5180
5181
5182
5183
5184
5185
5186
5187
5188
5189
5190
5191
5192
5193
5194
5195
5196
5197
5198
5199
5200
5201
5202
5203
5204
5205
5206
5207
5208
5209
5210
5211
5212
5213
5214
5215
5216
5217
5218
5219
5220
5221
5222
5223
5224
5225
5226
5227
5228
5229
5230
5231
5232
5233
5234
5235
5236
5237
5238
5239
5240
5241
5242
5243
5244
5245
5246
5247
5248
5249
5250
5251
5252
5253
5254
5255
5256
5257
5258
5259
5260
5261
5262
5263
5264
5265
5266
5267
5268
5269
5270
5271
5272
5273
5274
5275
5276
5277
5278
5279
5280
5281
5282
5283
5284
5285
5286
5287
5288
5289
5290
5291
5292
5293
5294
5295
5296
5297
5298
5299
5300
5301
5302
5303
5304
5305
5306
5307
5308
5309
5310
5311
5312
5313
5314
5315
5316
5317
5318
5319
5320
5321
5322
5323
5324
5325
5326
5327
5328
5329
5330
5331
5332
5333
5334
5335
5336
5337
5338
5339
5340
5341
5342
5343
5344
5345
5346
5347
5348
5349
5350
5351
5352
5353
5354
5355
5356
5357
5358
5359
5360
5361
5362
5363
5364
5365
5366
5367
5368
5369
5370
5371
5372
5373
5374
5375
5376
5377
5378
5379
5380
5381
5382
5383
5384
5385
5386
5387
5388
5389
5390
5391
5392
5393
5394
5395
5396
5397
5398
5399
5400
5401
5402
5403
5404
5405
5406
5407
5408
5409
5410
5411
5412
5413
5414
5415
5416
5417
5418
5419
5420
5421
5422
5423
5424
5425
5426
5427
5428
5429
5430
5431
5432
5433
5434
5435
5436
5437
5438
5439
5440
5441
5442
5443
5444
5445
5446
5447
5448
5449
5450
5451
5452
5453
5454
5455
5456
5457
5458
5459
5460
5461
5462
5463
5464
5465
5466
5467
5468
5469
5470
5471
5472
5473
5474
5475
5476
5477
5478
5479
5480
5481
5482
5483
5484
5485
5486
5487
5488
5489
5490
5491
5492
5493
5494
5495
5496
5497
5498
5499
5500
5501
5502
5503
5504
5505
5506
5507
5508
5509
5510
5511
5512
5513
5514
5515
5516
5517
5518
5519
5520
5521
5522
5523
5524
5525
5526
5527
5528
5529
5530
5531
5532
5533
5534
5535
5536
5537
5538
5539
5540
5541
5542
5543
5544
5545
5546
5547
5548
5549
5550
5551
5552
5553
5554
5555
5556
5557
5558
5559
5560
5561
5562
5563
5564
5565
5566
5567
5568
5569
5570
5571
5572
5573
5574
5575
5576
5577
5578
5579
5580
5581
5582
5583
5584
5585
5586
5587
5588
5589
5590
5591
5592
5593
5594
5595
5596
5597
5598
5599
5600
5601
5602
5603
5604
5605
5606
5607
5608
5609
5610
5611
5612
5613
5614
5615
5616
5617
5618
5619
5620
5621
5622
5623
5624
5625
5626
5627
5628
5629
5630
5631
5632
5633
5634
5635
5636
5637
5638
5639
5640
5641
5642
5643
5644
5645
5646
5647
5648
5649
5650
5651
5652
5653
5654
5655
5656
5657
5658
5659
5660
5661
5662
5663
5664
5665
5666
5667
5668
5669
5670
5671
5672
5673
5674
5675
5676
5677
5678
5679
5680
5681
5682
5683
5684
5685
5686
5687
5688
5689
5690
5691
5692
5693
5694
5695
5696
5697
5698
5699
5700
5701
5702
5703
5704
5705
5706
5707
5708
5709
5710
5711
5712
5713
5714
5715
5716
5717
5718
5719
5720
5721
5722
5723
5724
5725
5726
5727
5728
5729
5730
5731
5732
5733
5734
5735
5736
5737
5738
5739
5740
5741
5742
5743
5744
5745
5746
5747
5748
5749
5750
5751
5752
5753
5754
5755
5756
5757
5758
5759
5760
5761
5762
5763
5764
5765
5766
5767
5768
5769
5770
5771
5772
5773
5774
5775
5776
5777
5778
5779
5780
5781
5782
5783
5784
5785
5786
5787
5788
5789
5790
5791
5792
5793
5794
5795
5796
5797
5798
5799
5800
5801
5802
5803
5804
5805
5806
5807
5808
5809
5810
5811
5812
5813
5814
5815
5816
5817
5818
5819
% arara: indent: {overwrite: yes}
% A sample chapter file- it contains a lot of
% environments, including tabulars, align, etc
%
% Don't try and compile this file using pdflatex etc, just
% compare the *format* of it to the format of the
% sampleAFTER.tex
%
% In particular, compare the tabular and align-type
% environments before and after running the script

\section{Polynomial functions}
 \reformatstepslist{P} % the steps list should be P1, P2, \ldots
 In your previous mathematics classes you have studied \emph{linear} and
 \emph{quadratic} functions. The most general forms of these types of
 functions can be represented (respectively) by  the functions $f$
 and $g$ that have formulas
 \begin{equation}\label{poly:eq:linquad}
 	f(x)=mx+b, \qquad g(x)=ax^2+bx+c
 \end{equation}
 We know that $m$ is the slope of $f$, and that $a$ is the \emph{leading coefficient}
 of $g$. We also know that the \emph{signs} of $m$ and $a$ completely
 determine the behavior of the functions $f$ and $g$. For example, if $m>0$
 then $f$ is an \emph{increasing} function, and if $m<0$ then $f$ is
 a \emph{decreasing} function.  Similarly, if $a>0$ then $g$ is
 \emph{concave up} and if $a<0$ then $g$ is \emph{concave down}. Graphical
 representations of these statements are given in \cref{poly:fig:linquad}.
   
 \begin{figure}[!htb]
 	\setlength{\figurewidth}{.2\textwidth}
 	\begin{subfigure}{\figurewidth}
 		\begin{tikzpicture}
 			\begin{axis}[
 					framed,
 					xmin=-10,xmax=10,
 					ymin=-10,ymax=10,
 					width=\textwidth,
 					xtick={-11},
 					ytick={-11},
 				]
 				\addplot expression[domain=-10:8]{(x+2)};
 			\end{axis}
 		\end{tikzpicture}
 		\caption{$m>0$}
 	\end{subfigure}
 	\hfill
 	\begin{subfigure}{\figurewidth}
 		\begin{tikzpicture}
 			\begin{axis}[
 					framed,
 					xmin=-10,xmax=10,
 					ymin=-10,ymax=10,
 					width=\textwidth,
 					xtick={-11},
 					ytick={-11},
 				]
 				\addplot expression[domain=-10:8]{-(x+2)};
 			\end{axis}
 		\end{tikzpicture}
 		\caption{$m<0$}
 	\end{subfigure}
 	\hfill
 	\begin{subfigure}{\figurewidth}
 		\begin{tikzpicture}
 			\begin{axis}[
 					framed,
 					xmin=-10,xmax=10,
 					ymin=-10,ymax=10,
 					width=\textwidth,
 					xtick={-11},
 					ytick={-11},
 				]
 				\addplot expression[domain=-4:4]{(x^2-6)};
 			\end{axis}
 		\end{tikzpicture}
 		\caption{$a>0$}
 	\end{subfigure}
 	\hfill
 	\begin{subfigure}{\figurewidth}
 		\begin{tikzpicture}
 			\begin{axis}[
 					framed,
 					xmin=-10,xmax=10,
 					ymin=-10,ymax=10,
 					width=\textwidth,
 					xtick={-11},
 					ytick={-11},
 				]
 				\addplot expression[domain=-4:4]{-(x^2-6)};
 			\end{axis}
 		\end{tikzpicture}
 		\caption{$a<0$}
 	\end{subfigure}
 	\caption{Typical graphs of linear and quadratic functions.}
 	\label{poly:fig:linquad}
 \end{figure}
   
 Let's look a little more closely at the formulas for $f$ and $g$ in
 \cref{poly:eq:linquad}. Note that the \emph{degree}
 of $f$ is $1$ since the highest power of $x$ that is present in the
 formula for $f(x)$ is $1$. Similarly, the degree of $g$ is $2$ since
 the highest power of $x$ that is present in the formula for $g(x)$
 is $2$.
   
 In this section we will build upon our knowledge of these elementary
 functions. In particular, we will generalize the functions $f$ and $g$ to a function $p$ that has
 any degree that we wish.
   
 %===================================
 %   Author: Hughes
 %   Date:   March 2012
 %===================================
 \begin{essentialskills}
 	%===================================
 	%   Author: Hughes
 	%   Date:   March 2012
 	%===================================
 	\begin{problem}[Quadratic functions]
 	Every quadratic function has the form $y=ax^2+bx+c$; state the value
 	of $a$ for each of the following functions, and hence decide if the
 	parabola that represents the function opens upward or downward.
 	\begin{multicols}{2}
 		\begin{subproblem}
 			$F(x)=x^2+3$
 			\begin{shortsolution}
 				$a=1$; the parabola opens upward.
 			\end{shortsolution}
 		\end{subproblem}
 		\begin{subproblem}
 			$G(t)=4-5t^2$
 			\begin{shortsolution}
 				$a=-5$; the parabola opens downward.
 			\end{shortsolution}
 		\end{subproblem}
 		\begin{subproblem}
 			$H(y)=4y^2-96y+8$
 			\begin{shortsolution}
 				$a=4$; the parabola opens upward.
 			\end{shortsolution}
 		\end{subproblem}
 		\begin{subproblem}
 			$K(z)=-19z^2$
 			\begin{shortsolution}
 				$m=-19$; the parabola opens downward.
 			\end{shortsolution}
 		\end{subproblem}
 	\end{multicols}
 	Now let's generalize our findings for the most general quadratic function $g$
 	that has formula $g(x)=a_2x^2+a_1x+a_0$. Complete the following sentences.
 	\begin{subproblem}
 		When $a_2>0$, the parabola that represents $y=g(x)$ opens $\ldots$
 		\begin{shortsolution}
 			When $a_2>0$, the parabola that represents the function opens upward.
 		\end{shortsolution}
 	\end{subproblem}
 	\begin{subproblem}
 		When $a_2<0$, the parabola that represents $y=g(x)$ opens $\ldots$
 		\begin{shortsolution}
 			When $a_2<0$, the parabola that represents the function opens downward.
 		\end{shortsolution}
 	\end{subproblem}
 	\end{problem}
 \end{essentialskills}
   
 \subsection*{Power functions with positive exponents}
 	The study of polynomials will rely upon a good knowledge
 	of power functions| you may reasonably ask, what is a power function?
 	\begin{pccdefinition}[Power functions]
 		Power functions have the form
 		\[
 			f(x) = a_n x^n
 		\]
 		where $n$ can be any real number.
 		 		 		
 		Note that for this section we will only be concerned with the
 		case when $n$ is a positive integer.
 	\end{pccdefinition}
 	 	 	
 	You may find assurance in the fact that you are already very comfortable
 	with power functions that have $n=1$ (linear) and $n=2$ (quadratic). Let's
 	explore some power functions that you might not be so familiar with.
 	As you read \cref{poly:ex:oddpow,poly:ex:evenpow}, try and spot
 	as many patterns and similarities as you can.
 	 	 	
 	%===================================
 	%   Author: Hughes
 	%   Date:   March 2012
 	%===================================
 	\begin{pccexample}[Power functions with odd positive exponents]
 		\label{poly:ex:oddpow}
 		Graph each of the following functions, state their domain, and their
 		long-run behavior as $x\rightarrow\pm\infty$
 		\[
 			f(x)=x^3,   \qquad  g(x)=x^5, \qquad h(x)=x^7
 		\]
 		\begin{pccsolution}
 			The functions $f$, $g$, and $h$ are plotted in \cref{poly:fig:oddpow}.
 			The domain of each of the functions $f$, $g$, and $h$ is $(-\infty,\infty)$. Note that
 			the long-run behavior of each of the functions is the same, and in particular
 			\begin{align*}
 				f(x)\rightarrow\infty                           & \text{ as } x\rightarrow\infty  \\
 				\mathllap{\text{and }}   f(x)\rightarrow-\infty & \text{ as } x\rightarrow-\infty 
 			\end{align*}
 			The same results hold for $g$ and $h$.
 		\end{pccsolution}
 	\end{pccexample}
 	 	 	
 	\begin{figure}[!htb]
 		\begin{minipage}{.45\textwidth}
 			\begin{tikzpicture}
 				\begin{axis}[
 						framed,
 						xmin=-1.5,xmax=1.5,
 						ymin=-5,ymax=5,
 						xtick={-1.0,-0.5,...,1.0},
 						minor ytick={-3,-1,...,3},
 						grid=both,
 						width=\textwidth,
 						legend pos=north west,
 					]
 					\addplot expression[domain=-1.5:1.5]{x^3};
 					\addplot expression[domain=-1.379:1.379]{x^5};
 					\addplot expression[domain=-1.258:1.258]{x^7};
 					\addplot[soldot]coordinates{(-1,-1)} node[axisnode,anchor=north west]{$(-1,-1)$};
 					\addplot[soldot]coordinates{(1,1)} node[axisnode,anchor=south east]{$(1,1)$};
 					\legend{$f$,$g$,$h$}
 				\end{axis}
 			\end{tikzpicture}
 			\caption{Odd power functions}
 			\label{poly:fig:oddpow}
 		\end{minipage}%
 		\hfill
 		\begin{minipage}{.45\textwidth}
 			\begin{tikzpicture}
 				\begin{axis}[
 						framed,
 						xmin=-2.5,xmax=2.5,
 						ymin=-5,ymax=5,
 						xtick={-2.0,-1.5,...,2.0},
 						minor ytick={-3,-1,...,3},
 						grid=both,
 						width=\textwidth,
 						legend pos=south east,
 					]
 					\addplot expression[domain=-2.236:2.236]{x^2};
 					\addplot expression[domain=-1.495:1.495]{x^4};
 					\addplot expression[domain=-1.307:1.307]{x^6};
 					\addplot[soldot]coordinates{(-1,1)} node[axisnode,anchor=east]{$(-1,1)$};
 					\addplot[soldot]coordinates{(1,1)} node[axisnode,anchor=west]{$(1,1)$};
 					\legend{$F$,$G$,$H$}
 				\end{axis}
 			\end{tikzpicture}
 			\caption{Even power functions}
 			\label{poly:fig:evenpow}
 		\end{minipage}%
 	\end{figure}
 	 	 	
 	%===================================
 	%   Author: Hughes
 	%   Date:   March 2012
 	%===================================
 	\begin{pccexample}[Power functions with even positive exponents]\label{poly:ex:evenpow}%
 		Graph each of the following functions, state their domain, and their
 		long-run behavior as $x\rightarrow\pm\infty$
 		\[
 			F(x)=x^2, \qquad G(x)=x^4, \qquad H(x)=x^6
 		\]
 		\begin{pccsolution}
 			The functions $F$, $G$, and $H$ are plotted in \cref{poly:fig:evenpow}. The domain
 			of each of the functions is $(-\infty,\infty)$. Note that the long-run behavior
 			of each of the functions is the same, and in particular
 			\begin{align*}
 				F(x)\rightarrow\infty                          & \text{ as } x\rightarrow\infty  \\
 				\mathllap{\text{and }}   F(x)\rightarrow\infty & \text{ as } x\rightarrow-\infty 
 			\end{align*}
 			The same result holds for $G$ and $H$.
 		\end{pccsolution}
 	\end{pccexample}
 	 	 	
 	\begin{doyouunderstand}
 		\begin{problem}
 		Repeat \cref{poly:ex:oddpow,poly:ex:evenpow} using (respectively)
 		\begin{subproblem}
 			$f(x)=-x^3,   \qquad  g(x)=-x^5, \qquad h(x)=-x^7$
 			\begin{shortsolution}
 				The functions $f$, $g$, and $h$ have domain $(-\infty,\infty)$ and
 				are graphed below.
 				 				 				
 				\begin{tikzpicture}
 					\begin{axis}[
 							framed,
 							xmin=-1.5,xmax=1.5,
 							ymin=-5,ymax=5,
 							xtick={-1.0,-0.5,...,0.5},
 							minor ytick={-3,-1,...,3},
 							grid=both,
 							width=\solutionfigurewidth,
 							legend pos=north east,
 						]
 						\addplot expression[domain=-1.5:1.5]{-x^3};
 						\addplot expression[domain=-1.379:1.379]{-x^5};
 						\addplot expression[domain=-1.258:1.258]{-x^7};
 						\legend{$f$,$g$,$h$}
 					\end{axis}
 				\end{tikzpicture}
 				 				 				
 				Note that
 				\begin{align*}
 					f(x)\rightarrow-\infty                         & \text{ as } x\rightarrow\infty  \\
 					\mathllap{\text{and }}   f(x)\rightarrow\infty & \text{ as } x\rightarrow-\infty 
 				\end{align*}
 				The same is true for $g$ and $h$.
 			\end{shortsolution}
 		\end{subproblem}
 		\begin{subproblem}
 			$F(x)=-x^2,   \qquad  G(x)=-x^4, \qquad H(x)=-x^6$
 			\begin{shortsolution}
 				The functions $F$, $G$, and $H$ have domain $(-\infty,\infty)$ and
 				are graphed below.
 				 				 				
 				\begin{tikzpicture}
 					\begin{axis}[
 							framed,
 							xmin=-2.5,xmax=2.5,
 							ymin=-5,ymax=5,
 							xtick={-1.0,-0.5,...,0.5},
 							minor ytick={-3,-1,...,3},
 							grid=both,
 							width=\solutionfigurewidth,
 							legend pos=north east,
 						]
 						\addplot expression[domain=-2.236:2.236]{-x^2};
 						\addplot expression[domain=-1.495:1.495]{-x^4};
 						\addplot expression[domain=-1.307:1.307]{-x^6};
 						\legend{$F$,$G$,$H$}
 					\end{axis}
 				\end{tikzpicture}
 				 				 				
 				Note that
 				\begin{align*}
 					F(x)\rightarrow-\infty                          & \text{ as } x\rightarrow\infty  \\
 					\mathllap{\text{and }}   F(x)\rightarrow-\infty & \text{ as } x\rightarrow-\infty 
 				\end{align*}
 				The same is true for $G$ and $H$.
 			\end{shortsolution}
 		\end{subproblem}
 		\end{problem}
 	\end{doyouunderstand}
 	 	 	
 \subsection*{Polynomial functions}
 	Now that we have a little more familiarity with power functions,
 	we can define polynomial functions. Provided that you were comfortable
 	with our opening discussion about linear and quadratic functions (see
 	$f$ and $g$ in \cref{poly:eq:linquad}) then there is every chance
 	that you'll be able to master polynomial functions as well; just remember
 	that polynomial functions are a natural generalization of linear
 	and quadratic functions. Once you've studied the examples and problems
 	in this section, you'll hopefully agree that polynomial functions
 	are remarkably predictable.
 	 	 	
 	%===================================
 	%   Author: Hughes
 	%   Date:   May 2011
 	%===================================
 	\begin{pccdefinition}[Polynomial functions]
 		Polynomial functions have the form
 		\[
 			p(x)=a_nx^n+a_{n-1}x^{n-1}+\ldots+a_1x+a_0
 		\]
 		where $a_n$, $a_{n-1}$, $a_{n-2}$, \ldots, $a_0$ are real numbers.
 		\begin{itemize}
 			\item We call $n$ the degree of the polynomial, and require that $n$
 			      is a non-negative integer;
 			\item $a_n$, $a_{n-1}$, $a_{n-2}$, \ldots, $a_0$ are called the coefficients;
 			\item We typically write polynomial functions in descending powers of $x$.
 		\end{itemize}
 		In particular, we call $a_n$ the \emph{leading} coefficient, and $a_nx^n$ the
 		\emph{leading term}.
 		 		 		
 		Note that if a polynomial is given in factored form, then the degree can be found
 		by counting the number of linear factors.
 	\end{pccdefinition}
 	 	 	
 	%===================================
 	%   Author: Hughes
 	%   Date:   March 2012
 	%===================================
 	\begin{pccexample}[Polynomial or not]
 		Identify the following functions as polynomial or not; if the function
 		is a polynomial, state its degree.
 		\begin{multicols}{3}
 			\begin{enumerate}
 				\item $p(x)=x^2-3$
 				\item $q(x)=-4x^{\nicefrac{1}{2}}+10$
 				\item $r(x)=10x^5$
 				\item $s(x)=x^{-2}+x^{23}$
 				\item $f(x)=-8$
 				\item $g(x)=3^x$
 				\item $h(x)=\sqrt[3]{x^7}-x^2+x$
 				\item $k(x)=4x(x+2)(x-3)$
 				\item $j(x)=x^2(x-4)(5-x)$
 			\end{enumerate}
 		\end{multicols}
 		\begin{pccsolution}
 			\begin{enumerate}
 				\item $p$ is a polynomial, and its degree is $2$.
 				\item $q$ is \emph{not} a polynomial, because $\frac{1}{2}$ is not an integer.
 				\item $r$ is a polynomial, and its degree is $5$.
 				\item $s$ is \emph{not} a polynomial, because $-2$ is not a positive integer.
 				\item $f$ is a polynomial, and its degree is $0$.
 				\item $g$ is \emph{not} a polynomial, because the independent
 				      variable, $x$, is in the exponent.
 				\item $h$ is \emph{not} a polynomial, because $\frac{7}{3}$ is not an integer.
 				\item $k$ is a polynomial, and its degree is $3$.
 				\item $j$ is a polynomial, and its degree is $4$.
 			\end{enumerate}
 		\end{pccsolution}
 	\end{pccexample}
 	 	 	
 	%===================================
 	%   Author: Hughes
 	%   Date:   March 2012
 	%===================================
 	\begin{pccexample}[Typical graphs]\label{poly:ex:typical}
 		\Cref{poly:fig:typical} shows graphs of some polynomial functions;
 		the ticks have deliberately been left off the axis to allow us to concentrate
 		on the features of each graph. Note in particular that:
 		\begin{itemize}
 			\item \cref{poly:fig:typical1} shows a degree-$1$ polynomial (you might also
 			      classify the function as linear) whose leading coefficient, $a_1$, is positive.
 			\item \cref{poly:fig:typical2} shows a degree-$2$ polynomial (you might also
 			      classify the function as quadratic) whose leading coefficient, $a_2$, is positive.
 			\item \cref{poly:fig:typical3} shows a degree-$3$ polynomial whose leading coefficient, $a_3$,
 			      is positive| compare its overall
 			      shape and long-run behavior to the functions described in \cref{poly:ex:oddpow}.
 			\item \cref{poly:fig:typical4} shows a degree-$4$ polynomial whose leading coefficient, $a_4$,
 			      is positive|compare its overall shape and long-run behavior to the functions described in \cref{poly:ex:evenpow}.
 			\item \cref{poly:fig:typical5} shows a degree-$5$ polynomial whose leading coefficient, $a_5$,
 			      is positive| compare its overall
 			      shape and long-run behavior to the functions described in \cref{poly:ex:oddpow}.
 		\end{itemize}
 	\end{pccexample}
 	 	 	
 	%===================================
 	%   Author: Hughes
 	%   Date:   May 2011
 	%===================================
 	\begin{figure}[!htb]
 		\begin{widepage}
 		\setlength{\figurewidth}{\textwidth/6}
 		\begin{subfigure}{\figurewidth}
 			\begin{tikzpicture}
 				\begin{axis}[
 						framed,
 						xmin=-10,xmax=10,
 						ymin=-10,ymax=10,
 						width=\textwidth,
 						xtick={-11},
 						ytick={-11},
 					]
 					\addplot expression[domain=-10:8]{(x+2)};
 				\end{axis}
 			\end{tikzpicture}
 			\caption{$a_1>0$}
 			\label{poly:fig:typical1}
 		\end{subfigure}
 		\hfill
 		\begin{subfigure}{\figurewidth}
 			\begin{tikzpicture}
 				\begin{axis}[
 						framed,
 						xmin=-10,xmax=10,
 						ymin=-10,ymax=10,
 						width=\textwidth,
 						xtick={-11},
 						ytick={-11},
 					]
 					\addplot expression[domain=-4:4]{(x^2-6)};
 				\end{axis}
 			\end{tikzpicture}
 			\caption{$a_2>0$}
 			\label{poly:fig:typical2}
 		\end{subfigure}
 		\hfill
 		\begin{subfigure}{\figurewidth}
 			\begin{tikzpicture}
 				\begin{axis}[
 						framed,
 						xmin=-10,xmax=10,
 						ymin=-10,ymax=10,
 						width=\textwidth,
 						xtick={-11},
 						ytick={-11},
 					]
 					\addplot expression[domain=-7.5:7.5]{0.05*(x+6)*x*(x-6)};
 				\end{axis}
 			\end{tikzpicture}
 			\caption{$a_3>0$}
 			\label{poly:fig:typical3}
 		\end{subfigure}
 		\hfill
 		\begin{subfigure}{\figurewidth}
 			\begin{tikzpicture}
 				\begin{axis}[
 						framed,
 						xmin=-10,xmax=10,
 						ymin=-10,ymax=10,
 						width=\textwidth,
 						xtick={-11},
 						ytick={-11},
 					]
 					\addplot expression[domain=-2.35:5.35,samples=100]{0.2*(x-5)*x*(x-3)*(x+2)};
 				\end{axis}
 			\end{tikzpicture}
 			\caption{$a_4>0$}
 			\label{poly:fig:typical4}
 		\end{subfigure}
 		\hfill
 		\begin{subfigure}{\figurewidth}
 			\begin{tikzpicture}
 				\begin{axis}[
 						framed,
 						xmin=-10,xmax=10,
 						ymin=-10,ymax=10,
 						width=\textwidth,
 						xtick={-11},
 						ytick={-11},
 					]
 					\addplot expression[domain=-5.5:6.3,samples=100]{0.01*(x+2)*x*(x-3)*(x+5)*(x-6)};
 				\end{axis}
 			\end{tikzpicture}
 			\caption{$a_5>0$}
 			\label{poly:fig:typical5}
 		\end{subfigure}
 		\end{widepage}
 		\caption{Graphs to illustrate typical curves of polynomial functions.}
 		\label{poly:fig:typical}
 	\end{figure}
 	 	 	
 	%===================================
 	%   Author: Hughes
 	%   Date:   March 2012
 	%===================================
 	\begin{doyouunderstand}
 		\begin{problem}
 		Use \cref{poly:ex:typical} and \cref{poly:fig:typical} to help you sketch
 		the graphs of polynomial functions that have negative leading coefficients| note
 		that there are many ways to do this! The intention with this problem
 		is to use your knowledge of transformations- in particular, \emph{reflections}-
 		to guide you.
 		\begin{shortsolution}
 			$a_1<0$:
 			 			 			
 			\begin{tikzpicture}
 				\begin{axis}[
 						framed,
 						xmin=-10,xmax=10,
 						ymin=-10,ymax=10,
 						width=\solutionfigurewidth,
 						xtick={-11},
 						ytick={-11},
 					]
 					\addplot expression[domain=-10:8]{-(x+2)};
 				\end{axis}
 			\end{tikzpicture}
 			 			 			
 			$a_2<0$
 			 			 			
 			\begin{tikzpicture}
 				\begin{axis}[
 						framed,
 						xmin=-10,xmax=10,
 						ymin=-10,ymax=10,
 						width=\solutionfigurewidth,
 						xtick={-11},
 						ytick={-11},
 					]
 					\addplot expression[domain=-4:4]{-(x^2-6)};
 				\end{axis}
 			\end{tikzpicture}
 			 			 			
 			$a_3<0$
 			 			 			
 			\begin{tikzpicture}
 				\begin{axis}[
 						framed,
 						xmin=-10,xmax=10,
 						ymin=-10,ymax=10,
 						width=\solutionfigurewidth,
 						xtick={-11},
 						ytick={-11},
 					]
 					\addplot expression[domain=-7.5:7.5]{-0.05*(x+6)*x*(x-6)};
 				\end{axis}
 			\end{tikzpicture}
 			 			 			
 			$a_4<0$
 			 			 			
 			\begin{tikzpicture}
 				\begin{axis}[
 						framed,
 						xmin=-10,xmax=10,
 						ymin=-10,ymax=10,
 						width=\solutionfigurewidth,
 						xtick={-11},
 						ytick={-11},
 					]
 					\addplot expression[domain=-2.35:5.35,samples=100]{-0.2*(x-5)*x*(x-3)*(x+2)};
 				\end{axis}
 			\end{tikzpicture}
 			 			 			
 			$a_5<0$
 			 			 			
 			\begin{tikzpicture}
 				\begin{axis}[
 						framed,
 						xmin=-10,xmax=10,
 						ymin=-10,ymax=10,
 						width=\solutionfigurewidth,
 						xtick={-11},
 						ytick={-11},
 					]
 					\addplot expression[domain=-5.5:6.3,samples=100]{-0.01*(x+2)*x*(x-3)*(x+5)*(x-6)};
 				\end{axis}
 			\end{tikzpicture}
 		\end{shortsolution}
 		\end{problem}
 	\end{doyouunderstand}
 	 	 	
 	\fixthis{poly: Need a more basic example here- it can have a similar
 		format to the multiple zeros example, but just keep it simple; it should
 	be halfway between the 2 examples surrounding it}
 	 	 	
 	%===================================
 	%   Author: Hughes
 	%   Date:   May 2011
 	%===================================
 	\begin{pccexample}[Multiple zeros]
 		Consider the polynomial functions $p$, $q$, and $r$ which are
 		graphed in \cref{poly:fig:moremultiple}.
 		The formulas for $p$, $q$, and $r$ are as follows
 		\begin{align*}
 			p(x) & =(x-3)^2(x+4)^2       \\
 			q(x) & =x(x+2)^2(x-1)^2(x-3) \\
 			r(x) & =x(x-3)^3(x+1)^2      
 		\end{align*}
 		Find the degree of $p$, $q$, and $r$, and decide if the functions bounce off or cut
 		through the horizontal axis at each of their zeros.
 		\begin{pccsolution}
 			The degree of $p$ is 4. Referring to \cref{poly:fig:bouncep},
 			the curve bounces off the horizontal axis at both zeros, $3$ and $4$.
 			 			 			
 			The degree of $q$ is 6. Referring to \cref{poly:fig:bounceq},
 			the curve bounces off the horizontal axis at $-2$ and $1$, and cuts
 			through the horizontal axis at $0$ and $3$.
 			 			 			
 			The degree of $r$ is 6. Referring to \cref{poly:fig:bouncer},
 			the curve bounces off the horizontal axis at $-1$, and cuts through
 			the horizontal axis at $0$ and at $3$, although is flattened immediately to the left and right of $3$.
 		\end{pccsolution}
 	\end{pccexample}
 	 	 	
 	\setlength{\figurewidth}{0.25\textwidth}
 	\begin{figure}[!htb]
 		\begin{subfigure}{\figurewidth}
 			\begin{tikzpicture}
 				\begin{axis}[
 						xmin=-6,xmax=5,
 						ymin=-30,ymax=200,
 						xtick={-4,-2,...,4},
 						width=\textwidth,
 					]
 					\addplot expression[domain=-5.63733:4.63733,samples=50]{(x-3)^2*(x+4)^2};
 					\addplot[soldot]coordinates{(3,0)(-4,0)};
 				\end{axis}
 			\end{tikzpicture}
 			\caption{$y=p(x)$}
 			\label{poly:fig:bouncep}
 		\end{subfigure}
 		\hfill
 		\begin{subfigure}{\figurewidth}
 			\begin{tikzpicture}
 				\begin{axis}[
 						xmin=-3,xmax=4,
 						xtick={-2,...,3},
 						ymin=-60,ymax=40,
 						width=\textwidth,
 					]
 					\addplot+[samples=50] expression[domain=-2.49011:3.11054]{x*(x+2)^2*(x-1)^2*(x-3)};
 					\addplot[soldot]coordinates{(-2,0)(0,0)(1,0)(3,0)};
 				\end{axis}
 			\end{tikzpicture}
 			\caption{$y=q(x)$}
 			\label{poly:fig:bounceq}
 		\end{subfigure}
 		\hfill
 		\begin{subfigure}{\figurewidth}
 			\begin{tikzpicture}
 				\begin{axis}[
 						xmin=-2,xmax=4,
 						xtick={-1,...,3},
 						ymin=-40,ymax=40,
 						width=\textwidth,
 					]
 					\addplot expression[domain=-1.53024:3.77464,samples=50]{x*(x-3)^3*(x+1)^2};
 					\addplot[soldot]coordinates{(-1,0)(0,0)(3,0)};
 				\end{axis}
 			\end{tikzpicture}
 			\caption{$y=r(x)$}
 			\label{poly:fig:bouncer}
 		\end{subfigure}
 		\caption{}
 		\label{poly:fig:moremultiple}
 	\end{figure}
 	 	 	
 	\begin{pccdefinition}[Multiple zeros]\label{poly:def:multzero}
 		Let $p$ be a polynomial that has a repeated linear factor $(x-a)^n$. Then we say
 		that $p$ has a multiple zero at $a$ of multiplicity $n$ and
 		\begin{itemize}
 			\item if the factor $(x-a)$ is repeated an even number of times, the graph of $y=p(x)$ does not
 			      cross the $x$ axis at $a$, but `bounces' off the horizontal axis at $a$.
 			\item if the factor $(x-a)$ is repeated an odd number of times, the graph of $y=p(x)$ crosses the
 			      horizontal axis at $a$, but it looks `flattened' there
 		\end{itemize}
 		If $n=1$, then we say that $p$ has a \emph{simple} zero at $a$.
 	\end{pccdefinition}
 	 	 	
 	%===================================
 	%   Author: Hughes
 	%   Date:   May 2011
 	%===================================
 	\begin{pccexample}[Find a formula]
 		Find formulas for the polynomial functions, $p$ and $q$, graphed in \cref{poly:fig:findformulademoboth}.
 		\begin{figure}[!htb]
 			\begin{subfigure}{.45\textwidth}
 				\begin{tikzpicture}
 					\begin{axis}[framed,
 							xmin=-5,xmax=5,
 							ymin=-10,ymax=10,
 							xtick={-4,-2,...,4},
 							minor xtick={-3,-1,...,3},
 							ytick={-8,-6,...,8},
 							width=\textwidth,
 						grid=both]
 						\addplot expression[domain=-3.25842:2.25842,samples=50]{-x*(x-2)*(x+3)*(x+1)};
 						\addplot[soldot]coordinates{(1,8)}node[axisnode,inner sep=.35cm,anchor=west]{$(1,8)$};
 						\addplot[soldot]coordinates{(-3,0)(-1,0)(0,0)(2,0)};
 					\end{axis}
 				\end{tikzpicture}
 				\caption{$p$}
 				\label{poly:fig:findformulademo}
 			\end{subfigure}
 			\hfill
 			\begin{subfigure}{.45\textwidth}
 				\begin{tikzpicture}
 					\begin{axis}[framed,
 							xmin=-5,xmax=5,
 							ymin=-10,ymax=10,
 							xtick={-4,-2,...,4},
 							minor xtick={-3,-1,...,3},
 							ytick={-8,-6,...,8},
 							width=\textwidth,
 						grid=both]
 						\addplot expression[domain=-4.33:4.08152]{-.25*(x+2)^2*(x-3)};
 						\addplot[soldot]coordinates{(2,4)}node[axisnode,anchor=south west]{$(2,4)$};
 						\addplot[soldot]coordinates{(-2,0)(3,0)};
 					\end{axis}
 				\end{tikzpicture}
 				\caption{$q$}
 				\label{poly:fig:findformulademo1}
 			\end{subfigure}
 			\caption{}
 			\label{poly:fig:findformulademoboth}
 		\end{figure}
 		\begin{pccsolution}
 			\begin{enumerate}
 				\item We begin by noting that the horizontal intercepts of $p$ are $(-3,0)$, $(-1,0)$, $(0,0)$ and $(2,0)$.
 				      We also note that each zero is simple (multiplicity $1$).
 				      If we assume that $p$ has no other zeros, then we can start by writing
 				      \begin{align*}
 				      	p(x) & =(x+3)(x+1)(x-0)(x-2) \\
 				      	     & =x(x+3)(x+1)(x-2)     \\
 				      \end{align*}
 				      According to \cref{poly:fig:findformulademo}, the point $(1,8)$ lies
 				      on the curve $y=p(x)$.
 				      Let's check if the formula we have written satisfies this requirement
 				      \begin{align*}
 				      	p(1) & = (1)(4)(2)(-1) \\
 				      	     & = -8            
 				      \end{align*}
 				      which is clearly not correct| it is close though. We can correct this by
 				      multiplying $p$ by a constant $k$; so let's assume that
 				      \[
 				      	p(x)=kx(x+3)(x+1)(x-2)
 				      \]
 				      Then $p(1)=-8k$, and if this is to equal $8$, then $k=-1$. Therefore
 				      the formula for $p(x)$ is
 				      \[
 				      	p(x)=-x(x+3)(x+1)(x-2)
 				      \]
 				\item The function $q$ has a zero at $-2$ of multiplicity $2$, and zero of
 				      multiplicity $1$ at $3$ (so $3$ is a simple zero of $q$); we can therefore assume that $q$ has the form
 				      \[
 				      	q(x)=k(x+2)^2(x-3)
 				      \]
 				      where $k$ is some real number. In order to find $k$, we use the given ordered pair, $(2,4)$, and
 				      evaluate $p(2)$
 				      \begin{align*}
 				      	p(2) & =k(4)^2(-1) \\
 				      	     & =-16k       
 				      \end{align*}
 				      We solve the equation $4=-8k$ and obtain $k=-\frac{1}{4}$ and conclude that the
 				      formula for $q(x)$ is
 				      \[
 				      	q(x)=-\frac{1}{4}(x+2)^2(x-3)
 				      \]
 			\end{enumerate}
 		\end{pccsolution}
 	\end{pccexample}
 	 	 	
 	 	 	
 	\fixthis{Chris: need sketching polynomial problems}
 	\begin{pccspecialcomment}[Steps to follow when sketching polynomial functions]
 		\begin{steps}
 			\item \label{poly:step:first} Determine the degree of the polynomial,
 			its leading term and leading coefficient, and hence determine
 			the long-run behavior of the polynomial| does it behave like $\pm x^2$ or $\pm x^3$
 			as $x\rightarrow\pm\infty$?
 			\item Determine the zeros and their multiplicity. Mark all zeros
 			and the vertical intercept on the graph using solid circles $\bullet$.
 			\item \label{poly:step:last}  Deduce the overall shape of the curve, and sketch it. If there isn't
 			enough information from the previous steps, then construct a table of values.
 		\end{steps}
 		Remember that until we have the tools of calculus, we won't be able to
 		find the exact coordinates of local minimums, local maximums, and points
 		of inflection.
 	\end{pccspecialcomment}
 	Before we demonstrate some examples, it is important to remember the following:
 	\begin{itemize}
 		\item our sketches will give a good representation of the overall
 		      shape of the graph, but until we have the tools of calculus (from MTH 251)
 		      we can not find local minimums, local maximums, and inflection points algebraically. This
 		      means that we will make our best guess as to where these points are.
 		\item we will not concern ourselves too much with the vertical scale (because of
 		      our previous point)| we will, however, mark the vertical intercept (assuming there is one),
 		      and any horizontal asymptotes.
 	\end{itemize}
 	%===================================
 	%   Author: Hughes
 	%   Date:   May 2012
 	%===================================
 	\begin{pccexample}\label{poly:ex:simplecubic}
 		Use \crefrange{poly:step:first}{poly:step:last} to sketch a graph of the function $p$
 		that has formula
 		\[
 			p(x)=\frac{1}{2}(x-4)(x-1)(x+3)
 		\]
 		\begin{pccsolution}
 			\begin{steps}
 				\item $p$ has degree $3$. The leading term of $p$ is $\frac{1}{2}x^3$, so the leading coefficient of $p$
 				is $\frac{1}{2}$. The long-run behavior of $p$ is therefore similar to that of $x^3$.
 				\item The zeros of $p$ are $-3$, $1$, and $4$; each zero is simple (i.e, it has multiplicity $1$).
 				This means that the curve of $p$ cuts the horizontal axis at each zero. The vertical
 				intercept of $p$ is $(0,6)$.
 				\item We draw the details we have obtained so far on \cref{poly:fig:simplecubicp1}. Given
 				that the curve of $p$ looks like the curve of $x^3$ in the long-run, we are able to complete a sketch of the
 				graph of $p$ in \cref{poly:fig:simplecubicp2}.
 				 				 				
 				Note that we can not find the coordinates of the local minimums, local maximums, and inflection
 				points| for the moment we make reasonable guesses as to where these points are (you'll find how
 				to do this in calculus).
 			\end{steps}
 			 			 			
 			\begin{figure}[!htbp]
 				\begin{subfigure}{.45\textwidth}
 					\begin{tikzpicture}
 						\begin{axis}[
 								xmin=-10,xmax=10,
 								ymin=-10,ymax=15,
 								xtick={-8,-6,...,8},
 								ytick={-5,5},
 								width=\textwidth,
 							]
 							\addplot[soldot] coordinates{(-3,0)(1,0)(4,0)(0,6)}node[axisnode,anchor=south west]{$(0,6)$};
 						\end{axis}
 					\end{tikzpicture}
 					\caption{}
 					\label{poly:fig:simplecubicp1}
 				\end{subfigure}%
 				\hfill
 				\begin{subfigure}{.45\textwidth}
 					\begin{tikzpicture}
 						\begin{axis}[
 								xmin=-10,xmax=10,
 								ymin=-10,ymax=15,
 								xtick={-8,-6,...,8},
 								ytick={-5,5},
 								width=\textwidth,
 							]
 							\addplot[soldot] coordinates{(-3,0)(1,0)(4,0)(0,6)}node[axisnode,anchor=south west]{$(0,6)$};
 							\addplot[pccplot] expression[domain=-3.57675:4.95392,samples=100]{.5*(x-4)*(x-1)*(x+3)};
 						\end{axis}
 					\end{tikzpicture}
 					\caption{}
 					\label{poly:fig:simplecubicp2}
 				\end{subfigure}%
 				\caption{$y=\dfrac{1}{2}(x-4)(x-1)(x+3)$}
 				\label{poly:fig:simplecubic}
 			\end{figure}
 		\end{pccsolution}
 	\end{pccexample}
 	 	 	
 	%===================================
 	%   Author: Hughes
 	%   Date:   May 2012
 	%===================================
 	\begin{pccexample}\label{poly:ex:degree5}
 		Use \crefrange{poly:step:first}{poly:step:last} to sketch a graph of the function $q$
 		that has formula
 		\[
 			q(x)=\frac{1}{200}(x+7)^2(2-x)(x-6)^2
 		\]
 		\begin{pccsolution}
 			\begin{steps}
 				\item $q$ has degree $4$. The leading term of $q$ is
 				\[
 					-\frac{1}{200}x^5
 				\]
 				so the leading coefficient of $q$ is $-\frac{1}{200}$. The long-run behavior of $q$
 				is therefore similar to that of $-x^5$.
 				\item The zeros of $q$ are $-7$ (multiplicity 2), $2$ (simple), and $6$ (multiplicity $2$).
 				The curve of $q$ bounces off the horizontal axis at the zeros with multiplicity $2$ and
 				cuts the horizontal axis at the simple zeros. The vertical intercept of $q$ is $\left( 0,\frac{441}{25} \right)$.
 				\item We mark the details we have found so far on \cref{poly:fig:degree5p1}. Given that
 				the curve of $q$ looks like the curve of $-x^5$ in the long-run, we can complete \cref{poly:fig:degree5p2}.
 			\end{steps}
 			 			 			
 			\begin{figure}[!htbp]
 				\begin{subfigure}{.45\textwidth}
 					\begin{tikzpicture}
 						\begin{axis}[
 								xmin=-10,xmax=10,
 								ymin=-10,ymax=40,
 								xtick={-8,-6,...,8},
 								ytick={-5,0,...,35},
 								width=\textwidth,
 							]
 							\addplot[soldot] coordinates{(-7,0)(2,0)(6,0)(0,441/25)}node[axisnode,anchor=south west]{$\left( 0, \frac{441}{25} \right)$};
 						\end{axis}
 					\end{tikzpicture}
 					\caption{}
 					\label{poly:fig:degree5p1}
 				\end{subfigure}%
 				\hfill
 				\begin{subfigure}{.45\textwidth}
 					\begin{tikzpicture}
 						\begin{axis}[
 								xmin=-10,xmax=10,
 								ymin=-10,ymax=40,
 								xtick={-8,-6,...,8},
 								ytick={-5,0,...,35},
 								width=\textwidth,
 							]
 							\addplot[soldot] coordinates{(-7,0)(2,0)(6,0)(0,441/25)}node[axisnode,anchor=south west]{$\left( 0, \frac{441}{25} \right)$};
 							\addplot[pccplot] expression[domain=-8.83223:7.34784,samples=50]{1/200*(x+7)^2*(2-x)*(x-6)^2};
 						\end{axis}
 					\end{tikzpicture}
 					\caption{}
 					\label{poly:fig:degree5p2}
 				\end{subfigure}%
 				\caption{$y=\dfrac{1}{200}(x+7)^2(2-x)(x-6)^2$}
 				\label{poly:fig:degree5}
 			\end{figure}
 		\end{pccsolution}
 	\end{pccexample}
 	 	 	
 	%===================================
 	%   Author: Hughes
 	%   Date:   May 2012
 	%===================================
 	\begin{pccexample}
 		Use \crefrange{poly:step:first}{poly:step:last} to sketch a graph of the function $r$
 		that has formula
 		\[
 			r(x)=\frac{1}{100}x^3(x+4)(x-4)(x-6)
 		\]
 		\begin{pccsolution}
 			\begin{steps}
 				\item $r$ has degree $6$. The leading term of $r$ is
 				\[
 					\frac{1}{100}x^6
 				\]
 				so the leading coefficient of $r$ is $\frac{1}{100}$. The long-run behavior of $r$
 				is therefore similar to that of $x^6$.
 				\item The zeros of $r$ are $-4$ (simple), $0$ (multiplicity $3$), $4$ (simple),
 				and $6$ (simple). The vertical intercept of $r$ is $(0,0)$. The curve of $r$
 				cuts the horizontal axis at the simple zeros, and goes through the axis
 				at $(0,0)$, but does so in a flattened way.
 				\item We mark the zeros and vertical intercept on \cref{poly:fig:degree6p1}. Given that
 				the curve of $r$ looks like the curve of $x^6$ in the long-run, we complete the graph
 				of $r$ in \cref{poly:fig:degree6p2}.
 			\end{steps}
 			 			 			
 			\begin{figure}[!htbp]
 				\begin{subfigure}{.45\textwidth}
 					\begin{tikzpicture}
 						\begin{axis}[
 								xmin=-5,xmax=10,
 								ymin=-20,ymax=10,
 								xtick={-4,-2,...,8},
 								ytick={-15,-10,...,5},
 								width=\textwidth,
 							]
 							\addplot[soldot] coordinates{(-4,0)(0,0)(4,0)(6,0)};
 						\end{axis}
 					\end{tikzpicture}
 					\caption{}
 					\label{poly:fig:degree6p1}
 				\end{subfigure}%
 				\hfill
 				\begin{subfigure}{.45\textwidth}
 					\begin{tikzpicture}
 						\begin{axis}[
 								xmin=-5,xmax=10,
 								ymin=-20,ymax=10,
 								xtick={-4,-2,...,8},
 								ytick={-15,-10,...,5},
 								width=\textwidth,
 							]
 							\addplot[soldot] coordinates{(-4,0)(0,0)(4,0)(6,0)};
 							\addplot[pccplot] expression[domain=-4.16652:6.18911,samples=100]{1/100*(x+4)*x^3*(x-4)*(x-6)};
 						\end{axis}
 					\end{tikzpicture}
 					\caption{}
 					\label{poly:fig:degree6p2}
 				\end{subfigure}%
 				\caption{$y=\dfrac{1}{100}(x+4)x^3(x-4)(x-6)$}
 			\end{figure}
 		\end{pccsolution}
 	\end{pccexample}
 	 	 	
 	%===================================
 	%   Author: Hughes
 	%   Date:   March 2012
 	%===================================
 	\begin{pccexample}[An open-topped box]
 		A cardboard company makes open-topped boxes for their clients. The specifications
 		dictate that the box must have a square base, and that it must be open-topped.
 		The company uses sheets of cardboard that are $\unit[1200]{cm^2}$. Assuming that
 		the base of each box has side $x$ (measured in cm), it can be shown that the volume of each box, $V(x)$,
 		has formula
 		\[
 			V(x)=\frac{x}{4}(1200-x^2)
 		\]
 		Find the dimensions of the box that maximize the volume.
 		\begin{pccsolution}
 			We graph $y=V(x)$ in \cref{poly:fig:opentoppedbox}. Note that because
 			$x$ represents the length of a side, and $V(x)$ represents the volume
 			of the box, we necessarily require both values to be positive; we illustrate
 			the part of the curve that applies to this problem using a solid line.
 			 			 			
 			\begin{figure}[!htb]
 				\centering
 				\begin{tikzpicture}
 					\begin{axis}[framed,
 							xmin=-50,xmax=50,
 							ymin=-5000,ymax=5000,
 							xtick={-40,-30,...,40},
 							minor xtick={-45,-35,...,45},
 							minor ytick={-3000,-1000,1000,3000},
 							width=.75\textwidth,
 							height=.5\textwidth,
 						grid=both]
 						\addplot[pccplot,dashed,<-] expression[domain=-40:0,samples=50]{x/4*(1200-x^2)};
 						\addplot[pccplot,-] expression[domain=0:34.64,samples=50]{x/4*(1200-x^2)};
 						\addplot[pccplot,dashed,->] expression[domain=34.64:40,samples=50]{x/4*(1200-x^2)};
 						\addplot[soldot] coordinates{(20,4000)};
 					\end{axis}
 				\end{tikzpicture}
 				\caption{$y=V(x)$}
 				\label{poly:fig:opentoppedbox}
 			\end{figure}
 			 			 			
 			According to \cref{poly:fig:opentoppedbox}, the maximum volume of such a box is
 			approximately $\unit[4000]{cm^2}$, and we achieve it using a base of length
 			approximately $\unit[20]{cm}$. Since the base is square and each sheet of cardboard
 			is $\unit[1200]{cm^2}$, we conclude that the dimensions of each box are $\unit[20]{cm}\times\unit[20]{cm}\times\unit[30]{cm}$.
 		\end{pccsolution}
 	\end{pccexample}
 	 	 	
 \subsection*{Complex zeros}
 	There has been a pattern to all of the examples that we have seen so far|
 	the degree of the polynomial has dictated the number of \emph{real} zeros that the
 	polynomial has. For example, the function $p$ in \cref{poly:ex:simplecubic}
 	has degree $3$, and $p$ has $3$ real zeros; the function $q$ in \cref{poly:ex:degree5}
 	has degree $5$ and $q$ has $5$ real zeros.
 	 	 	
 	You may wonder if this result can be generalized| does every polynomial that
 	has degree $n$ have $n$ real zeros? Before we tackle the general result,
 	let's consider an example that may help motivate it.
 	%===================================
 	%   Author: Hughes
 	%   Date:   June 2012
 	%===================================
 	\begin{pccexample}\label{poly:ex:complx}
 		Consider the polynomial function $c$ that has formula
 		\[
 			c(x)=x(x^2+1)
 		\]
 		It is clear that $c$ has degree $3$, and that $c$ has a (simple) zero at $0$. Does
 		$c$ have any other zeros, i.e, can we find any values of $x$ that satisfy the equation
 		\begin{equation}\label{poly:eq:complx}
 			x^2+1=0
 		\end{equation}
 		The solutions to \cref{poly:eq:complx} are $\pm i$.
 		 		 		
 		We conclude that $c$ has $3$ zeros: $0$ and $\pm i$; we note that \emph{not
 		all of them are real}.
 	\end{pccexample}
 	\Cref{poly:ex:complx} shows that not every degree-$3$ polynomial has $3$
 	\emph{real} zeros; however, if we are prepared to venture into the complex numbers,
 	then we can state the following theorem.
 	%===================================
 	%   Author: Hughes
 	%   Date:   June 2012
 	%===================================
 	\begin{pccspecialcomment}[The fundamental theorem of algebra]
 		Every polynomial function of degree $n$ has $n$ roots, some of which may
 		be complex, and some may be repeated.
 	\end{pccspecialcomment}
 	\fixthis{Fundamental theorem of algebra: is this wording ok? do we want
 	it as a theorem?}
 	%===================================
 	%   Author: Hughes
 	%   Date:   June 2012
 	%===================================
 	\begin{pccexample}
 		Find all the zeros of the polynomial function $p$ that has formula
 		\[
 			p(x)=x^4-2x^3+5x^2
 		\]
 		\begin{pccsolution}
 			We begin by factoring $p$
 			\begin{align*}
 				p(x) & =x^4-2x^3+5x^2 \\
 				     & =x^2(x^2-2x+5) 
 			\end{align*}
 			We note that $0$ is a zero of $p$ with multiplicity $2$. The other zeros of $p$
 			can be found by solving the equation
 			\[
 				x^2-2x+5=0
 			\]
 			This equation can not be factored, so we use the quadratic formula
 			\begin{align*}
 				x & =\frac{2\pm\sqrt{(-2)^2}-20}{2(1)} \\
 				  & =\frac{2\pm\sqrt{-16}}{2}          \\
 				  & =1\pm 2i                           
 			\end{align*}
 			We conclude that $p$ has $4$ zeros: $0$ (multiplicity $2$), and $1\pm 2i$ (simple).
 		\end{pccsolution}
 	\end{pccexample}
 	%===================================
 	%   Author: Hughes
 	%   Date:   June 2012
 	%===================================
 	\begin{pccexample}
 		Find a polynomial that has zeros at $2\pm i\sqrt{2}$.
 		\begin{pccsolution}
 			We know that the zeros of a polynomial can be found by analyzing the linear
 			factors. We are given the zeros, and have to work backwards to find the
 			linear factors.
 			 			 			
 			We begin by assuming that $p$ has the form
 			\begin{align*}
 				p(x) & =(x-(2-i\sqrt{2}))(x-(2+i\sqrt{2}))                           \\
 				     & =x^2-x(2+i\sqrt{2})-x(2-i\sqrt{2})+(2-i\sqrt{2})(2+i\sqrt{2}) \\
 				     & =x^2-4x+(4-2i^2)                                              \\
 				     & =x^2-4x+6                                                     
 			\end{align*}
 			We conclude that a possible formula for a polynomial function, $p$,
 			that has zeros at $2\pm i\sqrt{2}$ is
 			\[
 				p(x)=x^2-4x+6
 			\]
 			Note that we could multiply $p$ by any real number and still ensure
 			that $p$ has the same zeros.
 		\end{pccsolution}
 	\end{pccexample}
 	\investigation*{}
 	%===================================
 	%   Author: Hughes
 	%   Date:   May 2011
 	%===================================
 	\begin{problem}[Find a formula from a graph]
 	For each of the polynomials in \cref{poly:fig:findformula}
 	\begin{enumerate}
 		\item count the number of times the curve turns round, and cuts/bounces off the $x$ axis;
 		\item approximate the degree of the polynomial;
 		\item use your information to find the linear factors of each polynomial, and therefore write a possible formula for each;
 		\item make sure your polynomial goes through the given ordered pair.
 	\end{enumerate}
 	\begin{shortsolution}
 		\Vref{poly:fig:findformdeg2}:
 		\begin{enumerate}
 			\item the curve turns round once;
 			\item the degree could be 2;
 			\item based on the zeros, the linear factors are $(x+5)$ and $(x-3)$; since the
 			      graph opens downwards, we will assume the leading coefficient is negative: $p(x)=-k(x+5)(x-3)$;
 			\item $p$ goes through $(2,2)$, so we need to solve $2=-k(7)(-1)$ and therefore $k=\nicefrac{2}{7}$, so
 			      \[
 			      	p(x)=-\frac{2}{7}(x+5)(x-3)
 			      \]
 		\end{enumerate}
 		\Vref{poly:fig:findformdeg3}:
 		\begin{enumerate}
 			\item the curve turns around twice;
 			\item the degree could be 3;
 			\item based on the zeros, the linear factors are $(x+2)^2$, and $(x-1)$;
 			      based on the behavior of $p$, we assume that the leading coefficient is positive, and try $p(x)=k(x+2)^2(x-1)$;
 			\item $p$ goes through $(0,-2)$, so we need to solve $-2=k(4)(-1)$ and therefore $k=\nicefrac{1}{2}$, so
 			      \[
 			      	p(x)=\frac{1}{2}(x+2)^2(x-1)
 			      \]
 		\end{enumerate}
 		\Vref{poly:fig:findformdeg5}:
 		\begin{enumerate}
 			\item the curve turns around 4 times;
 			\item the degree could be 5;
 			\item based on the zeros, the linear factors are $(x+5)^2$, $(x+1)$, $(x-2)$, $(x-3)$;
 			      based on the behavior of $p$, we assume that the leading coefficient is positive, and try $p(x)=k(x+5)^2(x+1)(x-2)(x-3)$;
 			\item $p$ goes through $(-3,-50)$, so we need to solve $-50=k(64)(-2)(-5)(-6)$ and therefore $k=\nicefrac{5}{384}$, so
 			      \[
 			      	p(x)=\frac{5}{384}(x+5)^2(x+1)(x-2)(x-3)
 			      \]
 		\end{enumerate}
 	\end{shortsolution}
 	\end{problem}
 	 	 	
 	 	 	
 	\begin{figure}[!htb]
 		\setlength{\figurewidth}{0.3\textwidth}
 		\begin{subfigure}{\figurewidth}
 			\begin{tikzpicture}
 				\begin{axis}[
 						xmin=-5,xmax=5,
 						ymin=-2,ymax=5,
 						width=\textwidth,
 					]
 					\addplot expression[domain=-4.5:3.75]{-1/3*(x+4)*(x-3)};
 					\addplot[soldot] coordinates{(-4,0)(3,0)(2,2)} node[axisnode,above right]{$(2,2)$};
 				\end{axis}
 			\end{tikzpicture}
 			\caption{}
 			\label{poly:fig:findformdeg2}
 		\end{subfigure}
 		\hfill
 		\begin{subfigure}{\figurewidth}
 			\begin{tikzpicture}
 				\begin{axis}[
 						xmin=-3,xmax=2,
 						ymin=-2,ymax=4,
 						xtick={-2,...,1},
 						width=\textwidth,
 					]
 					\addplot expression[domain=-2.95:1.75]{1/3*(x+2)^2*(x-1)};
 					\addplot[soldot]coordinates{(-2,0)(1,0)(0,-1.33)}node[axisnode,anchor=north west]{$(0,-2)$};
 				\end{axis}
 			\end{tikzpicture}
 			\caption{}
 			\label{poly:fig:findformdeg3}
 		\end{subfigure}
 		\hfill
 		\begin{subfigure}{\figurewidth}
 			\begin{tikzpicture}
 				\begin{axis}[
 						xmin=-5,xmax=5,
 						ymin=-100,ymax=150,
 						width=\textwidth,
 					]
 					\addplot expression[domain=-4.5:3.4,samples=50]{(x+4)^2*(x+1)*(x-2)*(x-3)};
 					\addplot[soldot]coordinates{(-4,0)(-1,0)(2,0)(3,0)(-3,-60)}node[axisnode,anchor=north]{$(-3,-50)$};
 				\end{axis}
 			\end{tikzpicture}
 			\caption{}
 			\label{poly:fig:findformdeg5}
 		\end{subfigure}
 		\caption{}
 		\label{poly:fig:findformula}
 	\end{figure}
 	 	 	
 	 	 	
 	 	 	
 	 	 	
 	\begin{exercises}
 	%===================================
 	%   Author: Hughes
 	%   Date:   March 2012
 	%===================================
 	\begin{problem}[Prerequisite classifacation skills]
 	Decide if each of the following functions are linear or quadratic.
 	\begin{multicols}{3}
 		\begin{subproblem}
 			$f(x)=2x+3$
 			\begin{shortsolution}
 				$f$ is linear.
 			\end{shortsolution}
 		\end{subproblem}
 		\begin{subproblem}
 			$g(x)=10-7x$
 			\begin{shortsolution}
 				$g$ is linear
 			\end{shortsolution}
 		\end{subproblem}
 		\begin{subproblem}
 			$h(x)=-x^2+3x-9$
 			\begin{shortsolution}
 				$h$ is quadratic.
 			\end{shortsolution}
 		\end{subproblem}
 		\begin{subproblem}
 			$k(x)=-17$
 			\begin{shortsolution}
 				$k$ is linear.
 			\end{shortsolution}
 		\end{subproblem}
 		\begin{subproblem}
 			$l(x)=-82x^2-4$
 			\begin{shortsolution}
 				$l$ is quadratic
 			\end{shortsolution}
 		\end{subproblem}
 		\begin{subproblem}
 			$m(x)=6^2x-8$
 			\begin{shortsolution}
 				$m$ is linear.
 			\end{shortsolution}
 		\end{subproblem}
 	\end{multicols}
 	\end{problem}
 	%===================================
 	%   Author: Hughes
 	%   Date:   March 2012
 	%===================================
 	\begin{problem}[Prerequisite slope identification]
 	State the slope of each of the following linear functions, and
 	hence decide if each function is increasing or decreasing.
 	\begin{multicols}{4}
 		\begin{subproblem}
 			$\alpha(x)=4x+1$
 			\begin{shortsolution}
 				$m=4$; $\alpha$ is increasing.
 			\end{shortsolution}
 		\end{subproblem}
 		\begin{subproblem}
 			$\beta(x)=-9x$
 			\begin{shortsolution}
 				$m=-9$; $\beta$ is decreasing.
 			\end{shortsolution}
 		\end{subproblem}
 		\begin{subproblem}
 			$\gamma(t)=18t+100$
 			\begin{shortsolution}
 				$m=18$; $\gamma$ is increasing.
 			\end{shortsolution}
 		\end{subproblem}
 		\begin{subproblem}
 			$\delta(y)=23-y$
 			\begin{shortsolution}
 				$m=-1$; $\delta$ is decreasing.
 			\end{shortsolution}
 		\end{subproblem}
 	\end{multicols}
 	Now let's generalize our findings for the most general linear function $f$
 	that has formula $f(x)=mx+b$. Complete the following sentences.
 	\begin{subproblem}
 		When $m>0$, the function $f$ is $\ldots$
 		\begin{shortsolution}
 			When $m>0$, the function $f$ is $\ldots$  \emph{increasing}.
 		\end{shortsolution}
 	\end{subproblem}
 	\begin{subproblem}
 		When $m<0$, the function $f$ is $\ldots$
 		\begin{shortsolution}
 			When $m<0$, the function $f$ is $\ldots$  \emph{decreasing}.
 		\end{shortsolution}
 	\end{subproblem}
 	\end{problem}
 	%===================================
 	%   Author: Hughes
 	%   Date:   May 2011
 	%===================================
 	\begin{problem}[Polynomial or not?]
 	Identify whether each of the following functions is a polynomial or not.
 	If the function is a polynomial, state its degree.
 	\begin{multicols}{3}
 		\begin{subproblem}
 			$p(x)=2x+1$
 			\begin{shortsolution}
 				$p$ is a polynomial (you might also describe $p$ as linear). The degree of $p$ is 1.
 			\end{shortsolution}
 		\end{subproblem}
 		\begin{subproblem}
 			$p(x)=7x^2+4x$
 			\begin{shortsolution}
 				$p$ is a polynomial (you might also describe $p$ as quadratic). The degree of $p$ is 2.
 			\end{shortsolution}
 		\end{subproblem}
 		\begin{subproblem}
 			$p(x)=\sqrt{x}+2x+1$
 			\begin{shortsolution}
 				$p$ is not a polynomial; we require the powers of $x$ to be integer values.
 			\end{shortsolution}
 		\end{subproblem}
 		\begin{subproblem}
 			$p(x)=2^x-45$
 			\begin{shortsolution}
 				$p$ is not a polynomial; the $2^x$ term is exponential.
 			\end{shortsolution}
 		\end{subproblem}
 		\begin{subproblem}
 			$p(x)=6x^4-5x^3+9$
 			\begin{shortsolution}
 				$p$ is a polynomial, and the degree of $p$ is $6$.
 			\end{shortsolution}
 		\end{subproblem}
 		\begin{subproblem}
 			$p(x)=-5x^{17}+9x+2$
 			\begin{shortsolution}
 				$p$ is a polynomial, and the degree of $p$ is 17.
 			\end{shortsolution}
 		\end{subproblem}
 		\begin{subproblem}
 			$p(x)=4x(x+7)^2(x-3)^3$
 			\begin{shortsolution}
 				$p$ is a polynomial, and the degree of $p$ is $6$.
 			\end{shortsolution}
 		\end{subproblem}
 		\begin{subproblem}
 			$p(x)=4x^{-5}-x^2+x$
 			\begin{shortsolution}
 				$p$ is not a polynomial because $-5$ is not a positive integer.
 			\end{shortsolution}
 		\end{subproblem}
 		\begin{subproblem}
 			$p(x)=-x^6(x^2+1)(x^3-2)$
 			\begin{shortsolution}
 				$p$ is a polynomial, and the degree of $p$ is $11$.
 			\end{shortsolution}
 		\end{subproblem}
 	\end{multicols}
 	\end{problem}
 	%===================================
 	%   Author: Hughes
 	%   Date:   May 2011
 	%===================================
 	\begin{problem}[Polynomial graphs]
 	Three polynomial functions $p$, $m$, and $n$ are shown in \crefrange{poly:fig:functionp}{poly:fig:functionn}.
 	The functions have the following formulas
 	\begin{align*}
 		p(x) & = (x-1)(x+2)(x-3)           \\
 		m(x) & = -(x-1)(x+2)(x-3)          \\
 		n(x) & = (x-1)(x+2)(x-3)(x+1)(x+4) 
 	\end{align*}
 	Note that for our present purposes we are not concerned with the vertical scale of the graphs.
 	\begin{subproblem}
 		Identify both on the graph {\em and} algebraically, the zeros of each polynomial.
 		\begin{shortsolution}
 			$y=p(x)$ is shown below.
 			 			 			
 			\begin{tikzpicture}
 				\begin{axis}[
 						xmin=-5,xmax=5,
 						ymin=-10,ymax=10,
 						width=\solutionfigurewidth,
 					]
 					\addplot expression[domain=-2.5:3.5,samples=50]{(x-1)*(x+2)*(x-3)};
 					\addplot[soldot] coordinates{(-2,0)(1,0)(3,0)};
 				\end{axis}
 			\end{tikzpicture}
 			 			 			
 			$y=m(x)$ is shown below.
 			 			 			
 			\begin{tikzpicture}
 				\begin{axis}[
 						xmin=-5,xmax=5,
 						ymin=-10,ymax=10,
 						width=\solutionfigurewidth,
 					]
 					\addplot expression[domain=-2.5:3.5,samples=50]{-1*(x-1)*(x+2)*(x-3)};
 					\addplot[soldot] coordinates{(-2,0)(1,0)(3,0)};
 				\end{axis}
 			\end{tikzpicture}
 			 			 			
 			$y=n(x)$ is shown below.
 			 			 			
 			\begin{tikzpicture}
 				\begin{axis}[
 						xmin=-5,xmax=5,
 						ymin=-90,ymax=70,
 						width=\solutionfigurewidth,
 					]
 					\addplot expression[domain=-4.15:3.15,samples=50]{(x-1)*(x+2)*(x-3)*(x+1)*(x+4)};
 					\addplot[soldot] coordinates{(-4,0)(-2,0)(-1,0)(1,0)(3,0)};
 				\end{axis}
 			\end{tikzpicture}
 			 			 			
 			The zeros of $p$ are $-2$, $1$, and $3$; the zeros of $m$ are $-2$, $1$, and $3$; the zeros of $n$ are
 			$-4$, $-2$, $-1$, and $3$.
 		\end{shortsolution}
 	\end{subproblem}
 	\begin{subproblem}
 		Write down the degree, how many times the curve of each function `turns around',
 		and how many zeros it has
 		\begin{shortsolution}
 			\begin{itemize}
 				\item The degree of $p$ is 3, and the curve $y=p(x)$ turns around twice.
 				\item The degree of $q$ is also 3, and the curve $y=q(x)$ turns around twice.
 				\item The degree of $n$ is $5$, and the curve $y=n(x)$ turns around 4 times.
 			\end{itemize}
 		\end{shortsolution}
 	\end{subproblem}
 	\end{problem}
 	 	 	
 	\begin{figure}[!htb]
 		\begin{widepage}
 		\setlength{\figurewidth}{0.3\textwidth}
 		\begin{subfigure}{\figurewidth}
 			\begin{tikzpicture}
 				\begin{axis}[
 						xmin=-5,xmax=5,
 						ymin=-10,ymax=10,
 						ytick={-5,5},
 						width=\textwidth,
 					]
 					\addplot expression[domain=-2.5:3.5,samples=50]{(x-1)*(x+2)*(x-3)};
 					\addplot[soldot]coordinates{(-2,0)(1,0)(3,0)};
 				\end{axis}
 			\end{tikzpicture}
 			\caption{$y=p(x)$}
 			\label{poly:fig:functionp}
 		\end{subfigure}
 		\hfill
 		\begin{subfigure}{\figurewidth}
 			\begin{tikzpicture}
 				\begin{axis}[
 						xmin=-5,xmax=5,
 						ymin=-10,ymax=10,
 						ytick={-5,5},
 						width=\textwidth,
 					]
 					\addplot expression[domain=-2.5:3.5,samples=50]{-1*(x-1)*(x+2)*(x-3)};
 					\addplot[soldot]coordinates{(-2,0)(1,0)(3,0)};
 				\end{axis}
 			\end{tikzpicture}
 			\caption{$y=m(x)$}
 			\label{poly:fig:functionm}
 		\end{subfigure}
 		\hfill
 		\begin{subfigure}{\figurewidth}
 			\begin{tikzpicture}
 				\begin{axis}[
 						xmin=-5,xmax=5,
 						ymin=-90,ymax=70,
 						width=\textwidth,
 					]
 					\addplot expression[domain=-4.15:3.15,samples=100]{(x-1)*(x+2)*(x-3)*(x+1)*(x+4)};
 					\addplot[soldot]coordinates{(-4,0)(-2,0)(-1,0)(1,0)(3,0)};
 				\end{axis}
 			\end{tikzpicture}
 			\caption{$y=n(x)$}
 			\label{poly:fig:functionn}
 		\end{subfigure}
 		\caption{}
 		\end{widepage}
 	\end{figure}
 	%===================================
 	%   Author: Hughes
 	%   Date:   May 2011
 	%===================================
 	\begin{problem}[Horizontal intercepts]\label{poly:prob:matchpolys}%
 	State the horizontal intercepts (as ordered pairs) of the following polynomials.
 	\begin{multicols}{2}
 		\begin{subproblem}\label{poly:prob:degree5}
 			$p(x)=(x-1)(x+2)(x-3)(x+1)(x+4)$
 			\begin{shortsolution}
 				$(-4,0)$, $(-2,0)$, $(-1,0)$, $(1,0)$, $(3,0)$
 			\end{shortsolution}
 		\end{subproblem}
 		\begin{subproblem}
 			$q(x)=-(x-1)(x+2)(x-3)$
 			\begin{shortsolution}
 				$(-2,0)$, $(1,0)$, $(3,0)$
 			\end{shortsolution}
 		\end{subproblem}
 		\begin{subproblem}
 			$r(x)=(x-1)(x+2)(x-3)$
 			\begin{shortsolution}
 				$(-2,0)$, $(1,0)$, $(3,0)$
 			\end{shortsolution}
 		\end{subproblem}
 		\begin{subproblem}\label{poly:prob:degree2}
 			$s(x)=(x-2)(x+2)$
 			\begin{shortsolution}
 				$(-2,0)$, $(2,0)$
 			\end{shortsolution}
 		\end{subproblem}
 	\end{multicols}
 	\end{problem}
 	%===================================
 	%   Author: Hughes
 	%   Date:   March 2012
 	%===================================
 	\begin{problem}[Minimums, maximums, and concavity]\label{poly:prob:incdec}
 	Four polynomial functions are graphed in \cref{poly:fig:incdec}. The formulas
 	for these functions are (not respectively)
 	\begin{gather*}
 		p(x)=\frac{x^3}{6}-\frac{x^2}{4}-3x, \qquad q(x)=\frac{x^4}{20}+\frac{x^3}{15}-\frac{6}{5}x^2+1\\
 		r(x)=-\frac{x^5}{50}-\frac{x^4}{40}+\frac{2x^3}{5}+6, \qquad s(x)=-\frac{x^6}{6000}-\frac{x^5}{2500}+\frac{67x^4}{4000}+\frac{17x^3}{750}-\frac{42x^2}{125}
 	\end{gather*}
 	\begin{figure}[!htb]
 		\begin{widepage}
 		\setlength{\figurewidth}{.23\textwidth}
 		\centering
 		\begin{subfigure}{\figurewidth}
 			\begin{tikzpicture}
 				\begin{axis}[
 						framed,
 						width=\textwidth,
 						xmin=-10,xmax=10,
 						ymin=-10,ymax=10,
 						xtick={-8,-6,...,8},
 						ytick={-8,-6,...,8},
 						grid=major,
 					]
 					\addplot expression[domain=-5.28:4.68,samples=50]{-x^5/50-x^4/40+2*x^3/5+6};
 				\end{axis}
 			\end{tikzpicture}
 			\caption{}
 			\label{poly:fig:incdec3}
 		\end{subfigure}
 		\hfill
 		\begin{subfigure}{\figurewidth}
 			\begin{tikzpicture}
 				\begin{axis}[
 						framed,
 						width=\textwidth,
 						xmin=-10,xmax=10,ymin=-10,ymax=10,
 						xtick={-8,-6,...,8},
 						ytick={-8,-6,...,8},
 						grid=major,
 					]
 					\addplot expression[domain=-6.08:4.967,samples=50]{x^4/20+x^3/15-6/5*x^2+1};
 				\end{axis}
 			\end{tikzpicture}
 			\caption{}
 			\label{poly:fig:incdec2}
 		\end{subfigure}
 		\hfill
 		\begin{subfigure}{\figurewidth}
 			\begin{tikzpicture}
 				\begin{axis}[
 						framed,
 						width=\textwidth,
 						xmin=-6,xmax=8,ymin=-10,ymax=10,
 						xtick={-4,-2,...,6},
 						ytick={-8,-4,4,8},
 						minor ytick={-6,-2,...,6},
 						grid=both,
 					]
 					\addplot expression[domain=-4.818:6.081,samples=50]{x^3/6-x^2/4-3*x};
 				\end{axis}
 			\end{tikzpicture}
 			\caption{}
 			\label{poly:fig:incdec1}
 		\end{subfigure}
 		\hfill
 		\begin{subfigure}{\figurewidth}
 			\begin{tikzpicture}
 				\begin{axis}[
 						framed,
 						width=\textwidth,
 						xmin=-10,xmax=10,ymin=-10,ymax=10,
 						xtick={-8,-4,4,8},
 						ytick={-8,-4,4,8},
 						minor xtick={-6,-2,...,6},
 						minor ytick={-6,-2,...,6},
 						grid=both,
 					]
 					\addplot expression[domain=-9.77:8.866,samples=50]{-x^6/6000-x^5/2500+67*x^4/4000+17/750*x^3-42/125*x^2};
 				\end{axis}
 			\end{tikzpicture}
 			\caption{}
 			\label{poly:fig:incdec4}
 		\end{subfigure}
 		\caption{Graphs for \cref{poly:prob:incdec}.}
 		\label{poly:fig:incdec}
 		\end{widepage}
 	\end{figure}
 	\begin{subproblem}
 		Match each of the formulas with one of the given graphs.
 		\begin{shortsolution}
 			\begin{itemize}
 				\item $p$ is graphed in \vref{poly:fig:incdec1};
 				\item $q$ is graphed in \vref{poly:fig:incdec2};
 				\item $r$ is graphed in \vref{poly:fig:incdec3};
 				\item $s$ is graphed in \vref{poly:fig:incdec4}.
 			\end{itemize}
 		\end{shortsolution}
 	\end{subproblem}
 	\begin{subproblem}
 		Approximate the zeros of each function using the appropriate graph.
 		\begin{shortsolution}
 			\begin{itemize}
 				\item $p$ has simple zeros at about $-3.8$, $0$, and $5$.
 				\item $q$ has simple zeros at about $-5.9$, $-1$, $1$, and $4$.
 				\item $r$ has simple zeros at about $-5$, $-2.9$, and $4.1$.
 				\item $s$ has simple zeros at about $-9$, $-6$, $4.2$, $8.1$, and a zero of multiplicity $2$ at $0$.
 			\end{itemize}
 		\end{shortsolution}
 	\end{subproblem}
 	\begin{subproblem}
 		Approximate the local maximums and minimums of each of the functions.
 		\begin{shortsolution}
 			\begin{itemize}
 				\item $p$ has a local maximum of approximately $3.9$ at $-2$, and a local minimum of approximately $-6.5$ at $3$.
 				\item $q$ has a local minimum of approximately $-10$ at $-4$, and $-4$ at $3$; $q$ has a local maximum of approximately $1$ at $0$.
 				\item $r$ has a local minimum of approximately $-5.5$ at $-4$, and a local maximum of approximately $10$ at $3$.
 				\item $s$ has a local maximum of approximately $5$ at $-8$, $0$ at $0$, and $5$ at  $7$; $s$ has local minimums
 				      of approximately $-3$ at $-4$, and $-1$ at $3$.
 			\end{itemize}
 		\end{shortsolution}
 	\end{subproblem}
 	\begin{subproblem}
 		Approximate the global maximums and minimums of each of the functions.
 		\begin{shortsolution}
 			\begin{itemize}
 				\item $p$ does not have a global maximum, nor a global minimum.
 				\item $q$ has a global minimum of approximately $-10$; it does not have a global maximum.
 				\item $r$ does not have a global maximum, nor a global minimum.
 				\item $s$ has a global maximum of approximately $5$; it does not have a global minimum.
 			\end{itemize}
 		\end{shortsolution}
 	\end{subproblem}
 	\begin{subproblem}
 		Approximate the intervals on which each function is increasing and decreasing.
 		\begin{shortsolution}
 			\begin{itemize}
 				\item $p$ is increasing on $(-\infty,-2)\cup (3,\infty)$, and decreasing on $(-2,3)$.
 				\item $q$ is increasing on $(-4,0)\cup (3,\infty)$, and decreasing on $(-\infty,-4)\cup (0,3)$.
 				\item $r$ is increasing on $(-4,3)$, and decreasing on $(-\infty,-4)\cup (3,\infty)$.
 				\item $s$ is increasing on $(-\infty,-8)\cup (-4,0)\cup (3,5)$, and decreasing on $(-8,-4)\cup (0,3)\cup (5,\infty)$.
 			\end{itemize}
 		\end{shortsolution}
 	\end{subproblem}
 	\begin{subproblem}
 		Approximate the intervals on which each function is concave up and concave down.
 		\begin{shortsolution}
 			\begin{itemize}
 				\item $p$ is concave up on  $(1,\infty)$, and concave down on  $(-\infty,1)$.
 				\item $q$ is concave up on $(-\infty,-1)\cup (1,\infty)$, and concave down on $(-1,1)$.
 				\item $r$ is concave up on $(-\infty,-3)\cup (0,2)$, and concave down on $(-3,0)\cup (2,\infty)$.
 				\item $s$ is concave up on $(-6,-2)\cup (2,5)$, and concave down on $(-\infty,-6)\cup (-2,2)\cup (5,\infty)$.
 			\end{itemize}
 		\end{shortsolution}
 	\end{subproblem}
 	\begin{subproblem}
 		The degree of $q$ is $5$. Assuming that all of the real zeros of $q$ are
 		shown in its graph, how many complex zeros does $q$ have?
 		\begin{shortsolution}
 			\Vref{poly:fig:incdec2} shows that $q$ has $3$ real zeros
 			since the curve of $q$ cuts the horizontal axis $3$ times.
 			Since $q$ has degree $5$, $q$ must have $2$ complex zeros.
 		\end{shortsolution}
 	\end{subproblem}
 	\end{problem}
 	 	 	
 	%===================================
 	%   Author: Hughes
 	%   Date:   May 2011
 	%===================================
 	\begin{problem}[Long-run behaviour of polynomials]
 	Describe the long-run behavior of each of polynomial functions in
 	\crefrange{poly:prob:degree5}{poly:prob:degree2}.
 	\begin{shortsolution}
 		$\dd\lim_{x\rightarrow-\infty}p(x)=-\infty$,
 		$\dd\lim_{x\rightarrow\infty}p(x)=\infty$,
 		$\dd\lim_{x\rightarrow-\infty}q(x)=\infty$,
 		$\dd\lim_{x\rightarrow\infty}q(x)=-\infty$,
 		$\dd\lim_{x\rightarrow-\infty}r(x)=-\infty$,
 		$\dd\lim_{x\rightarrow\infty}r(x)=\infty$,
 		$\dd\lim_{x\rightarrow-\infty}s(x)=\infty$,
 		$\dd\lim_{x\rightarrow\infty}s(x)=\infty$,
 	\end{shortsolution}
 	\end{problem}
 	 	 	
 	%===================================
 	%   Author: Hughes
 	%   Date:   May 2011
 	%===================================
 	\begin{problem}[True of false?]
 	Let $p$ be a polynomial function.
 	Label each of the following statements as true (T) or false (F); if they are false,
 	provide an example that supports your answer.
 	\begin{subproblem}
 		If $p$ has degree $3$, then $p$ has $3$ distinct zeros.
 		\begin{shortsolution}
 			False. Consider $p(x)=x^2(x+1)$ which has only 2 distinct zeros.
 		\end{shortsolution}
 	\end{subproblem}
 	\begin{subproblem}
 		If $p$ has degree $4$, then $\dd\lim_{x\rightarrow-\infty}p(x)=\infty$ and $\dd\lim_{x\rightarrow\infty}p(x)=\infty$.
 		\begin{shortsolution}
 			False. Consider $p(x)=-x^4$.
 		\end{shortsolution}
 	\end{subproblem}
 	\begin{subproblem}
 		If $p$ has even degree, then it is possible that $p$ can have no real zeros.
 		\begin{shortsolution}
 			True.
 		\end{shortsolution}
 	\end{subproblem}
 	\begin{subproblem}
 		If $p$ has odd degree, then it is possible that $p$ can have no real zeros.
 		\begin{shortsolution}
 			False. All odd degree polynomials will cut the horizontal axis at least once.
 		\end{shortsolution}
 	\end{subproblem}
 	\end{problem}
 	%===================================
 	%   Author: Hughes
 	%   Date:   May 2011
 	%===================================
 	\begin{problem}[Find a formula from a description]
 	In each of the following problems, give a possible formula for a polynomial
 	function that has the specified properties.
 	\begin{subproblem}
 		Degree 2 and has zeros at $4$ and $5$.
 		\begin{shortsolution}
 			Possible option: $p(x)=(x-4)(x-5)$. Note we could multiply $p$ by any real number, and still meet the requirements.
 		\end{shortsolution}
 	\end{subproblem}
 	\begin{subproblem}
 		Degree 3 and has zeros at $4$,$5$ and $-3$.
 		\begin{shortsolution}
 			Possible option: $p(x)=(x-4)(x-5)(x+3)$. Note we could multiply $p$ by any real number, and still meet the requirements.
 		\end{shortsolution}
 	\end{subproblem}
 	\begin{subproblem}
 		Degree 4 and has zeros at $0$, $4$, $5$, $-3$.
 		\begin{shortsolution}
 			Possible option: $p(x)=x(x-4)(x-5)(x+3)$. Note we could multiply $p$ by any real number, and still meet the requirements.
 		\end{shortsolution}
 	\end{subproblem}
 	\begin{subproblem}
 		Degree 4, with zeros that make the graph cut at $2$, $-5$, and a zero that makes the graph touch at $-2$;
 		\begin{shortsolution}
 			Possible option: $p(x)=(x-2)(x+5)(x+2)^2$. Note we could multiply $p$ by any real number, and still meet the requirements.
 		\end{shortsolution}
 	\end{subproblem}
 	\begin{subproblem}
 		Degree 3, with only one zero at $-1$.
 		\begin{shortsolution}
 			Possible option: $p(x)=(x+1)^3$. Note we could multiply $p$ by any real number, and still meet the requirements.
 		\end{shortsolution}
 	\end{subproblem}
 	\end{problem}
 	%===================================
 	%   Author: Hughes
 	%   Date:   June 2012
 	%===================================
 	\begin{problem}[\Cref{poly:step:last}]
 	\pccname{Saheed} is graphing a polynomial function, $p$.
 	He is following \crefrange{poly:step:first}{poly:step:last} and has so far
 	marked the zeros of $p$ on \cref{poly:fig:optionsp1}. Saheed tells you that
 	$p$ has degree $3$, but does \emph{not} say if the leading coefficient
 	of $p$ is positive or negative.
 	\begin{figure}[!htbp]
 		\begin{widepage}
 		\begin{subfigure}{.45\textwidth}
 			\begin{tikzpicture}
 				\begin{axis}[
 						xmin=-10,xmax=10,
 						ymin=-10,ymax=10,
 						xtick={-8,-6,...,8},
 						ytick={-15},
 						width=\textwidth,
 						height=.5\textwidth,
 					]
 					\addplot[soldot] coordinates{(-5,0)(2,0)(6,0)};
 				\end{axis}
 			\end{tikzpicture}
 			\caption{}
 			\label{poly:fig:optionsp1}
 		\end{subfigure}%
 		\hfill
 		\begin{subfigure}{.45\textwidth}
 			\begin{tikzpicture}
 				\begin{axis}[
 						xmin=-10,xmax=10,
 						ymin=-10,ymax=10,
 						xtick={-8,-6,...,8},
 						ytick={-15},
 						width=\textwidth,
 						height=.5\textwidth,
 					]
 					\addplot[soldot] coordinates{(-5,0)(6,0)};
 				\end{axis}
 			\end{tikzpicture}
 			\caption{}
 			\label{poly:fig:optionsp2}
 		\end{subfigure}%
 		\caption{}
 		\end{widepage}
 	\end{figure}
 	\begin{subproblem}
 		Use the information in \cref{poly:fig:optionsp1} to help sketch $p$, assuming that the leading coefficient
 		is positive.
 		\begin{shortsolution}
 			Assuming that $a_3>0$:
 			 			 			
 			\begin{tikzpicture}
 				\begin{axis}[
 						xmin=-10,xmax=10,
 						ymin=-10,ymax=10,
 						xtick={-8,-6,...,8},
 						ytick={-15},
 						width=\solutionfigurewidth,
 					]
 					\addplot expression[domain=-6.78179:8.35598,samples=50]{1/20*(x+5)*(x-2)*(x-6)};
 					\addplot[soldot] coordinates{(-5,0)(2,0)(6,0)};
 				\end{axis}
 			\end{tikzpicture}
 		\end{shortsolution}
 	\end{subproblem}
 	\begin{subproblem}
 		Use the information in \cref{poly:fig:optionsp1} to help sketch $p$, assuming that the leading coefficient
 		is negative.
 		\begin{shortsolution}
 			Assuming that $a_3<0$:
 			 			 			
 			\begin{tikzpicture}
 				\begin{axis}[
 						xmin=-10,xmax=10,
 						ymin=-10,ymax=10,
 						xtick={-8,-6,...,8},
 						ytick={-15},
 						width=\solutionfigurewidth,
 					]
 					\addplot expression[domain=-6.78179:8.35598,samples=50]{-1/20*(x+5)*(x-2)*(x-6)};
 					\addplot[soldot] coordinates{(-5,0)(2,0)(6,0)};
 				\end{axis}
 			\end{tikzpicture}
 		\end{shortsolution}
 	\end{subproblem}
 	Saheed now turns his attention to another polynomial function, $q$. He finds
 	the zeros of $q$ (there are only $2$) and marks them on \cref{poly:fig:optionsp2}.
 	Saheed knows that $q$ has degree $3$, but doesn't know if the leading
 	coefficient is positive or negative.
 	\begin{subproblem}
 		Use the information in \cref{poly:fig:optionsp2} to help sketch $q$, assuming that the leading
 		coefficient of $q$ is positive. Hint: only one of the zeros is simple.
 		\begin{shortsolution}
 			Assuming that $a_4>0$ there are $2$ different options:
 			 			 			
 			\begin{tikzpicture}
 				\begin{axis}[
 						xmin=-10,xmax=10,
 						ymin=-10,ymax=10,
 						xtick={-8,-6,...,8},
 						ytick={-15},
 						width=\solutionfigurewidth,
 					]
 					\addplot expression[domain=-8.68983:7.31809,samples=50]{1/20*(x+5)^2*(x-6)};
 					\addplot expression[domain=-6.31809:9.68893,samples=50]{1/20*(x+5)*(x-6)^2};
 					\addplot[soldot] coordinates{(-5,0)(6,0)};
 				\end{axis}
 			\end{tikzpicture}
 		\end{shortsolution}
 	\end{subproblem}
 	\begin{subproblem}
 		Use the information in \cref{poly:fig:optionsp2} to help sketch $q$, assuming that the leading
 		coefficient of $q$ is negative.
 		\begin{shortsolution}
 			Assuming that $a_4<0$ there are $2$ different options:
 			 			 			
 			\begin{tikzpicture}
 				\begin{axis}[
 						xmin=-10,xmax=10,
 						ymin=-10,ymax=10,
 						xtick={-8,-6,...,8},
 						ytick={-15},
 						width=\solutionfigurewidth,
 					]
 					\addplot expression[domain=-8.68983:7.31809,samples=50]{-1/20*(x+5)^2*(x-6)};
 					\addplot expression[domain=-6.31809:9.68893,samples=50]{-1/20*(x+5)*(x-6)^2};
 					\addplot[soldot] coordinates{(-5,0)(6,0)};
 				\end{axis}
 			\end{tikzpicture}
 		\end{shortsolution}
 	\end{subproblem}
 	\end{problem}
 	%===================================
 	%   Author: Hughes
 	%   Date:   June 2012
 	%===================================
 	\begin{problem}[Zeros]
 	Find all zeros of each of the following polynomial functions, making
 	sure to detail their multiplicity. Note that
 	you may need to use factoring, or the quadratic formula, or both! Also note
 	that some zeros may be repeated, and some may be complex.
 	\begin{multicols}{3}
 		\begin{subproblem}
 			$p(x)=x^2+1$
 			\begin{shortsolution}
 				$\pm i$ (simple).
 			\end{shortsolution}
 		\end{subproblem}
 		\begin{subproblem}
 			$q(y)=(y^2-9)(y^2-7)$
 			\begin{shortsolution}
 				$\pm 3$, $\pm \sqrt{7}$ (all are simple).
 			\end{shortsolution}
 		\end{subproblem}
 		\begin{subproblem}
 			$r(z)=-4z^3(z^2+3)(z^2+64)$
 			\begin{shortsolution}
 				$0$ (multiplicity $3$), $\pm\sqrt{3}$ (simple), $\pm\sqrt{8}$ (simple).
 			\end{shortsolution}
 		\end{subproblem}
 		\begin{subproblem}
 			$a(x)=x^4-81$
 			\begin{shortsolution}
 				$\pm 3$, $\pm 3i$ (all are simple).
 			\end{shortsolution}
 		\end{subproblem}
 		\begin{subproblem}
 			$b(y)=y^3-8$
 			\begin{shortsolution}
 				$2$, $-1\pm i\sqrt{3}$ (all are simple).
 			\end{shortsolution}
 		\end{subproblem}
 		\begin{subproblem}
 			$c(m)=m^3-m^2$
 			\begin{shortsolution}
 				$0$ (multiplicity $2$), $1$ (simple).
 			\end{shortsolution}
 		\end{subproblem}
 		\begin{subproblem}
 			$h(n)=(n+1)(n^2+4)$
 			\begin{shortsolution}
 				$-1$, $\pm 2i$ (all are simple).
 			\end{shortsolution}
 		\end{subproblem}
 		\begin{subproblem}
 			$f(\alpha)=(\alpha^2-16)(\alpha^2-5\alpha+4)$
 			\begin{shortsolution}
 				$-4$ (simple), $4$ (multiplicity $2$), $1$ (simple).
 			\end{shortsolution}
 		\end{subproblem}
 		\begin{subproblem}
 			$g(\beta)=(\beta^2-25)(\beta^2-5\beta-4)$
 			\begin{shortsolution}
 				$\pm 5$, $\dfrac{5\pm\sqrt{41}}{2}$ (all are simple).
 			\end{shortsolution}
 		\end{subproblem}
 	\end{multicols}
 	\end{problem}
 	%===================================
 	%   Author: Hughes
 	%   Date:   June 2012
 	%===================================
 	\begin{problem}[Given zeros, find a formula]
 	In each of the following problems you are given the zeros of a polynomial.
 	Write a possible formula for each polynomial| you may leave your
 	answer in factored form, but it may not contain complex numbers. Unless
 	otherwise stated, assume that the zeros are simple.
 	\begin{multicols}{3}
 		\begin{subproblem}
 			$1$, $2$
 			\begin{shortsolution}
 				$p(x)=(x-1)(x-2)$
 			\end{shortsolution}
 		\end{subproblem}
 		\begin{subproblem}
 			$0$, $5$, $13$
 			\begin{shortsolution}
 				$p(x)=x(x-5)(x-13)$
 			\end{shortsolution}
 		\end{subproblem}
 		\begin{subproblem}
 			$-7$, $2$ (multiplicity $3$), $5$
 			\begin{shortsolution}
 				$p(x)=(x+7)(x-2)^3(x-5)$
 			\end{shortsolution}
 		\end{subproblem}
 		\begin{subproblem}
 			$0$, $\pm i$
 			\begin{shortsolution}
 				$p(x)=x(x^2+1)$
 			\end{shortsolution}
 		\end{subproblem}
 		\begin{subproblem}
 			$\pm 2i$, $\pm 7$
 			\begin{shortsolution}
 				$p(x)=(x^2+4)(x^2-49)$
 			\end{shortsolution}
 		\end{subproblem}
 		\begin{subproblem}
 			$-2\pm i\sqrt{6}$
 		\end{subproblem}
 	\end{multicols}
 	\end{problem}
 	%===================================
 	%   Author: Hughes
 	%   Date:   June 2012
 	%===================================
 	\begin{problem}[Composition of polynomials]
 	Let $p$ and $q$ be polynomial functions that have formulas
 	\[
 		p(x)=(x+1)(x+2)(x+5), \qquad q(x)=3-x^4
 	\]
 	Evaluate each of the following.
 	\begin{multicols}{4}
 		\begin{subproblem}
 			$(p\circ q)(0)$
 			\begin{shortsolution}
 				$160$
 			\end{shortsolution}
 		\end{subproblem}
 		\begin{subproblem}
 			$(q\circ p)(0)$
 			\begin{shortsolution}
 				$-9997$
 			\end{shortsolution}
 		\end{subproblem}
 		\begin{subproblem}
 			$(p\circ q)(1)$
 			\begin{shortsolution}
 				$84$
 			\end{shortsolution}
 		\end{subproblem}
 		\begin{subproblem}
 			$(p\circ p)(0)$
 			\begin{shortsolution}
 				$1980$
 			\end{shortsolution}
 		\end{subproblem}
 	\end{multicols}
 	\end{problem}
 	%===================================
 	%   Author: Hughes
 	%   Date:   June 2012
 	%===================================
 	\begin{problem}[Piecewise polynomial functions]
 	Let $P$ be the piecewise-defined function with formula
 	\[
 		P(x)=\begin{cases}
 		(1-x)(2x+5)(x^2+1), &   x\leq -3\\
 		4-x^2,              &   -3<x < 4\\
 		x^3                 &   x\geq 4
 		\end{cases}
 	\]
 	Evaluate each of the following
 	\begin{multicols}{5}
 		\begin{subproblem}
 			$P(-4)$
 			\begin{shortsolution}
 				$-255$
 			\end{shortsolution}
 		\end{subproblem}
 		\begin{subproblem}
 			$P(0)$
 			\begin{shortsolution}
 				$4$
 			\end{shortsolution}
 		\end{subproblem}
 		\begin{subproblem}
 			$P(4)$
 			\begin{shortsolution}
 				$64$
 			\end{shortsolution}
 		\end{subproblem}
 		\begin{subproblem}
 			$P(-3)$
 			\begin{shortsolution}
 				$-40$
 			\end{shortsolution}
 		\end{subproblem}
 		\begin{subproblem}
 			$(P\circ P)(0)$
 			\begin{shortsolution}
 				$64$
 			\end{shortsolution}
 		\end{subproblem}
 	\end{multicols}
 	\end{problem}
 	 	 	
 	%===================================
 	%   Author: Hughes
 	%   Date:   July 2012
 	%===================================
 	\begin{problem}[Function algebra]
 	Let $p$ and $q$ be the polynomial functions that have formulas
 	\[
 		p(x)=x(x+1)(x-3)^2, \qquad q(x)=7-x^2
 	\]
 	Evaluate each of the following (if possible).
 	\begin{multicols}{4}
 		\begin{subproblem}
 			$(p+q)(1)$
 			\begin{shortsolution}
 				$14$
 			\end{shortsolution}
 		\end{subproblem}
 		\begin{subproblem}
 			$(p-q)(0)$
 			\begin{shortsolution}
 				$7$
 			\end{shortsolution}
 		\end{subproblem}
 		\begin{subproblem}
 			$(p\cdot q)(\sqrt{7})$
 			\begin{shortsolution}
 				$0$
 			\end{shortsolution}
 		\end{subproblem}
 		\begin{subproblem}
 			$\left( \frac{q}{p} \right)(1)$
 			\begin{shortsolution}
 				$\frac{3}{4}$
 			\end{shortsolution}
 		\end{subproblem}
 	\end{multicols}
 	\begin{subproblem}
 		What is the domain of the function $\frac{q}{p}$?
 		\begin{shortsolution}
 			$(-\infty,-1)\cup (-1,0)\cup (0,3)\cup (3,\infty)$
 		\end{shortsolution}
 	\end{subproblem}
 	\end{problem}
 	 	 	
 	%===================================
 	%   Author: Hughes
 	%   Date:   July 2012
 	%===================================
 	\begin{problem}[Transformations: given the transformation, find the formula]
 	Let $p$ be the polynomial function that has formula.
 	\[
 		p(x)=4x(x^2-1)(x+3)
 	\]
 	In each of the following
 	problems apply the given transformation to the function $p$ and
 	write a formula for the transformed version of $p$.
 	\begin{multicols}{2}
 		\begin{subproblem}
 			Shift $p$ to the right by $5$ units.
 			\begin{shortsolution}
 				$p(x-5)=4(x-5)(x-2)(x^2-10x+24)$
 			\end{shortsolution}
 		\end{subproblem}
 		\begin{subproblem}
 			Shift $p$ to the left by $6$ units.
 			\begin{shortsolution}
 				$p(x+6)=4(x+6)(x+9)(x^2+12x+35)$
 			\end{shortsolution}
 		\end{subproblem}
 		\begin{subproblem}
 			Shift $p$ up by $12$ units.
 			\begin{shortsolution}
 				$p(x)+12=4x(x^2-1)(x+3)+12$
 			\end{shortsolution}
 		\end{subproblem}
 		\begin{subproblem}
 			Shift $p$ down by $2$ units.
 			\begin{shortsolution}
 				$p(x)-2=4x(x^2-1)(x+3)-2$
 			\end{shortsolution}
 		\end{subproblem}
 		\begin{subproblem}
 			Reflect $p$ over the horizontal axis.
 			\begin{shortsolution}
 				$-p(x)=-4x(x^2-1)(x+3)$
 			\end{shortsolution}
 		\end{subproblem}
 		\begin{subproblem}
 			Reflect $p$ over the vertical axis.
 			\begin{shortsolution}
 				$p(-x)=-4x(x^2-1)(3-x)$
 			\end{shortsolution}
 		\end{subproblem}
 	\end{multicols}
 	\end{problem}
 	 	 	
 	%===================================
 	%   Author: Hughes
 	%   Date:   May 2011
 	%===================================
 	\begin{problem}[Find a formula from a table]\label{poly:prob:findformula}
 	\Crefrange{poly:tab:findformulap}{poly:tab:findformulas} show values of polynomial functions, $p$, $q$,
 	$r$, and $s$.
 	 	 	
 	\begin{table}[!htb]
 		\centering
 		\begin{widepage}
 		\caption{Tables for \cref{poly:prob:findformula}}
 		\label{poly:tab:findformula}
 		\begin{subtable}{.2\textwidth}
 			\centering
 			\caption{$y=p(x)$}
 			\label{poly:tab:findformulap}
 			\begin{tabular}{rr}
 				\beforeheading
 				\heading{$x$} & \heading{$y$} \\
 				\afterheading
 				$-4$          & $-56$         \\\normalline
 				$-3$          & $-18$         \\\normalline
 				$-2$          & $0$           \\\normalline
 				$-1$          & $4$           \\\normalline
 				$0$           & $0$           \\\normalline
 				$1$           & $-6$          \\\normalline
 				$2$           & $-8$          \\\normalline
 				$3$           & $0$           \\\normalline
 				$4$           & $24$          \\\lastline
 			\end{tabular}
 		\end{subtable}
 		\hfill
 		\begin{subtable}{.2\textwidth}
 			\centering
 			\caption{$y=q(x)$}
 			\label{poly:tab:findformulaq}
 			\begin{tabular}{rr}
 				\beforeheading
 				\heading{$x$} & \heading{$y$} \\ \afterheading
 				$-4$          & $-16$         \\\normalline
 				$-3$          & $-3$          \\\normalline
 				$-2$          & $0$           \\\normalline
 				$-1$          & $-1$          \\\normalline
 				$0$           & $0$           \\\normalline
 				$1$           & $9$           \\\normalline
 				$2$           & $32$          \\\normalline
 				$3$           & $75$          \\\normalline
 				$4$           & $144$         \\\lastline
 			\end{tabular}
 		\end{subtable}
 		\hfill
 		\begin{subtable}{.2\textwidth}
 			\centering
 			\caption{$y=r(x)$}
 			\label{poly:tab:findformular}
 			\begin{tabular}{rr}
 				\beforeheading
 				\heading{$x$} & \heading{$y$} \\ \afterheading
 				$-4$          & $105$         \\\normalline
 				$-3$          & $0$           \\\normalline
 				$-2$          & $-15$         \\\normalline
 				$-1$          & $0$           \\\normalline
 				$0$           & $9$           \\\normalline
 				$1$           & $0$           \\\normalline
 				$2$           & $-15$         \\\normalline
 				$3$           & $0$           \\\normalline
 				$4$           & $105$         \\\lastline
 			\end{tabular}
 		\end{subtable}
 		\hfill
 		\begin{subtable}{.2\textwidth}
 			\centering
 			\caption{$y=s(x)$}
 			\label{poly:tab:findformulas}
 			\begin{tabular}{rr}
 				\beforeheading
 				\heading{$x$} & \heading{$y$} \\ \afterheading
 				$-4$          & $75$          \\\normalline
 				$-3$          & $0$           \\\normalline
 				$-2$          & $-9$          \\\normalline
 				$-1$          & $0$           \\\normalline
 				$0$           & $3$           \\\normalline
 				$1$           & $0$           \\\normalline
 				$2$           & $15$          \\\normalline
 				$3$           & $96$          \\\normalline
 				$4$           & $760$         \\\lastline
 			\end{tabular}
 		\end{subtable}
 		\end{widepage}
 	\end{table}
 	 	 	
 	\begin{subproblem}
 		Assuming that all of the zeros of $p$ are shown (in \cref{poly:tab:findformulap}), how many zeros does $p$ have?
 		\begin{shortsolution}
 			$p$ has 3 zeros.
 		\end{shortsolution}
 	\end{subproblem}
 	\begin{subproblem}
 		What is the degree of $p$?
 		\begin{shortsolution}
 			$p$ is degree 3.
 		\end{shortsolution}
 	\end{subproblem}
 	\begin{subproblem}
 		Write a formula for $p(x)$.
 		\begin{shortsolution}
 			$p(x)=x(x+2)(x-3)$
 		\end{shortsolution}
 	\end{subproblem}
 	\begin{subproblem}
 		Assuming that all of the zeros of $q$ are shown (in \cref{poly:tab:findformulaq}), how many zeros does $q$ have?
 		\begin{shortsolution}
 			$q$ has 2 zeros.
 		\end{shortsolution}
 	\end{subproblem}
 	\begin{subproblem}
 		Describe the difference in behavior of $p$ and $q$ at $-2$.
 		\begin{shortsolution}
 			$p$ changes sign at $-2$, and $q$ does not change sign at $-2$.
 		\end{shortsolution}
 	\end{subproblem}
 	\begin{subproblem}
 		Given that $q$ is a degree-$3$ polynomial, write a formula for $q(x)$.
 		\begin{shortsolution}
 			$q(x)=x(x+2)^2$
 		\end{shortsolution}
 	\end{subproblem}
 	\begin{subproblem}
 		Assuming that all of the zeros of $r$ are shown (in \cref{poly:tab:findformular}), find a formula for $r(x)$.
 		\begin{shortsolution}
 			$r(x)=(x+3)(x+1)(x-1)(x-3)$
 		\end{shortsolution}
 	\end{subproblem}
 	\begin{subproblem}
 		Assuming that all of the zeros of $s$ are shown (in \cref{poly:tab:findformulas}), find a formula for $s(x)$.
 		\begin{shortsolution}
 			$s(x)=(x+3)(x+1)(x-1)^2$
 		\end{shortsolution}
 	\end{subproblem}
 	\end{problem}
 	\end{exercises}
 	 	 	
\section{Rational functions}
 \subsection*{Power functions with negative exponents}
 	The study of rational functions will rely upon a good knowledge
 	of power functions with negative exponents. \Cref{rat:ex:oddpow,rat:ex:evenpow} are
 	simple but fundamental to understanding the behavior of rational functions.
 	%===================================
 	%   Author: Hughes
 	%   Date:   May 2011
 	%===================================
 	\begin{pccexample}[Power functions with odd negative exponents]\label{rat:ex:oddpow}
 		Graph each of the following functions on your calculator, state their domain in interval notation, and their
 		behavior as $x\rightarrow 0^-$ and $x\rightarrow 0^+$.
 		\[
 			f(x)=\frac{1}{x},\qquad g(x)=\dfrac{1}{x^3},\qquad h(x)=\dfrac{1}{x^5}
 		\]
 		\begin{pccsolution}
 			The functions $f$, $g$, and $k$ are plotted in \cref{rat:fig:oddpow}.
 			The domain of each of the functions $f$, $g$, and $h$ is $(-\infty,0)\cup (0,\infty)$. Note that
 			the long-run behavior of each of the functions is the same, and in particular
 			\begin{align*}
 				f(x)\rightarrow 0                         & \text{ as } x\rightarrow\infty  \\
 				\mathllap{\text{and }}  f(x)\rightarrow 0 & \text{ as } x\rightarrow-\infty 
 			\end{align*}
 			The same results hold for $g$ and $h$. Note also that each of the functions
 			has a \emph{vertical asymptote} at $0$. We see that
 			\begin{align*}
 				f(x)\rightarrow -\infty                         & \text{ as } x\rightarrow 0^- \\
 				\mathllap{\text{and }}   f(x)\rightarrow \infty & \text{ as } x\rightarrow 0^+ 
 			\end{align*}
 			The same results hold for $g$ and $h$.
 			 			 			
 			The curve of a function that has a vertical asymptote is necessarily separated
 			into \emph{branches}| each of the functions $f$, $g$, and $h$ have $2$ branches.
 		\end{pccsolution}
 	\end{pccexample}
 	 	 	
 	\begin{figure}[!htb]
 		\begin{minipage}{.45\textwidth}
 			\begin{tikzpicture}
 				\begin{axis}[
 						framed,
 						xmin=-3,xmax=3,
 						ymin=-5,ymax=5,
 						xtick={-2,-1,...,2},
 						minor ytick={-3,-1,...,3},
 						grid=both,
 						width=\textwidth,
 						legend pos=north west,
 					]
 					\addplot expression[domain=-3:-0.2]{1/x};
 					\addplot expression[domain=-3:-0.584]{1/x^3};
 					\addplot expression[domain=-3:-0.724]{1/x^5};
 					\addplot expression[domain=0.2:3]{1/x};
 					\addplot expression[domain=0.584:3]{1/x^3};
 					\addplot expression[domain=0.724:3]{1/x^5};
 					\addplot[soldot]coordinates{(-1,-1)}node[axisnode,anchor=north east]{$(-1,-1)$};
 					\addplot[soldot]coordinates{(1,1)}node[axisnode,anchor=south west]{$(1,1)$};
 					\legend{$f$,$g$,$h$}
 				\end{axis}
 			\end{tikzpicture}
 			\caption{}
 			\label{rat:fig:oddpow}
 		\end{minipage}%
 		\hfill
 		\begin{minipage}{.45\textwidth}
 			\begin{tikzpicture}
 				\begin{axis}[
 						framed,
 						xmin=-3,xmax=3,
 						ymin=-5,ymax=5,
 						xtick={-2,-1,...,2},
 						minor ytick={-3,-1,...,3},
 						grid=both,
 						width=\textwidth,
 						legend pos=south east,
 					]
 					\addplot expression[domain=-3:-0.447]{1/x^2};
 					\addplot expression[domain=-3:-0.668]{1/x^4};
 					\addplot expression[domain=-3:-0.764]{1/x^6};
 					\addplot expression[domain=0.447:3]{1/x^2};
 					\addplot expression[domain=0.668:3]{1/x^4};
 					\addplot expression[domain=0.764:3]{1/x^6};
 					\addplot[soldot]coordinates{(-1,1)}node[axisnode,anchor=south east]{$(-1,1)$};
 					\addplot[soldot]coordinates{(1,1)}node[axisnode,anchor=south west]{$(1,1)$};
 					\legend{$F$,$G$,$H$}
 				\end{axis}
 			\end{tikzpicture}
 			\caption{}
 			\label{rat:fig:evenpow}
 		\end{minipage}%
 	\end{figure}
 	 	 	
 	 	 	
 	%===================================
 	%   Author: Hughes
 	%   Date:   May 2011
 	%===================================
 	\begin{pccexample}[Power functions with even negative exponents]\label{rat:ex:evenpow}%
 		Graph each of the following functions, state their domain, and their
 		behavior as $x\rightarrow 0^-$ and $x\rightarrow 0^+$.
 		\[
 			f(x)=\frac{1}{x^2},\qquad g(x)=\frac{1}{x^4},\qquad h(x)=\frac{1}{x^6}
 		\]
 		\begin{pccsolution}
 			The functions $F$, $G$, and $H$ are plotted in \cref{rat:fig:evenpow}.
 			The domain of each of the functions $F$, $G$, and $H$ is $(-\infty,0)\cup (0,\infty)$. Note that
 			the long-run behavior of each of the functions is the same, and in particular
 			\begin{align*}
 				F(x)\rightarrow 0                           & \text{ as } x\rightarrow\infty  \\
 				\mathllap{\text{and }}    f(x)\rightarrow 0 & \text{ as } x\rightarrow-\infty 
 			\end{align*}
 			As in \cref{rat:ex:oddpow}, $F$ has a horizontal asymptote  that
 			has equation $y=0$.
 			The same results hold for $G$ and $H$. Note also that each of the functions
 			has a \emph{vertical asymptote} at $0$. We see that
 			\begin{align*}
 				F(x)\rightarrow \infty                          & \text{ as } x\rightarrow 0^- \\
 				\mathllap{\text{and }}   F(x)\rightarrow \infty & \text{ as } x\rightarrow 0^+ 
 			\end{align*}
 			The same results hold for $G$ and $H$. Each of the functions $F$, $G$, and $H$
 			have $2$ branches.
 		\end{pccsolution}
 	\end{pccexample}
 	%===================================
 	%   Author: Hughes
 	%   Date:   March 2012
 	%===================================
 	\begin{doyouunderstand}
 		\begin{problem}
 		Repeat \cref{rat:ex:oddpow,rat:ex:evenpow} using (respectively)
 		\begin{subproblem}
 			$k(x)=-\dfrac{1}{x}$, $ m(x)=-\dfrac{1}{x^3}$, $ n(x)=-\dfrac{1}{x^5}$
 			\begin{shortsolution}
 				The functions $k$, $m$, and $n$ have domain $(-\infty,0)\cup (0,\infty)$, and
 				are graphed below.
 				 				 				
 				\begin{tikzpicture}
 					\begin{axis}[
 							framed,
 							xmin=-3,xmax=3,
 							ymin=-5,ymax=5,
 							xtick={-2,-1,...,2},
 							minor ytick={-3,-1,...,3},
 							grid=both,
 							width=\solutionfigurewidth,
 							legend pos=north east,
 						]
 						\addplot expression[domain=-3:-0.2]{-1/x};
 						\addplot expression[domain=-3:-0.584]{-1/x^3};
 						\addplot expression[domain=-3:-0.724]{-1/x^5};
 						\addplot expression[domain=0.2:3]{-1/x};
 						\addplot expression[domain=0.584:3]{-1/x^3};
 						\addplot expression[domain=0.724:3]{-1/x^5};
 						\legend{$k$,$m$,$n$}
 					\end{axis}
 				\end{tikzpicture}
 				 				 				
 				Note that
 				\begin{align*}
 					k(x)\rightarrow 0                                & \text{ as } x\rightarrow\infty  \\
 					\mathllap{\text{and }}    k(x)\rightarrow 0      & \text{ as } x\rightarrow-\infty \\
 					\intertext{and also}
 					k(x)\rightarrow \infty                           & \text{ as } x\rightarrow 0^-    \\
 					\mathllap{\text{and }}   k(x)\rightarrow -\infty & \text{ as } x\rightarrow 0^+    
 				\end{align*}
 				The same are true for $m$ and $n$.
 			\end{shortsolution}
 		\end{subproblem}
 		\begin{subproblem}
 			$ K(x)=-\dfrac{1}{x^2}$, $ M(x)=-\dfrac{1}{x^4}$, $ N(x)=-\dfrac{1}{x^6}$
 			\begin{shortsolution}
 				The functions $K$, $M$, and $N$ have domain $(-\infty,0)\cup (0,\infty)$, and
 				are graphed below.
 				 				 				
 				\begin{tikzpicture}
 					\begin{axis}[
 							framed,
 							xmin=-3,xmax=3,
 							ymin=-5,ymax=5,
 							xtick={-2,-1,...,2},
 							minor ytick={-3,-1,...,3},
 							grid=both,
 							width=\solutionfigurewidth,
 							legend pos=north east,
 						]
 						\addplot expression[domain=-3:-0.447]{-1/x^2};
 						\addplot expression[domain=-3:-0.668]{-1/x^4};
 						\addplot expression[domain=-3:-0.764]{-1/x^6};
 						\addplot expression[domain=0.447:3]{-1/x^2};
 						\addplot expression[domain=0.668:3]{-1/x^4};
 						\addplot expression[domain=0.764:3]{-1/x^6};
 						\legend{$K$,$M$,$N$}
 					\end{axis}
 				\end{tikzpicture}
 				 				 				
 				Note that
 				\begin{align*}
 					K(x)\rightarrow 0                                & \text{ as } x\rightarrow\infty  \\
 					\mathllap{\text{and }}    K(x)\rightarrow 0      & \text{ as } x\rightarrow-\infty \\
 					\intertext{and also}
 					K(x)\rightarrow -\infty                          & \text{ as } x\rightarrow 0^-    \\
 					\mathllap{\text{and }}   K(x)\rightarrow -\infty & \text{ as } x\rightarrow 0^+    
 				\end{align*}
 				The same are true for $M$ and $N$.
 			\end{shortsolution}
 		\end{subproblem}
 		\end{problem}
 	\end{doyouunderstand}
 	 	 	
 \subsection*{Rational functions}
 	\begin{pccdefinition}[Rational functions]\label{rat:def:function}
 		Rational functions have the form
 		\[
 			r(x) = \frac{p(x)}{q(x)}
 		\]
 		where both $p$ and $q$ are polynomials.
 		 		 		
 		Note that
 		\begin{itemize}
 			\item the domain or $r$ will be all real numbers, except those that
 			      make the \emph{denominator}, $q(x)$, equal to $0$;
 			\item the zeros of $r$ are the zeros of $p$, i.e the real numbers
 			      that make the \emph{numerator}, $p(x)$, equal to $0$.
 		\end{itemize}
 		 		 		
 		\Cref{rat:ex:oddpow,rat:ex:evenpow} are particularly important because $r$
 		will behave like $\frac{1}{x}$, or $\frac{1}{x^2}$ around its vertical asymptotes,
 		depending on the power that the relevant term is raised to| we will demonstrate
 		this in what follows.
 	\end{pccdefinition}
 	 	 	
 	%===================================
 	%   Author: Hughes
 	%   Date:   May 2011
 	%===================================
 	\begin{pccexample}[Rational or not]
 		Identify whether each of the following functions is a rational or not. If
 		the function is rational, state the domain.
 		\begin{multicols}{3}
 			\begin{enumerate}
 				\item $r(x)=\dfrac{1}{x}$
 				\item $f(x)=2^x+3$
 				\item $g(x)=19$
 				\item $h(x)=\dfrac{3+x}{4-x}$
 				\item $k(x)=\dfrac{x^3+2x}{x-15}$
 				\item $l(x)=9-4x$
 				\item $m(x)=\dfrac{x+5}{(x-7)(x+9)}$
 				\item $n(x)=x^2+6x+7$
 				\item $q(x)=1-\dfrac{3}{x+1}$
 			\end{enumerate}
 		\end{multicols}
 		\begin{pccsolution}
 			\begin{enumerate}
 				\item $r$ is rational; the domain of $r$ is $(-\infty,0)\cup(0,\infty)$.
 				\item $f$ is not rational.
 				\item $g$ is not rational; $g$ is constant.
 				\item $h$ is rational; the domain of $h$ is $(-\infty,4)\cup(4,\infty)$.
 				\item $k$ is rational; the domain of $k$ is $(-\infty,15)\cup(15,\infty)$.
 				\item $l$ is not rational; $l$ is linear.
 				\item $m$ is rational; the domain of $m$ is $(-\infty,-9)\cup(-9,7)\cup(7,\infty)$.
 				\item $n$ is not rational; $n$ is quadratic (or you might describe $n$ as a polynomial).
 				\item $q$ is rational; the domain of $q$ is $(-\infty,-1)\cup (-1,\infty)$.
 			\end{enumerate}
 		\end{pccsolution}
 	\end{pccexample}
 	 	 	
 	%===================================
 	%   Author: Hughes
 	%   Date:   May 2011
 	%===================================
 	\begin{pccexample}[Match formula to graph]
 		Each of the following functions is graphed in \cref{rat:fig:whichiswhich}.
 		Which is which?
 		\[
 			r(x)=\frac{1}{x-3}, \qquad q(x)=\frac{x-2}{x+5}, \qquad k(x)=\frac{1}{(x+2)(x-3)}
 		\]
 		\begin{figure}[!htb]
 			\setlength{\figurewidth}{0.3\textwidth}
 			\begin{subfigure}{\figurewidth}
 				\begin{tikzpicture}[/pgf/declare function={f=(x-2)/(x+5);}]
 					\begin{axis}[
 							framed,
 							xmin=-10,xmax=10,
 							ymin=-6,ymax=6,
 							xtick={-8,-6,...,8},
 							minor ytick={-4,-3,...,4},
 							grid=both,
 							width=\textwidth,
 						]
 						\addplot[pccplot] expression[domain=-10:-6.37]{f};
 						\addplot[pccplot] expression[domain=-3.97:10]{f};
 						\addplot[soldot] coordinates{(2,0)};
 						\addplot[asymptote,domain=-6:6]({-5},{x});
 					\end{axis}
 				\end{tikzpicture}
 				\caption{}
 				\label{rat:fig:which1}
 			\end{subfigure}
 			\hfill
 			\begin{subfigure}{\figurewidth}
 				\begin{tikzpicture}[/pgf/declare function={f=1/(x-3);}]
 					\begin{axis}[
 							framed,
 							xmin=-10,xmax=10,
 							ymin=-5,ymax=6,
 							xtick={-8,-6,...,8},
 							ytick={-4,4},
 							minor ytick={-3,...,5},
 							grid=both,
 							width=\textwidth,
 						]
 						\addplot[pccplot] expression[domain=-10:2.8]{f};
 						\addplot[pccplot] expression[domain=3.17:10]{f};
 						\addplot[asymptote,domain=-6:6]({3},{x});
 					\end{axis}
 				\end{tikzpicture}
 				\caption{}
 				\label{rat:fig:which2}
 			\end{subfigure}
 			\hfill
 			\begin{subfigure}{\figurewidth}
 				\begin{tikzpicture}[/pgf/declare function={f=1/((x-3)*(x+2));}]
 					\begin{axis}[
 							framed,
 							xmin=-10,xmax=10,
 							ymin=-5,ymax=5,
 							xtick={-8,-6,...,8},
 							ytick={-4,4},
 							minor ytick={-3,...,3},
 							grid=both,
 							width=\textwidth,
 						]
 						\addplot[pccplot] expression[domain=-10:-2.03969]{f};
 						\addplot[pccplot] expression[domain=-1.95967:2.95967]{f};
 						\addplot[pccplot] expression[domain=3.03969:10]{f};
 						\addplot[asymptote,domain=-5:5]({-2},{x});
 						\addplot[asymptote,domain=-5:5]({3},{x});
 					\end{axis}
 				\end{tikzpicture}
 				\caption{}
 				\label{rat:fig:which3}
 			\end{subfigure}
 			\caption{}
 			\label{rat:fig:whichiswhich}
 		\end{figure}
 		 		 		
 		\begin{pccsolution}
 			Let's start with the function $r$. Note that domain of $r$ is $(-\infty,3)\cup(0,3)$, so
 			we search for a function that has a vertical asymptote at $3$. There
 			are two possible choices: the functions graphed in \cref{rat:fig:which2,rat:fig:which3},
 			but note that the function in \cref{rat:fig:which3} also has a vertical asymptote at $-2$
 			which is not consistent with the formula for $r(x)$. Therefore, $y=r(x)$
 			is graphed in \cref{rat:fig:which2}.
 			 			 			
 			The function $q$ has domain $(-\infty,-5)\cup(-5,\infty)$, so we search
 			for a function that has a vertical asymptote at $-5$. The only candidate
 			is the curve shown in \cref{rat:fig:which1}; note that the curve also goes through $(2,0)$,
 			which is consistent with the formula for $q(x)$, since $q(2)=0$, i.e $q$
 			has a zero at $2$.
 			 			 			
 			The function $k$ has domain $(-\infty,-2)\cup(-2,3)\cup(3,\infty)$, and
 			has vertical asymptotes at $-2$ and $3$. This is consistent with
 			the graph in \cref{rat:fig:which3} (and is the only curve that
 			has $3$ branches).
 			 			 			
 			We note that each function behaves like $\frac{1}{x}$ around its vertical asymptotes,
 			because each linear factor in each denominator is raised to the power $1$; if (for example)
 			the definition of $r$ was instead
 			\[
 				r(x)=\frac{1}{(x-3)^2}
 			\]
 			then we would see that $r$ behaves like $\frac{1}{x^2}$ around its vertical asymptote, and
 			the graph of $r$ would be very different. We will deal with these cases in the examples that follow.
 		\end{pccsolution}
 	\end{pccexample}
 	 	 	
 	%===================================
 	%   Author: Hughes
 	%   Date:   May 2011
 	%===================================
 	\begin{pccexample}[Repeated factors in the denominator]
 		Consider the functions $f$, $g$, and $h$ that have formulas
 		\[
 			f(x)=\frac{x-2}{(x-3)(x+2)}, \qquad g(x)=\frac{x-2}{(x-3)^2(x+2)}, \qquad h(x)=\frac{x-2}{(x-3)(x+2)^2}
 		\]
 		which are graphed in \cref{rat:fig:repfactd}. Note that each function has $2$
 		vertical asymptotes, and the domain of each function is
 		\[
 			(-\infty,-2)\cup(-2,3)\cup(3,\infty)
 		\]
 		so we are not surprised to see that each curve has $3$ branches. We also note that
 		the numerator of each function is the same, which tells us that each function has
 		only $1$ zero at $2$.
 		 		 		
 		The functions $g$ and $h$ are different from those that we have considered previously,
 		because they have a repeated factor in the denominator. Notice in particular
 		the way that the functions behave around their asymptotes:
 		\begin{itemize}
 			\item $f$ behaves like $\frac{1}{x}$ around both of its asymptotes;
 			\item $g$ behaves like $\frac{1}{x}$ around $-2$, and like $\frac{1}{x^2}$ around $3$;
 			\item $h$ behaves like $\frac{1}{x^2}$ around $-2$, and like $\frac{1}{x}$ around $3$.
 		\end{itemize}
 	\end{pccexample}
 	\begin{figure}[!htb]
 		\setlength{\figurewidth}{0.3\textwidth}
 		\begin{subfigure}{\figurewidth}
 			\begin{tikzpicture}[/pgf/declare function={f=(x-2)/((x+2)*(x-3));}]
 				\begin{axis}[
 						%                    framed,
 						xmin=-5,xmax=5,
 						ymin=-4,ymax=4,
 						xtick={-4,-2,...,4},
 						ytick={-2,2},
 						%                    grid=both,
 						width=\textwidth,
 					]
 					\addplot[pccplot] expression[domain=-5:-2.201]{f};
 					\addplot[pccplot] expression[domain=-1.802:2.951]{f};
 					\addplot[pccplot] expression[domain=3.052:5]{f};
 					\addplot[soldot] coordinates{(2,0)};
 					%                 \addplot[asymptote,domain=-6:6]({-2},{x});
 					%                 \addplot[asymptote,domain=-6:6]({3},{x});
 				\end{axis}
 			\end{tikzpicture}
 			\caption{$y=\dfrac{x-2}{(x+2)(x-3)}$}
 			\label{rat:fig:repfactd1}
 		\end{subfigure}
 		\hfill
 		\begin{subfigure}{\figurewidth}
 			\begin{tikzpicture}[/pgf/declare function={f=(x-2)/((x+2)*(x-3)^2);}]
 				\begin{axis}[
 						%                    framed,
 						xmin=-5,xmax=5,
 						ymin=-4,ymax=4,
 						xtick={-4,-2,...,4},
 						ytick={-2,2},
 						%                    grid=both,
 						width=\textwidth,
 					]
 					\addplot[pccplot] expression[domain=-5:-2.039]{f};
 					\addplot[pccplot] expression[domain=-1.959:2.796]{f};
 					\addplot[pccplot] expression[domain=3.243:5]{f};
 					\addplot[soldot] coordinates{(2,0)};
 					%                 \addplot[asymptote,domain=-4:4]({-2},{x});
 					%                 \addplot[asymptote,domain=-4:4]({3},{x});
 				\end{axis}
 			\end{tikzpicture}
 			\caption{$y=\dfrac{x-2}{(x+2)(x-3)^2}$}
 			\label{rat:fig:repfactd2}
 		\end{subfigure}
 		\hfill
 		\begin{subfigure}{\figurewidth}
 			\begin{tikzpicture}[/pgf/declare function={f=(x-2)/((x+2)^2*(x-3));}]
 				\begin{axis}[
 						%                    framed,
 						xmin=-5,xmax=5,
 						ymin=-4,ymax=4,
 						xtick={-4,-2,...,2},
 						ytick={-2,2},
 						%                    grid=both,
 						width=\textwidth,
 					]
 					\addplot[pccplot] expression[domain=-5:-2.451]{f};
 					\addplot[pccplot] expression[domain=-1.558:2.990]{f};
 					\addplot[pccplot] expression[domain=3.010:6]{f};
 					\addplot[soldot] coordinates{(2,0)};
 					%                 \addplot[asymptote,domain=-4:4]({-2},{x});
 					%                 \addplot[asymptote,domain=-4:4]({3},{x});
 				\end{axis}
 			\end{tikzpicture}
 			\caption{$y=\dfrac{x-2}{(x+2)^2(x-3)}$}
 			\label{rat:fig:repfactd3}
 		\end{subfigure}
 		\caption{}
 		\label{rat:fig:repfactd}
 	\end{figure}
 	 	 	
 	\Cref{rat:def:function} says that the zeros of
 	the rational function $r$ that has formula $r(x)=\frac{p(x)}{q(x)}$ are
 	the zeros of $p$. Let's explore this a little more.
 	%===================================
 	%   Author: Hughes
 	%   Date:   May 2012
 	%===================================
 	\begin{pccexample}[Zeros] Find the zeros of each of the following functions
 		\[
 			\alpha(x)=\frac{x+5}{3x-7}, \qquad \beta(x)=\frac{9-x}{x+1}, \qquad \gamma(x)=\frac{17x^2-10}{2x+1}
 		\]
 		\begin{pccsolution}
 			We find the zeros of each function in turn by setting the numerator equal to $0$. The zeros of
 			$\alpha$ are found by solving
 			\[
 				x+5=0
 			\]
 			The zero of $\alpha$ is $-5$.
 			 			 			
 			Similarly, we may solve $9-x=0$ to find the zero of $\beta$, which is clearly $9$.
 			 			 			
 			The zeros of $\gamma$ satisfy the equation
 			\[
 				17x^2-10=0
 			\]
 			which we can solve using the square root property to obtain
 			\[
 				x=\pm\frac{10}{17}
 			\]
 			The zeros of $\gamma$ are $\pm\frac{10}{17}$.
 		\end{pccsolution}
 	\end{pccexample}
 	 	 	
 \subsection*{Long-run behavior}
 	Our focus so far has been on the behavior of rational functions around
 	their \emph{vertical} asymptotes. In fact, rational functions also
 	have interesting long-run behavior around their \emph{horizontal} or
 	\emph{oblique} asymptotes. A rational function will always have either
 	a horizontal or an oblique asymptote| the case is determined by the degree
 	of the numerator and the degree of the denominator.
 	\begin{pccdefinition}[Long-run behavior]\label{rat:def:longrun}
 		Let $r$ be the rational function that has formula
 		\[
 			r(x) = \frac{a_n x^n + a_{n-1}x^{n-1}+\ldots + a_0}{b_m x^m + b_{m-1}x^{m-1}+\ldots+b_0}
 		\]
 		We can classify the long-run behavior of the rational function $r$
 		according to the following criteria:
 		\begin{itemize}
 			\item if $n<m$ then  $r$ has a horizontal asymptote with equation $y=0$;
 			\item if $n=m$ then $r$ has a horizontal asymptote with equation $y=\dfrac{a_n}{b_m}$;
 			\item if $n>m$ then $r$ will have an oblique asymptote as $x\rightarrow\pm\infty$ (more on this in \cref{rat:sec:oblique})
 		\end{itemize}
 	\end{pccdefinition}
 	We will concentrate on functions that have horizontal asymptotes until
 	we reach \cref{rat:sec:oblique}.
 	 	 	
 	%===================================
 	%   Author: Hughes
 	%   Date:   May 2012
 	%===================================
 	\begin{pccexample}[Long-run behavior graphically]\label{rat:ex:horizasymp}
 		\pccname{Kebede} has graphed the following functions in his graphing calculator
 		\[
 			r(x)=\frac{x+1}{x-3}, \qquad s(x)=\frac{2(x+1)}{x-3}, \qquad t(x)=\frac{3(x+1)}{x-3}
 		\]
 		and obtained the curves shown in \cref{rat:fig:horizasymp}. Kebede decides
 		to test his knowledgeable friend \pccname{Oscar}, and asks him
 		to match the formulas to the graphs.
 		 		 		
 		\begin{figure}[!htb]
 			\setlength{\figurewidth}{0.3\textwidth}
 			\begin{subfigure}{\figurewidth}
 				\begin{tikzpicture}[/pgf/declare function={f=2*(x+1)/(x-3);}]
 					\begin{axis}[
 							framed,
 							xmin=-15,xmax=15,
 							ymin=-6,ymax=6,
 							xtick={-12,-8,...,12},
 							minor ytick={-4,-3,...,4},
 							grid=both,
 							width=\textwidth,
 						]
 						\addplot[pccplot] expression[domain=-15:2]{f};
 						\addplot[pccplot] expression[domain=5:15]{f};
 						\addplot[soldot] coordinates{(-1,0)};
 						\addplot[asymptote,domain=-6:6]({3},{x});
 						\addplot[asymptote,domain=-15:15]({x},{2});
 					\end{axis}
 				\end{tikzpicture}
 				\caption{}
 				\label{rat:fig:horizasymp1}
 			\end{subfigure}
 			\hfill
 			\begin{subfigure}{\figurewidth}
 				\begin{tikzpicture}[/pgf/declare function={f=(x+1)/(x-3);}]
 					\begin{axis}[
 							framed,
 							xmin=-15,xmax=15,
 							ymin=-6,ymax=6,
 							xtick={-12,-8,...,12},
 							minor ytick={-4,-3,...,4},
 							grid=both,
 							width=\textwidth,
 						]
 						\addplot[pccplot] expression[domain=-15:2.42857,samples=50]{f};
 						\addplot[pccplot] expression[domain=3.8:15,samples=50]{f};
 						\addplot[soldot] coordinates{(-1,0)};
 						\addplot[asymptote,domain=-6:6]({3},{x});
 						\addplot[asymptote,domain=-15:15]({x},{1});
 					\end{axis}
 				\end{tikzpicture}
 				\caption{}
 				\label{rat:fig:horizasymp2}
 			\end{subfigure}
 			\hfill
 			\begin{subfigure}{\figurewidth}
 				\begin{tikzpicture}[/pgf/declare function={f=3*(x+1)/(x-3);}]
 					\begin{axis}[
 							framed,
 							xmin=-15,xmax=15,
 							ymin=-6,ymax=6,
 							xtick={-12,-8,...,12},
 							minor ytick={-4,-3,...,4},
 							grid=both,
 							width=\textwidth,
 						]
 						\addplot[pccplot] expression[domain=-15:1.6666,samples=50]{f};
 						\addplot[pccplot] expression[domain=7:15]{f};
 						\addplot[soldot] coordinates{(-1,0)};
 						\addplot[asymptote,domain=-6:6]({3},{x});
 						\addplot[asymptote,domain=-15:15]({x},{3});
 					\end{axis}
 				\end{tikzpicture}
 				\caption{}
 				\label{rat:fig:horizasymp3}
 			\end{subfigure}
 			\caption{Horizontal asymptotes}
 			\label{rat:fig:horizasymp}
 		\end{figure}
 		 		 		
 		Oscar notices that each function has a vertical asymptote at $3$ and a zero at $-1$.
 		The main thing that catches Oscar's eye is that each function has a different
 		coefficient in the numerator, and that each curve has a different horizontal asymptote.
 		In particular, Oscar notes that
 		\begin{itemize}
 			\item the curve shown in \cref{rat:fig:horizasymp1} has a horizontal asymptote with equation $y=2$;
 			\item the curve shown in \cref{rat:fig:horizasymp2} has a horizontal asymptote with equation $y=1$;
 			\item the curve shown in \cref{rat:fig:horizasymp3} has a horizontal asymptote with equation $y=3$.
 		\end{itemize}
 		Oscar is able to tie it all together for Kebede by referencing \cref{rat:def:longrun}. He says
 		that since the degree of the numerator and the degree of the denominator is the same
 		for each of the functions $r$, $s$, and $t$, the horizontal asymptote will be determined
 		by evaluating the ratio of their leading coefficients.
 		 		 		
 		Oscar therefore says that $r$ should have a horizontal asymptote $y=\frac{1}{1}=1$, $s$ should
 		have a horizontal asymptote $y=\frac{2}{1}=2$, and $t$ should have a horizontal asymptote
 		$y=\frac{3}{1}=3$. Kebede is able to finish the problem from here, and says that $r$ is
 		shown in \cref{rat:fig:horizasymp2}, $s$ is shown in \cref{rat:fig:horizasymp1}, and
 		$t$ is shown in \cref{rat:fig:horizasymp3}.
 	\end{pccexample}
 	 	 	
 	%===================================
 	%   Author: Hughes
 	%   Date:   May 2012
 	%===================================
 	\begin{pccexample}[Long-run behavior numerically]
 		\pccname{Xiao} and \pccname{Dwayne} saw \cref{rat:ex:horizasymp} but are a little confused
 		about horizontal asymptotes. What does it mean to say that a function $r$ has a horizontal
 		asymptote?
 		 		 		
 		They decide to explore the concept by
 		constructing a table of values for the rational functions $R$ and  $S$ that have formulas
 		\[
 			R(x)=\frac{-5(x+1)}{x-3}, \qquad S(x)=\frac{7(x-5)}{2(x+1)}
 		\]
 		In \cref{rat:tab:plusinfty} they model the behavior of $R$ and $S$ as $x\rightarrow\infty$,
 		and in \cref{rat:tab:minusinfty} they model the behavior of $R$ and $S$ as $x\rightarrow-\infty$
 		by substituting very large values of $|x|$ into each function.
 		\begin{table}[!htb]
 			\begin{minipage}{.5\textwidth}
 				\centering
 				\caption{$R$ and $S$ as $x\rightarrow\infty$}
 				\label{rat:tab:plusinfty}
 				\begin{tabular}{crr}
 					\beforeheading
 					$x$            & $R(x)$     & $S(x)$    \\ \afterheading
 					$1\times 10^2$ & $-5.20619$ & $3.29208$ \\\normalline
 					$1\times 10^3$ & $-5.02006$ & $3.47902$ \\\normalline
 					$1\times 10^4$ & $-5.00200$ & $3.49790$ \\\normalline
 					$1\times 10^5$ & $-5.00020$ & $3.49979$ \\\normalline
 					$1\times 10^6$ & $-5.00002$ & $3.49998$ \\\lastline
 				\end{tabular}
 			\end{minipage}%
 			\begin{minipage}{.5\textwidth}
 				\centering
 				\caption{$R$ and $S$ as $x\rightarrow-\infty$}
 				\label{rat:tab:minusinfty}
 				\begin{tabular}{crr}
 					\beforeheading
 					$x$             & $R(x)$     & $S(x)$    \\ \afterheading
 					$-1\times 10^2$ & $-4.80583$ & $3.71212$ \\\normalline
 					$-1\times 10^3$ & $-4.98006$ & $3.52102$ \\\normalline
 					$-1\times 10^4$ & $-4.99800$ & $3.50210$ \\\normalline
 					$-1\times 10^5$ & $-4.99980$ & $3.50021$ \\\normalline
 					$-1\times 10^6$ & $-4.99998$ & $3.50002$ \\\lastline
 				\end{tabular}
 			\end{minipage}
 		\end{table}
 		 		 		
 		Xiao and Dwayne study \cref{rat:tab:plusinfty,rat:tab:minusinfty} and decide that
 		the functions $R$ and $S$ never actually touch their horizontal asymptotes, but they
 		do get infinitely close. They also feel as if they have a better understanding of
 		what it means to study the behavior of a function as $x\rightarrow\pm\infty$.
 	\end{pccexample}
 	 	 	
 	%===================================
 	%   Author: Hughes
 	%   Date:   May 2011
 	%===================================
 	\begin{pccexample}[Repeated factors in the numerator]
 		Consider the functions $f$, $g$, and $h$ that have formulas
 		\[
 			f(x)=\frac{(x-2)^2}{(x-3)(x+1)}, \qquad g(x)=\frac{x-2}{(x-3)(x+1)}, \qquad h(x)=\frac{(x-2)^3}{(x-3)(x+1)}
 		\]
 		which are graphed in \cref{rat:fig:repfactn}. We note that each function has vertical
 		asymptotes at $-1$ and $3$, and so the domain of each function is
 		\[
 			(-\infty,-1)\cup(-1,3)\cup(3,\infty)
 		\]
 		We also notice that the numerators of each function are quite similar| indeed, each
 		function has a zero at $2$, but how does each function behave around their zero?
 		 		 		
 		Using \cref{rat:fig:repfactn} to guide us, we note that
 		\begin{itemize}
 			\item $f$ has a horizontal intercept $(2,0)$, but the curve of
 			      $f$ does not cut the horizontal axis| it bounces off it;
 			\item $g$ also has a horizontal intercept $(2,0)$, and the curve
 			      of $g$ \emph{does} cut the horizontal axis;
 			\item $h$ has a horizontal intercept $(2,0)$, and the curve of $h$
 			      also cuts the axis, but appears flattened as it does so.
 		\end{itemize}
 		 		 		
 		We can further enrich our study by discussing the long-run behavior of each function.
 		Using the tools of \cref{rat:def:longrun}, we can deduce that
 		\begin{itemize}
 			\item $f$ has a horizontal asymptote with equation $y=1$;
 			\item $g$ has a horizontal asymptote with equation $y=0$;
 			\item $h$ does \emph{not} have a horizontal asymptote| it has an oblique asymptote (we'll
 			      study this more in \cref{rat:sec:oblique}).
 		\end{itemize}
 	\end{pccexample}
 	 	 	
 	\begin{figure}[!htb]
 		\setlength{\figurewidth}{0.3\textwidth}
 		\begin{subfigure}{\figurewidth}
 			\begin{tikzpicture}[/pgf/declare function={f=(x-2)^2/((x+1)*(x-3));}]
 				\begin{axis}[
 						%                    framed,
 						xmin=-5,xmax=5,
 						ymin=-10,ymax=10,
 						xtick={-4,-2,...,4},
 						ytick={-8,-4,...,8},
 						%                    grid=both,
 						width=\figurewidth,
 					]
 					\addplot[pccplot] expression[domain=-5:-1.248,samples=50]{f};
 					\addplot[pccplot] expression[domain=-0.794:2.976,samples=50]{f};
 					\addplot[pccplot] expression[domain=3.026:5,samples=50]{f};
 					\addplot[soldot] coordinates{(2,0)};
 					%                 \addplot[asymptote,domain=-6:6]({-1},{x});
 					%                 \addplot[asymptote,domain=-6:6]({3},{x});
 				\end{axis}
 			\end{tikzpicture}
 			\caption{$y=\dfrac{(x-2)^2}{(x+1)(x-3)}$}
 			\label{rat:fig:repfactn1}
 		\end{subfigure}
 		\hfill
 		\begin{subfigure}{\figurewidth}
 			\begin{tikzpicture}[/pgf/declare function={f=(x-2)/((x+1)*(x-3));}]
 				\begin{axis}[
 						%                    framed,
 						xmin=-5,xmax=5,
 						ymin=-10,ymax=10,
 						xtick={-4,-2,...,4},
 						ytick={-8,-4,...,8},
 						%                    grid=both,
 						width=\figurewidth,
 					]
 					\addplot[pccplot] expression[domain=-5:-1.075]{f};
 					\addplot[pccplot] expression[domain=-0.925:2.975]{f};
 					\addplot[pccplot] expression[domain=3.025:5]{f};
 					\addplot[soldot] coordinates{(2,0)};
 					%                 \addplot[asymptote,domain=-6:6]({-1},{x});
 					%                 \addplot[asymptote,domain=-6:6]({3},{x});
 				\end{axis}
 			\end{tikzpicture}
 			\caption{$y=\dfrac{x-2}{(x+1)(x-3)}$}
 			\label{rat:fig:repfactn2}
 		\end{subfigure}
 		\hfill
 		\begin{subfigure}{\figurewidth}
 			\begin{tikzpicture}[/pgf/declare function={f=(x-2)^3/((x+1)*(x-3));}]
 				\begin{axis}[
 						%                    framed,
 						xmin=-5,xmax=5,
 						xtick={-8,-6,...,8},
 						%                    grid=both,
 						ymin=-30,ymax=30,
 						width=\figurewidth,
 					]
 					\addplot[pccplot] expression[domain=-5:-1.27]{f};
 					\addplot[pccplot] expression[domain=-0.806:2.99185]{f};
 					\addplot[pccplot] expression[domain=3.0085:5]{f};
 					\addplot[soldot] coordinates{(2,0)};
 					%                 \addplot[asymptote,domain=-30:30]({-1},{x});
 					%                 \addplot[asymptote,domain=-30:30]({3},{x});
 				\end{axis}
 			\end{tikzpicture}
 			\caption{$y=\dfrac{(x-2)^3}{(x+1)(x-3)}$}
 			\label{rat:fig:repfactn3}
 		\end{subfigure}
 		\caption{}
 		\label{rat:fig:repfactn}
 	\end{figure}
 	 	 	
 \subsection*{Holes}
 	Rational functions have a vertical asymptote at $a$ if the denominator is $0$ at $a$.
 	What happens if the numerator is $0$ at the same place? In this case, we say that the rational
 	function has a \emph{hole} at $a$.
 	\begin{pccdefinition}[Holes]
 		The rational function
 		\[
 			r(x)=\frac{p(x)}{q(x)}
 		\]
 		has a hole at $a$ if $p(a)=q(a)=0$. Note that holes are different from
 		a vertical asymptotes. We represent that $r$ has a hole at the point
 		$(a,r(a))$ on the curve $y=r(x)$ by
 		using a hollow circle, $\circ$.
 	\end{pccdefinition}
 	 	 	
 	%===================================
 	%   Author: Hughes
 	%   Date:   March 2012
 	%===================================
 	\begin{pccexample}
 		\pccname{Mohammed} and \pccname{Sue} have graphed the function $r$ that has formula
 		\[
 			r(x)=\frac{x^2+x-6}{(x-2)}
 		\]
 		in their calculators, and can not decide if the correct graph
 		is \cref{rat:fig:hole} or \cref{rat:fig:hole1}.
 		 		 		
 		Luckily for them, Oscar is nearby, and can help them settle the debate.
 		Oscar demonstrates that
 		\begin{align*}
 			r(x) & =\frac{(x+3)(x-2)}{(x-2)} \\
 			     & = x+3                     
 		\end{align*}
 		but only when $x\ne 2$, because the function is undefined at $2$. Oscar
 		says that this necessarily means that the domain or $r$ is
 		\[
 			(-\infty,2)\cup(2,\infty)
 		\]
 		and that $r$ must have a hole at $2$.
 		 		 		
 		Mohammed and Sue are very grateful for the clarification, and conclude that
 		the graph of $r$ is shown in \cref{rat:fig:hole1}.
 		\begin{figure}[!htb]
 			\begin{minipage}{.45\textwidth}
 				\begin{tikzpicture}
 					\begin{axis}[
 							framed,
 							xmin=-10,xmax=10,
 							ymin=-10,ymax=10,
 							xtick={-8,-4,...,8},
 							ytick={-8,-4,...,8},
 							grid=both,
 							width=\textwidth,
 						]
 						\addplot expression[domain=-10:7]{x+3};
 						\addplot[soldot] coordinates{(-3,0)};
 					\end{axis}
 				\end{tikzpicture}
 				\caption{}
 				\label{rat:fig:hole}
 			\end{minipage}%
 			\hfill
 			\begin{minipage}{.45\textwidth}
 				\begin{tikzpicture}
 					\begin{axis}[
 							framed,
 							xmin=-10,xmax=10,
 							ymin=-10,ymax=10,
 							xtick={-8,-4,...,8},
 							ytick={-8,-4,...,8},
 							grid=both,
 							width=\textwidth,
 						]
 						\addplot expression[domain=-10:7]{x+3};
 						\addplot[holdot] coordinates{(2,5)};
 						\addplot[soldot] coordinates{(-3,0)};
 					\end{axis}
 				\end{tikzpicture}
 				\caption{}
 				\label{rat:fig:hole1}
 			\end{minipage}%
 		\end{figure}
 	\end{pccexample}
 	 	 	
 	%===================================
 	%   Author: Hughes
 	%   Date:   May 2011
 	%===================================
 	\begin{pccexample}
 		Consider the function $f$ that has formula
 		\[
 			f(x)=\frac{x(x+3)}{x^2-4x}
 		\]
 		The domain of $f$ is $(-\infty,0)\cup(0,4)\cup(4,\infty)$ because both $0$ and $4$
 		make the denominator equal to $0$. Notice that
 		\begin{align*}
 			f(x) & = \frac{x(x+3)}{x(x-4)} \\
 			     & = \frac{x+3}{x-4}       
 		\end{align*}
 		provided that $x\ne 0$. Since $0$ makes the numerator
 		and the denominator 0 at the same time, we say that $f$ has a hole at $(0,-\nicefrac{3}{4})$.
 		Note that this necessarily means that $f$ does not have a vertical intercept.
 		 		 		
 		We also note $f$ has a vertical asymptote at $4$; the function is graphed in \cref{rat:fig:holeex}.
 		\begin{figure}[!htb]
 			\centering
 			\begin{tikzpicture}[/pgf/declare function={f=(x+3)/(x-4);}]
 				\begin{axis}[
 						framed,
 						xmin=-10,xmax=10,
 						ymin=-10,ymax=10,
 						xtick={-8,-6,...,8},
 						ytick={-8,-6,...,8},
 						grid=both,
 					]
 					\addplot[pccplot] expression[domain=-10:3.36364,samples=50]{f};
 					\addplot[pccplot] expression[domain=4.77:10]{f};
 					\addplot[asymptote,domain=-10:10]({4},{x});
 					\addplot[holdot]coordinates{(0,-0.75)};
 					\addplot[soldot] coordinates{(-3,0)};
 				\end{axis}
 			\end{tikzpicture}
 			\caption{$y=\dfrac{x(x+3)}{x^2-4x}$}
 			\label{rat:fig:holeex}
 		\end{figure}
 	\end{pccexample}
 	 	 	
 	 	 	
 	 	 	
 	%===================================
 	%   Author: Hughes
 	%   Date:   March 2012
 	%===================================
 	\begin{pccexample}[Minimums and maximums]
 		\pccname{Seamus} and \pccname{Trang} are discussing rational functions. Seamus says that
 		if a rational function has a vertical asymptote, then it can
 		not possibly have local minimums and maximums, nor can it have
 		global minimums and maximums.
 		 		 		
 		Trang says this statement is not always true. She plots the functions
 		$f$ and $g$ that have formulas
 		\[
 			f(x)=-\frac{32(x-1)(x+1)}{(x-2)^2(x+2)^2}, \qquad g(x)=\frac{32(x-1)(x+1)}{(x-2)^2(x+2)^2}
 		\]
 		in \cref{rat:fig:minmax1,rat:fig:minmax2} and shows them to Seamus. On seeing the graphs,
 		Seamus quickly corrects himself, and says  that $f$ has a local (and global)
 		maximum of $2$ at $0$, and that $g$ has a local (and global) minimum of $-2$ at $0$.
 		 		 		
 		\begin{figure}[!htb]
 			\begin{minipage}{.45\textwidth}
 				\begin{tikzpicture}[/pgf/declare function={f=-32*(x-1)*(x+1)/(( x-2)^2*(x+2)^2);}]
 					\begin{axis}[
 							framed,
 							xmin=-10,xmax=10,
 							ymin=-10,ymax=10,
 							xtick={-8,-6,...,8},
 							ytick={-8,-6,...,8},
 							grid=both,
 							width=\textwidth,
 						]
 						\addplot[pccplot] expression[domain=-10:-3.01]{f};
 						\addplot[pccplot] expression[domain=-1.45:1.45]{f};
 						\addplot[pccplot] expression[domain=3.01:10]{f};
 						\addplot[soldot] coordinates{(-1,0)(1,0)};
 					\end{axis}
 				\end{tikzpicture}
 				\caption{$y=f(x)$}
 				\label{rat:fig:minmax1}
 			\end{minipage}%
 			\hfill
 			\begin{minipage}{.45\textwidth}
 				\begin{tikzpicture}[/pgf/declare function={f=32*(x-1)*(x+1)/(( x-2)^2*(x+2)^2);}]
 					\begin{axis}[
 							framed,
 							xmin=-10,xmax=10,
 							ymin=-10,ymax=10,
 							xtick={-8,-6,...,8},
 							ytick={-8,-6,...,8},
 							grid=both,
 							width=\textwidth,
 						]
 						\addplot[pccplot] expression[domain=-10:-3.01]{f};
 						\addplot[pccplot] expression[domain=-1.45:1.45]{f};
 						\addplot[pccplot] expression[domain=3.01:10]{f};
 						\addplot[soldot] coordinates{(-1,0)(1,0)};
 					\end{axis}
 				\end{tikzpicture}
 				\caption{$y=g(x)$}
 				\label{rat:fig:minmax2}
 			\end{minipage}%
 		\end{figure}
 		 		 		
 		Seamus also notes that (in its domain) the function $f$ is always concave down, and
 		that (in its domain) the function $g$ is always concave up. Furthermore, Trang
 		observes that each function behaves like $\frac{1}{x^2}$ around each of its vertical
 		asymptotes, because each linear factor in the denominator is raised to the power $2$.
 		 		 		
 		\pccname{Oscar} stops by and reminds both students about the long-run behavior; according
 		to \cref{rat:def:longrun} since the degree of the denominator is greater than the
 		degree of the numerator (in both functions), each function has a horizontal asymptote
 		at $y=0$.
 	\end{pccexample}
 	 	 	
 	 	 	
 	\investigation*{}
 	%===================================
 	%   Author: Pettit/Hughes
 	%   Date:   March 2012
 	%===================================
 	\begin{problem}[The spaghetti incident]
 	The same Queen from \vref{exp:prob:queenschessboard} has recovered from
 	the rice experiments, and has called her loyal jester for another challenge.
 	 	 	
 	The jester has an $11-$inch piece of uncooked spaghetti that he puts on a table;
 	he uses a book to cover $\unit[1]{inch}$ of it so that
 	$\unit[10]{inches}$ hang over the edge. The jester then produces a box of $\unit{mg}$
 	weights that can be hung from the spaghetti.
 	 	 	
 	The jester says it will take $\unit[y]{mg}$ to break the spaghetti when hung
 	$\unit[x]{inches}$ from the edge, according to the rule $y=\frac{100}{x}$.
 	\begin{margintable}
 		\centering
 		\captionof{table}{}
 		\label{rat:tab:spaghetti}
 		\begin{tabular}{cc}
 			\beforeheading
 			\heading{$x$} & \heading{$y$} \\
 			\afterheading
 			$1$           &               \\\normalline
 			$2$           &               \\\normalline
 			$3$           &               \\\normalline
 			$4$           &               \\\normalline
 			$5$           &               \\\normalline
 			$6$           &               \\\normalline
 			$7$           &               \\\normalline
 			$8$           &               \\\normalline
 			$9$           &               \\\normalline
 			$10$          &               \\\lastline
 		\end{tabular}
 	\end{margintable}
 	\begin{subproblem}\label{rat:prob:spaggt1}
 		Help the Queen complete \cref{rat:tab:spaghetti}, and use $2$ digits after the decimal
 		where appropriate.
 		\begin{shortsolution}
 			\begin{tabular}[t]{ld{2}}
 				\beforeheading
 				\heading{$x$} & \heading{$y$} \\
 				\afterheading
 				$1$           & 100           \\\normalline
 				$2$           & 50            \\\normalline
 				$3$           & 33.33         \\\normalline
 				$4$           & 25            \\\normalline
 				$5$           & 20            \\\normalline
 				$6$           & 16.67         \\\normalline
 				$7$           & 14.29         \\\normalline
 				$8$           & 12.50         \\\normalline
 				$9$           & 11.11         \\\normalline
 				$10$          & 10            \\\lastline
 			\end{tabular}
 		\end{shortsolution}
 	\end{subproblem}
 	\begin{subproblem}
 		What do you notice about the number of $\unit{mg}$ that it takes to break
 		the spaghetti as $x$ increases?
 		\begin{shortsolution}
 			It seems that the number of $\unit{mg}$ that it takes to break the spaghetti decreases
 			as $x$ increases.
 		\end{shortsolution}
 	\end{subproblem}
 	\begin{subproblem}\label{rat:prob:spaglt1}
 		The Queen wonders what happens when $x$ gets very small| help the Queen construct
 		a table of values for $x$ and $y$ when $x=0.0001, 0.001, 0.01, 0.1, 0.5, 1$.
 		\begin{shortsolution}
 			\begin{tabular}[t]{d{2}l}
 				\beforeheading
 				\heading{$x$} & \heading{$y$} \\
 				\afterheading
 				0.0001        & $1000000$     \\\normalline
 				0.001         & $100000$      \\\normalline
 				0.01          & $10000$       \\\normalline
 				0.1           & $1000$        \\\normalline
 				0.5           & $200$         \\\normalline
 				1             & $100$         \\\lastline
 			\end{tabular}
 		\end{shortsolution}
 	\end{subproblem}
 	\begin{subproblem}
 		What do you notice about the number of $\unit{mg}$ that it takes to break the spaghetti
 		as $x\rightarrow 0$? Would it ever make sense to let $x=0$?
 		\begin{shortsolution}
 			The number of $\unit{mg}$ required to break the spaghetti increases as $x\rightarrow 0$.
 			We can not allow $x$ to be $0$, as we can not divide by $0$, and we can not
 			be $0$ inches from the edge of the table.
 		\end{shortsolution}
 	\end{subproblem}
 	\begin{subproblem}
 		Plot your results from \cref{rat:prob:spaggt1,rat:prob:spaglt1} on the same graph,
 		and join the points using a smooth curve| set the maximum value of $y$ as $200$, and
 		note that this necessarily means that you will not be able to plot all of the points.
 		\begin{shortsolution}
 			The graph of $y=\frac{100}{x}$ is shown below.
 			 			 			
 			\begin{tikzpicture}
 				\begin{axis}[
 						framed,
 						xmin=-2,xmax=11,
 						ymin=-20,ymax=200,
 						xtick={2,4,...,10},
 						ytick={20,40,...,180},
 						grid=major,
 						width=\solutionfigurewidth,
 					]
 					\addplot+[-] expression[domain=0.5:10]{100/x};
 					\addplot[soldot] coordinates{(0.5,200)(1,100)(2,50)(3,33.33)
 					(4,25)(5,20)(16.67)(7,14.29)(8,12.50)(9,11.11)(10,10)};
 				\end{axis}
 			\end{tikzpicture}
 		\end{shortsolution}
 	\end{subproblem}
 	\begin{subproblem}
 		Using your graph, observe what happens to $y$ as $x$ increases. If we could somehow
 		construct a piece of uncooked spaghetti that was $\unit[101]{inches}$ long, how many
 		$\unit{mg}$ would it take to break the spaghetti?
 		\begin{shortsolution}
 			As $x$ increases, $y\rightarrow 0$. If we could construct a piece of spaghetti
 			$\unit[101]{inches}$ long, it would only take $\unit[1]{mg}$ to break it $\left(\frac{100}{100}=1\right)$. Of course,
 			the weight of spaghetti would probably cause it to break without the weight.
 		\end{shortsolution}
 	\end{subproblem}
 	The Queen looks forward to more food-related investigations from her jester.
 	\end{problem}
 	 	 	
 	 	 	
 	 	 	
 	%===================================
 	%   Author: Adams (Hughes)
 	%   Date:   March 2012
 	%===================================
 	\begin{problem}[Debt Amortization]
 	To amortize a debt means to pay it off in a given length of time using
 	equal periodic payments. The payments include interest on the unpaid
 	balance. The following formula gives the monthly payment, $M$, in dollars
 	that is necessary to amortize a debt of $P$ dollars in $n$ months
 	at a monthly interest rate of $i$
 	\[
 		M=\frac{P\cdot i}{1-(1+i)^{-n}}
 	\]
 	Use this formula in each of the following problems.
 	\begin{subproblem}
 		What monthly payments are necessary on a credit card debt of \$2000 at
 		$\unit[1.5]{\%}$ monthly if you want to pay off the debt in $2$ years?
 		In one year? How much money will you save by paying off the debt in the
 		shorter amount of time?
 		\begin{shortsolution}
 			Paying off the debt in $2$ years, we use
 			\begin{align*}
 				M & = \frac{2000\cdot 0.015}{1-(1+0.015)^{-24}} \\
 				  & \approx 99.85                               
 			\end{align*}
 			The monthly payments are \$99.85.
 			 			 			
 			Paying off the debt in $1$ year, we use
 			\begin{align*}
 				M & = \frac{2000\cdot 0.015}{1-(1+0.015)^{-12}} \\
 				  & \approx 183.36                              
 			\end{align*}
 			The monthly payments are \$183.36
 			 			 			
 			In the $2$-year model we would pay a total of $\$99.85\cdot 12=\$2396.40$. In the
 			$1$-year model we would pay a total of $\$183.36\cdot 12=\$2200.32$. We would therefore
 			save $\$196.08$ if we went with the $1$-year model instead of the $2$-year model.
 		\end{shortsolution}
 	\end{subproblem}
 	\begin{subproblem}
 		To purchase a home, a family needs a loan of \$300,000 at $\unit[5.2]{\%}$
 		annual interest.  Compare a $20$ year loan to a $30$ year loan and make
 		a recommendation for the family.
 		(Note: when given an annual interest rate, it is a common business practice to divide by
 		$12$ to get a monthly rate.)
 		\begin{shortsolution}
 			For the $20$-year loan we use
 			\begin{align*}
 				M & = \frac{300000\cdot \frac{0.052}{12}}{1-\left( 1+\frac{0.052}{12} \right)^{-12\cdot 20}} \\
 				  & \approx 2013.16                                                                          
 			\end{align*}
 			The monthly payments are \$2013.16.
 			 			 			
 			For the $30$-year loan we use
 			\begin{align*}
 				M & = \frac{300000\cdot \frac{0.052}{12}}{1-\left( 1+\frac{0.052}{12} \right)^{-12\cdot 30}} \\
 				  & \approx 1647.33                                                                          
 			\end{align*}
 			The monthly payments are \$1647.33.
 			 			 			
 			The total amount paid during the $20$-year loan is $\$2013.16\cdot 12\cdot 20=\$483,158.40$.
 			The total amount paid during the $30$-year loan is $\$1647.33\cdot 12\cdot 30=\$593,038.80$.
 			 			 			
 			Recommendation: if you can afford the payments, choose the $20$-year loan.
 		\end{shortsolution}
 	\end{subproblem}
 	\begin{subproblem}
 		\pccname{Ellen} wants to make monthly payments of \$100 to pay off a debt of \$3000
 		at \unit[12]{\%} annual interest. How long will it take her to pay off the
 		debt?
 		\begin{shortsolution}
 			We are given $M=100$, $P=3000$, $i=0.01$, and we need to find $n$
 			in the equation
 			\[
 				100 = \frac{3000\cdot 0.01}{1-(1+0.01)^{-n}}
 			\]
 			Using logarithms, we find that $n\approx 36$. It will take
 			Ellen about $3$ years to pay off the debt.
 		\end{shortsolution}
 	\end{subproblem}
 	\begin{subproblem}
 		\pccname{Jake} is going to buy a new car. He puts \$2000 down and wants to finance the
 		remaining \$14,000. The dealer will offer him \unit[4]{\%} annual interest for
 		$5$ years, or a \$2000
 		rebate which he can use to reduce the amount of the loan and \unit[8]{\%}
 		annual interest for 5 years. Which should he choose?
 		\begin{shortsolution}
 			\begin{description}
 				\item[Option 1:] $\unit[4]{\%}$ annual interest for $5$ years on \$14,000.
 				This means that the monthly payments will be calculated using
 				\begin{align*}
 					M & = \frac{14000\cdot \frac{0.04}{12}}{1-\left( 1+\frac{0.04}{12} \right)^{-12\cdot 5}} \\
 					  & \approx 257.83                                                                       
 				\end{align*}
 				The monthly payments will be $\$257.83$. The total amount paid will be
 				$\$257.83\cdot 5\cdot 12=\$15,469.80$, of which $\$1469.80$ is interest.
 				\item[Option 2:] $\unit[8]{\%}$ annual interest for $5$ years on \$12,000.
 				This means that the monthly payments will be calculated using
 				\begin{align*}
 					M & = \frac{12000\cdot \frac{0.08}{12}}{1-\left( 1+\frac{0.08}{12} \right)^{-12\cdot 5}} \\
 					  & \approx 243.32                                                                       
 				\end{align*}
 				The monthly payments will be $\$243.32$. The total amount paid
 				will be $\$243.32\cdot 5\cdot 12 =\$14,599.20$, of which $\$2599.2$ is
 				interest.
 			\end{description}
 			Jake should choose option 1 to minimize the amount of interest
 			he has to pay.
 		\end{shortsolution}
 	\end{subproblem}
 	\end{problem}
 	 	 	
 	\begin{exercises}
 	%===================================
 	%   Author: Hughes
 	%   Date:   March 2012
 	%===================================
 	\begin{problem}[Rational or not]
 	Decide if each of the following functions are rational or not. If
 	they are rational, state their domain.
 	\begin{multicols}{3}
 		\begin{subproblem}
 			$r(x)=\dfrac{3}{x}$
 			\begin{shortsolution}
 				$r$ is rational; the domain of $r$ is $(-\infty,0)\cup (0,\infty)$.
 			\end{shortsolution}
 		\end{subproblem}
 		\begin{subproblem}
 			$s(y)=\dfrac{y}{6}$
 			\begin{shortsolution}
 				$s$ is not rational ($s$ is linear).
 			\end{shortsolution}
 		\end{subproblem}
 		\begin{subproblem}
 			$t(z)=\dfrac{4-x}{7-8z}$
 			\begin{shortsolution}
 				$t$ is rational; the domain of $t$ is $\left( -\infty,\dfrac{7}{8} \right)\cup \left( \dfrac{7}{8},\infty \right)$.
 			\end{shortsolution}
 		\end{subproblem}
 		\begin{subproblem}
 			$u(w)=\dfrac{w^2}{(w-3)(w+4)}$
 			\begin{shortsolution}
 				$u$ is rational; the domain of $w$ is $(-\infty,-4)\cup(-4,3)\cup(3,\infty)$.
 			\end{shortsolution}
 		\end{subproblem}
 		\begin{subproblem}
 			$v(x)=\dfrac{4}{(x-2)^2}$
 			\begin{shortsolution}
 				$v$ is rational; the domain of $v$ is $(-\infty,2)\cup(2,\infty)$.
 			\end{shortsolution}
 		\end{subproblem}
 		\begin{subproblem}
 			$w(x)=\dfrac{9-x}{x+17}$
 			\begin{shortsolution}
 				$w$ is rational; the domain of $w$ is $(-\infty,-17)\cup(-17,\infty)$.
 			\end{shortsolution}
 		\end{subproblem}
 		\begin{subproblem}
 			$a(x)=x^2+4$
 			\begin{shortsolution}
 				$a$ is not rational ($a$ is quadratic, or a polynomial of degree $2$).
 			\end{shortsolution}
 		\end{subproblem}
 		\begin{subproblem}
 			$b(y)=3^y$
 			\begin{shortsolution}
 				$b$ is not rational ($b$ is exponential).
 			\end{shortsolution}
 		\end{subproblem}
 		\begin{subproblem}
 			$c(z)=\dfrac{z^2}{z^3}$
 			\begin{shortsolution}
 				$c$ is rational; the domain of $c$ is $(-\infty,0)\cup (0,\infty)$.
 			\end{shortsolution}
 		\end{subproblem}
 		\begin{subproblem}
 			$d(x)=x^2(x+3)(5x-7)$
 			\begin{shortsolution}
 				$d$ is not rational ($d$ is a polynomial).
 			\end{shortsolution}
 		\end{subproblem}
 		\begin{subproblem}
 			$e(\alpha)=\dfrac{\alpha^2}{\alpha^2-1}$
 			\begin{shortsolution}
 				$e$ is rational; the domain of $e$ is $(-\infty,-1)\cup(-1,1)\cup(1,\infty)$.
 			\end{shortsolution}
 		\end{subproblem}
 		\begin{subproblem}
 			$f(\beta)=\dfrac{3}{4}$
 			\begin{shortsolution}
 				$f$ is not rational ($f$ is constant).
 			\end{shortsolution}
 		\end{subproblem}
 	\end{multicols}
 	\end{problem}
 	%===================================
 	%   Author: Hughes
 	%   Date:   March 2012
 	%===================================
 	\begin{problem}[Function evaluation]
 	Let $r$ be the function that has formula
 	\[
 		r(x)=\frac{(x-2)(x+3)}{(x+5)(x-7)}
 	\]
 	Evaluate each of the following (if possible); if the value is undefined,
 	then state so.
 	\begin{multicols}{4}
 		\begin{subproblem}
 			$r(0)$
 			\begin{shortsolution}
 				$\begin{aligned}[t]
 					r(0) & =\frac{(0-2)(0+3)}{(0+5)(0-7)} \\
 					     & =\frac{-6}{-35}                \\
 					     & =\frac{6}{35}                  
 				\end{aligned}$
 			\end{shortsolution}
 		\end{subproblem}
 		\begin{subproblem}
 			$r(1)$
 			\begin{shortsolution}
 				$\begin{aligned}[t]
 					r(1) & =\frac{(1-2)(1+3)}{(1+5)(1-7)} \\
 					     & =\frac{-4}{-36}                \\
 					     & =\frac{1}{9}                   
 				\end{aligned}$
 			\end{shortsolution}
 		\end{subproblem}
 		\begin{subproblem}
 			$r(2)$
 			\begin{shortsolution}
 				$\begin{aligned}[t]
 					r(2) & =\frac{(2-2)(2+3)}{(2+5)(2-7)} \\
 					     & = \frac{0}{-50}                \\
 					     & =0                             
 				\end{aligned}$
 			\end{shortsolution}
 		\end{subproblem}
 		\begin{subproblem}
 			$r(4)$
 			\begin{shortsolution}
 				$\begin{aligned}[t]
 					r(4) & =\frac{(4-2)(4+3)}{(4+5)(4-7)} \\
 					     & =\frac{14}{-27}                \\
 					     & =-\frac{14}{27}                
 				\end{aligned}$
 			\end{shortsolution}
 		\end{subproblem}
 		\begin{subproblem}
 			$r(7)$
 			\begin{shortsolution}
 				$\begin{aligned}[t]
 					r(7) & =\frac{(7-2)(7+3)}{(7+5)(7-7)} \\
 					     & =\frac{50}{0}                  
 				\end{aligned}$
 				 				 				
 				$r(7)$ is undefined.
 			\end{shortsolution}
 		\end{subproblem}
 		\begin{subproblem}
 			$r(-3)$
 			\begin{shortsolution}
 				$\begin{aligned}[t]
 					r(-3) & =\frac{(-3-2)(-3+3)}{(-3+5)(-3-7)} \\
 					      & =\frac{0}{-20}                     \\
 					      & =0                                 
 				\end{aligned}$
 			\end{shortsolution}
 		\end{subproblem}
 		\begin{subproblem}
 			$r(-5)$
 			\begin{shortsolution}
 				$\begin{aligned}[t]
 					r(-5) & =\frac{(-5-2)(-5+3)}{(-5+5)(-5-7)} \\
 					      & =\frac{14}{0}                      
 				\end{aligned}$
 				 				 				
 				$r(-5)$ is undefined.
 			\end{shortsolution}
 		\end{subproblem}
 		\begin{subproblem}
 			$r\left( \frac{1}{2} \right)$
 			\begin{shortsolution}
 				$\begin{aligned}[t]
 					r\left( \frac{1}{2} \right) & = \frac{\left( \frac{1}{2}-2 \right)\left( \frac{1}{2}+3 \right)}{\left( \frac{1}{2}+5 \right)\left( \frac{1}{2}-7 \right)} \\
 					                            & =\frac{-\frac{3}{2}\cdot\frac{7}{2}}{\frac{11}{2}\left( -\frac{13}{2} \right)}                                              \\
 					                            & =\frac{-\frac{21}{4}}{-\frac{143}{4}}                                                                                       \\
 					                            & =\frac{37}{143}                                                                                                             
 				\end{aligned}$
 			\end{shortsolution}
 		\end{subproblem}
 	\end{multicols}
 	\end{problem}
 	%===================================
 	%   Author: Hughes
 	%   Date:   March 2012
 	%===================================
 	\begin{problem}[Holes or asymptotes?]
 	State the domain of each of the following rational functions. Identify
 	any holes or asymptotes.
 	\begin{multicols}{3}
 		\begin{subproblem}
 			$f(x)=\dfrac{12}{x-2}$
 			\begin{shortsolution}
 				$f$ has a vertical asymptote at $2$; the domain of $f$ is $(-\infty,2)\cup (2,\infty)$.
 			\end{shortsolution}
 		\end{subproblem}
 		\begin{subproblem}
 			$g(x)=\dfrac{x^2+x}{(x+1)(x-2)}$
 			\begin{shortsolution}
 				$g$ has a vertical asymptote at $2$, and a hole at $-1$; the domain of $g$ is $(-\infty,-1)\cup(-1,2)\cup(2,\infty)$.
 			\end{shortsolution}
 		\end{subproblem}
 		\begin{subproblem}
 			$h(x)=\dfrac{x^2+5x+4}{x^2+x-12}$
 			\begin{shortsolution}
 				$h$ has a vertical asymptote at $3$, and a whole at $-4$; the domain of $h$ is $(-\infty,-4)\cup(-4,3)\cup(3,\infty)$.
 			\end{shortsolution}
 		\end{subproblem}
 		\begin{subproblem}
 			$k(z)=\dfrac{z+2}{2z-3}$
 			\begin{shortsolution}
 				$k$ has a vertical asymptote at $\dfrac{3}{2}$; the domain of $k$ is $\left( -\infty,\dfrac{3}{2} \right)\cup\left( \dfrac{3}{2},\infty \right)$.
 			\end{shortsolution}
 		\end{subproblem}
 		\begin{subproblem}
 			$l(w)=\dfrac{w}{w^2+1}$
 			\begin{shortsolution}
 				$l$ does not have any vertical asymptotes nor holes; the domain of $w$ is $(-\infty,\infty)$.
 			\end{shortsolution}
 		\end{subproblem}
 		\begin{subproblem}
 			$m(t)=\dfrac{14}{13-t^2}$
 			\begin{shortsolution}
 				$m$ has vertical asymptotes at $\pm\sqrt{13}$; the domain of $m$ is $(-\infty,\sqrt{13})\cup(-\sqrt{13},\sqrt{13})\cup(\sqrt{13},\infty)$.
 			\end{shortsolution}
 		\end{subproblem}
 	\end{multicols}
 	\end{problem}
 	 	 	
 	%===================================
 	%   Author: Hughes
 	%   Date:   May 2011
 	%===================================
 	\begin{problem}[Find a formula from a graph]
 	Consider the rational functions graphed in \cref{rat:fig:findformula}. Find
 	the vertical asymptotes for each function, together with any zeros, and
 	give a possible formula for each.
 	\begin{shortsolution}
 		\begin{itemize}
 			\item \Vref{rat:fig:formula1}: possible formula is $r(x)=\dfrac{1}{x+5}$
 			\item \Vref{rat:fig:formula2}: possible formula is $r(x)=\dfrac{(x+3)}{(x-5)}$
 			\item \Vref{rat:fig:formula3}: possible formula is $r(x)=\dfrac{1}{(x-4)(x+3)}$.
 		\end{itemize}
 	\end{shortsolution}
 	\end{problem}
 	 	 	
 	\begin{figure}[!htb]
 		\begin{widepage}
 		\setlength{\figurewidth}{0.3\textwidth}
 		\begin{subfigure}{\figurewidth}
 			\begin{tikzpicture}[/pgf/declare function={f=1/(x+4);}]
 				\begin{axis}[
 						framed,
 						xmin=-10,xmax=10,
 						ymin=-6,ymax=6,
 						xtick={-8,-6,...,8},
 						minor ytick={-4,-3,...,4},
 						grid=both,
 						width=\textwidth,
 					]
 					\addplot[pccplot] expression[domain=-10:-4.16667,samples=50]{f};
 					\addplot[pccplot] expression[domain=-3.83333:10,samples=50]{f};
 					\addplot[asymptote,domain=-6:6]({-4},{x});
 				\end{axis}
 			\end{tikzpicture}
 			\caption{}
 			\label{rat:fig:formula1}
 		\end{subfigure}
 		\hfill
 		\begin{subfigure}{\figurewidth}
 			\begin{tikzpicture}[/pgf/declare function={f=(x+3)/(x-5);}]
 				\begin{axis}[
 						framed,
 						xmin=-10,xmax=10,
 						ymin=-6,ymax=6,
 						xtick={-8,-6,...,8},
 						minor ytick={-4,-3,...,4},
 						grid=both,
 						width=\textwidth,
 					]
 					\addplot[pccplot] expression[domain=-10:3.85714]{f};
 					\addplot[pccplot] expression[domain=6.6:10]{f};
 					\addplot[soldot] coordinates{(-3,0)};
 					\addplot[asymptote,domain=-6:6]({5},{x});
 					\addplot[asymptote,domain=-10:10]({x},{1});
 				\end{axis}
 			\end{tikzpicture}
 			\caption{}
 			\label{rat:fig:formula2}
 		\end{subfigure}
 		\hfill
 		\begin{subfigure}{\figurewidth}
 			\begin{tikzpicture}[/pgf/declare function={f=1/((x-4)*(x+3));}]
 				\begin{axis}[
 						framed,
 						xmin=-10,xmax=10,
 						ymin=-3,ymax=3,
 						xtick={-8,-6,...,8},
 						minor ytick={-4,-3,...,4},
 						grid=both,
 						width=\textwidth,
 					]
 					\addplot[pccplot] expression[domain=-10:-3.0473]{f};
 					\addplot[pccplot] expression[domain=-2.95205:3.95205]{f};
 					\addplot[pccplot] expression[domain=4.0473:10]{f};
 					\addplot[asymptote,domain=-3:3]({-3},{x});
 					\addplot[asymptote,domain=-3:3]({4},{x});
 					\addplot[asymptote,domain=-10:10]({x},{0});
 				\end{axis}
 			\end{tikzpicture}
 			\caption{}
 			\label{rat:fig:formula3}
 		\end{subfigure}
 		\caption{}
 		\label{rat:fig:findformula}
 		\end{widepage}
 	\end{figure}
 	 	 	
 	%===================================
 	%   Author: Hughes
 	%   Date:   May 2011
 	%===================================
 	\begin{problem}[Find a formula from a description]
 	In each of the following problems, give a formula of a rational
 	function that has the listed properties.
 	\begin{subproblem}
 		Vertical asymptote at $2$.
 		\begin{shortsolution}
 			Possible option: $r(x)=\dfrac{1}{x-2}$. Note that we could multiply the
 			numerator or denominator by any real number and still have the desired properties.
 		\end{shortsolution}
 	\end{subproblem}
 	\begin{subproblem}
 		Vertical asymptote at $5$.
 		\begin{shortsolution}
 			Possible option: $r(x)=\dfrac{1}{x-5}$. Note that we could multiply the
 			numerator or denominator by any real number and still have the desired properties.
 		\end{shortsolution}
 	\end{subproblem}
 	\begin{subproblem}
 		Vertical asymptote at $-2$, and zero at $6$.
 		\begin{shortsolution}
 			Possible option: $r(x)=\dfrac{x-6}{x+2}$. Note that we could multiply the
 			numerator or denominator by any real number and still have the desired properties.
 		\end{shortsolution}
 	\end{subproblem}
 	\begin{subproblem}
 		Zeros at $2$ and $-5$ and vertical asymptotes at $1$ and $-7$.
 		\begin{shortsolution}
 			Possible option: $r(x)=\dfrac{(x-2)(x+5)}{(x-1)(x+7)}$. Note that we could multiply the
 			numerator or denominator by any real number and still have the desired properties.
 		\end{shortsolution}
 	\end{subproblem}
 	\end{problem}
 	 	 	
 	%===================================
 	%   Author: Hughes
 	%   Date:   May 2011
 	%===================================
 	\begin{problem}[Given formula, find horizontal asymptotes]
 	Each of the following functions has a horizontal asymptote. Write the equation
 	of the horizontal asymptote for each function.
 	\begin{multicols}{3}
 		\begin{subproblem}
 			$f(x) = \dfrac{1}{x}$
 			\begin{shortsolution}
 				$y=0$
 			\end{shortsolution}
 		\end{subproblem}
 		\begin{subproblem}
 			$g(x) = \dfrac{2x+3}{x}$
 			\begin{shortsolution}
 				$y=2$
 			\end{shortsolution}
 		\end{subproblem}
 		\begin{subproblem}
 			$h(x) = \dfrac{x^2+2x}{x^2+3}$
 			\begin{shortsolution}
 				$y=1$
 			\end{shortsolution}
 		\end{subproblem}
 		\begin{subproblem}
 			$k(x) = \dfrac{x^2+7}{x}$
 			\begin{shortsolution}
 				$y=1$
 			\end{shortsolution}
 		\end{subproblem}
 		\begin{subproblem}
 			$l(x)=\dfrac{3x-2}{5x+8}$
 			\begin{shortsolution}
 				$y=\dfrac{3}{5}$
 			\end{shortsolution}
 		\end{subproblem}
 		\begin{subproblem}
 			$m(x)=\dfrac{3x-2}{5x^2+8}$
 			\begin{shortsolution}
 				$y=0$
 			\end{shortsolution}
 		\end{subproblem}
 		\begin{subproblem}
 			$n(x)=\dfrac{(6x+1)(x-7)}{(11x-8)(x-5)}$
 			\begin{shortsolution}
 				$y=\dfrac{6}{11}$
 			\end{shortsolution}
 		\end{subproblem}
 		\begin{subproblem}
 			$p(x)=\dfrac{19x^3}{5-x^4}$
 			\begin{shortsolution}
 				$y=0$
 			\end{shortsolution}
 		\end{subproblem}
 		\begin{subproblem}
 			$q(x)=\dfrac{14x^2+x}{1-7x^2}$
 			\begin{shortsolution}
 				$y=-2$
 			\end{shortsolution}
 		\end{subproblem}
 	\end{multicols}
 	\end{problem}
 	 	 	
 	%===================================
 	%   Author: Hughes
 	%   Date:   May 2012
 	%===================================
 	\begin{problem}[Given horizontal asymptotes, find formula]
 	In each of the following problems, give a formula for a function that
 	has the given horizontal asymptote. Note that there may be more than one option.
 	\begin{multicols}{4}
 		\begin{subproblem}
 			$y=7$
 			\begin{shortsolution}
 				Possible option: $f(x)=\dfrac{7(x-2)}{x+1}$. Note that there
 				are other options, provided that the degree of the numerator is the same as the degree
 				of the denominator, and that the ratio of the leading
 				coefficients is $7$.
 			\end{shortsolution}
 		\end{subproblem}
 		\begin{subproblem}
 			$y=-1$
 			\begin{shortsolution}
 				Possible option: $f(x)=\dfrac{5-x^2}{x^2+10}$. Note that there
 				are other options, provided that the degree of the numerator is the same as the degree
 				of the denominator, and that the ratio of the leading
 				coefficients is $10$.
 			\end{shortsolution}
 		\end{subproblem}
 		\begin{subproblem}
 			$y=53$
 			\begin{shortsolution}
 				Possible option: $f(x)=\dfrac{53x^3}{x^3+4x^2-7}$. Note that there
 				are other options, provided that the degree of the numerator is the same as the degree
 				of the denominator, and that the ratio of the leading
 				coefficients is $53$.
 			\end{shortsolution}
 		\end{subproblem}
 		\begin{subproblem}
 			$y=-17$
 			\begin{shortsolution}
 				Possible option: $f(x)=\dfrac{34(x+2)}{7-2x}$. Note that there
 				are other options, provided that the degree of the numerator is the same as the degree
 				of the denominator, and that the ratio of the leading
 				coefficients is $-17$.
 			\end{shortsolution}
 		\end{subproblem}
 		\begin{subproblem}
 			$y=\dfrac{3}{2}$
 			\begin{shortsolution}
 				Possible option: $f(x)=\dfrac{3x+4}{2(x+1)}$. Note that there
 				are other options, provided that the degree of the numerator is the same as the degree
 				of the denominator, and that the ratio of the leading
 				coefficients is $\dfrac{3}{2}$.
 			\end{shortsolution}
 		\end{subproblem}
 		\begin{subproblem}
 			$y=0$
 			\begin{shortsolution}
 				Possible option: $f(x)=\dfrac{4}{x}$. Note that there
 				are other options, provided that the degree of the numerator is less than the degree
 				of the denominator.
 			\end{shortsolution}
 		\end{subproblem}
 		\begin{subproblem}
 			$y=-1$
 			\begin{shortsolution}
 				Possible option: $f(x)=\dfrac{10x}{5-10x}$. Note that there
 				are other options, provided that the degree of the numerator is the same as the degree
 				of the denominator, and that the ratio of the leading
 				coefficients is $-1$.
 			\end{shortsolution}
 		\end{subproblem}
 		\begin{subproblem}
 			$y=2$
 			\begin{shortsolution}
 				Possible option: $f(x)=\dfrac{8x-3}{4x+1}$. Note that there
 				are other options, provided that the degree of the numerator is the same as the degree
 				of the denominator, and that the ratio of the leading
 				coefficients is $2$.
 			\end{shortsolution}
 		\end{subproblem}
 	\end{multicols}
 	\end{problem}
 	 	 	
 	%===================================
 	%   Author: Hughes
 	%   Date:   May 2011
 	%===================================
 	\begin{problem}[Find a formula from a description]
 	In each of the following problems, give a formula for a function that
 	has the prescribed properties. Note that there may be more than one option.
 	\begin{subproblem}
 		$f(x)\rightarrow 3$ as $x\rightarrow\pm\infty$.
 		\begin{shortsolution}
 			Possible option: $f(x)=\dfrac{3(x-2)}{x+7}$. Note that
 			the zero and asymptote of $f$ could be changed, and $f$ would still have the desired properties.
 		\end{shortsolution}
 	\end{subproblem}
 	\begin{subproblem}
 		$r(x)\rightarrow -4$ as $x\rightarrow\pm\infty$.
 		\begin{shortsolution}
 			Possible option: $r(x)=\dfrac{-4(x-2)}{x+7}$. Note that
 			the zero and asymptote of $r$ could be changed, and $r$ would still have the desired properties.
 		\end{shortsolution}
 	\end{subproblem}
 	\begin{subproblem}
 		$k(x)\rightarrow 2$ as $x\rightarrow\pm\infty$, and $k$ has vertical asymptotes at $-3$ and $5$.
 		\begin{shortsolution}
 			Possible option: $k(x)=\dfrac{2x^2}{(x+3)(x-5)}$. Note that the denominator
 			must have the given factors; the numerator could be any degree $2$ polynomial, provided the
 			leading coefficient is $2$.
 		\end{shortsolution}
 	\end{subproblem}
 	\end{problem}
 	 	 	
 	%===================================
 	%   Author: Hughes
 	%   Date:   Feb 2011
 	%===================================
 	\begin{problem}
 	Let $r$ be the rational function that has
 	\[
 		r(x) = \frac{(x+2)(x-1)}{(x+3)(x-4)}
 	\]
 	Each of the following questions are in relation to this function.
 	\begin{subproblem}
 		What is the vertical intercept of this function? State your answer as an
 		ordered pair. \index{rational functions!vertical intercept}
 		\begin{shortsolution}
 			$\left(0,\frac{1}{6}\right)$
 		\end{shortsolution}
 	\end{subproblem}
 	\begin{subproblem}\label{rat:prob:rational}
 		What values of $x$ make the denominator equal to $0$?
 		\begin{shortsolution}
 			$-3,4$
 		\end{shortsolution}
 	\end{subproblem}
 	\begin{subproblem}
 		Use your answer to \cref{rat:prob:rational} to write the domain of the function in
 		both interval, and set builder notation. %\index{rational functions!domain}\index{domain!rational functions}
 		\begin{shortsolution}
 			Interval notation: $(-\infty,-3)\cup (-3,4)\cup (4,\infty)$.
 			Set builder: $\{x|x\ne -3, \mathrm{and}\, x\ne 4\}$
 		\end{shortsolution}
 	\end{subproblem}
 	\begin{subproblem}
 		What are the vertical asymptotes of the function? State your answers in
 		the form $x=$
 		\begin{shortsolution}
 			$x=-3$ and $x=4$
 		\end{shortsolution}
 	\end{subproblem}
 	\begin{subproblem}\label{rat:prob:zeroes}
 		What values of $x$ make the numerator equal to $0$?
 		\begin{shortsolution}
 			$-2,1$
 		\end{shortsolution}
 	\end{subproblem}
 	\begin{subproblem}
 		Use your answer to \cref{rat:prob:zeroes} to write the horizontal intercepts of
 		$r$ as ordered pairs.
 		\begin{shortsolution}
 			$(-2,0)$ and $(1,0)$
 		\end{shortsolution}
 	\end{subproblem}
 	\end{problem}
 	 	 	
 	 	 	
 	%===================================
 	%   Author: Hughes
 	%   Date:   May 2011
 	%===================================
 	\begin{problem}[Holes]
 	\pccname{Josh} and \pccname{Pedro} are discussing the function
 	\[
 		r(x)=\frac{x^2-1}{(x+3)(x-1)}
 	\]
 	\begin{subproblem}
 		What is the domain of $r$?
 		\begin{shortsolution}
 			The domain of $r$ is $(-\infty,-3)\cup(-3,1)\cup(1,\infty)$.
 		\end{shortsolution}
 	\end{subproblem}
 	\begin{subproblem}
 		Josh notices that the numerator can be factored- can you see how?
 		\begin{shortsolution}
 			$(x^2-1)=(x-1)(x+1)$
 		\end{shortsolution}
 	\end{subproblem}
 	\begin{subproblem}
 		Pedro asks, `Doesn't that just mean that
 		\[
 			r(x)=\frac{x+1}{x+3}
 		\]
 		for all values of $x$?' Josh says, `Nearly\ldots but not for all values of $x$'.
 		What does Josh mean?
 		\begin{shortsolution}
 			$r(x)=\dfrac{x+1}{x+3}$ provided that $x\ne -1$.
 		\end{shortsolution}
 	\end{subproblem}
 	\begin{subproblem}
 		Where does $r$ have vertical asymptotes, and where does it have holes?
 		\begin{shortsolution}
 			The function $r$ has a vertical asymptote at $-3$, and a hole at $1$.
 		\end{shortsolution}
 	\end{subproblem}
 	\begin{subproblem}
 		Sketch a graph of $r$.
 		\begin{shortsolution}
 			A graph of $r$ is shown below.
 			 			 			
 			\begin{tikzpicture}
 				\begin{axis}[
 						framed,
 						xmin=-10,xmax=10,
 						ymin=-10,ymax=10,
 						xtick={-8,-6,...,8},
 						ytick={-8,-6,...,8},
 						grid=both,
 						width=\solutionfigurewidth,
 					]
 					\addplot[pccplot] expression[domain=-10:-3.25]{(x+1)/(x+3)};
 					\addplot[pccplot] expression[domain=-2.75:10]{(x+1)/(x+3)};
 					\addplot[asymptote,domain=-10:10]({-3},{x});
 					\addplot[holdot]coordinates{(1,0.5)};
 				\end{axis}
 			\end{tikzpicture}
 		\end{shortsolution}
 	\end{subproblem}
 	\end{problem}
 	 	 	
 	%===================================
 	%   Author: Hughes
 	%   Date:   July 2012
 	%===================================
 	\begin{problem}[Function algebra]
 	Let $r$ and $s$ be the rational functions that have formulas
 	\[
 		r(x)=\frac{2-x}{x+3}, \qquad s(x)=\frac{x^2}{x-4}
 	\]
 	Evaluate each of the following (if possible).
 	\begin{multicols}{4}
 		\begin{subproblem}
 			$(r+s)(5)$
 			\begin{shortsolution}
 				$\frac{197}{8}$
 			\end{shortsolution}
 		\end{subproblem}
 		\begin{subproblem}
 			$(r-s)(3)$
 			\begin{shortsolution}
 				$\frac{53}{6}$
 			\end{shortsolution}
 		\end{subproblem}
 		\begin{subproblem}
 			$(r\cdot s)(4)$
 			\begin{shortsolution}
 				Undefined.
 			\end{shortsolution}
 		\end{subproblem}
 		\begin{subproblem}
 			$\left( \frac{r}{s} \right)(1)$
 			\begin{shortsolution}
 				$-\frac{3}{4}$
 			\end{shortsolution}
 		\end{subproblem}
 	\end{multicols}
 	\end{problem}
 	 	 	
 	 	 	
 	%===================================
 	%   Author: Hughes
 	%   Date:   July 2012
 	%===================================
 	\begin{problem}[Transformations: given the transformation, find the formula]
 	Let $r$ be the rational function that has formula.
 	\[
 		r(x)=\frac{x+5}{2x-3}
 	\]
 	In each of the following problems apply the given transformation to the function $r$ and
 	write a formula for the transformed version of $r$.
 	\begin{multicols}{2}
 		\begin{subproblem}
 			Shift $r$ to the right by $3$ units.
 			\begin{shortsolution}
 				$r(x-3)=\frac{x+2}{2x-9}$
 			\end{shortsolution}
 		\end{subproblem}
 		\begin{subproblem}
 			Shift $r$ to the left by $4$ units.
 			\begin{shortsolution}
 				$r(x+4)=\frac{x+9}{2x+5}$
 			\end{shortsolution}
 		\end{subproblem}
 		\begin{subproblem}
 			Shift $r$ up by $\pi$ units.
 			\begin{shortsolution}
 				$r(x)+\pi=\frac{x+5}{2x-3}+\pi$
 			\end{shortsolution}
 		\end{subproblem}
 		\begin{subproblem}
 			Shift $r$ down by $17$ units.
 			\begin{shortsolution}
 				$r(x)-17=\frac{x+5}{2x-3}-17$
 			\end{shortsolution}
 		\end{subproblem}
 		\begin{subproblem}
 			Reflect $r$ over the horizontal axis.
 			\begin{shortsolution}
 				$-r(x)=-\frac{x+5}{2x-3}$
 			\end{shortsolution}
 		\end{subproblem}
 		\begin{subproblem}
 			Reflect $r$ over the vertical axis.
 			\begin{shortsolution}
 				$r(-x)=\frac{x-5}{2x+3}$
 			\end{shortsolution}
 		\end{subproblem}
 	\end{multicols}
 	\end{problem}
 	 	 	
 	 	 	
 	%===================================
 	%   Author: Hughes
 	%   Date:   May 2011
 	%===================================
 	\begin{problem}[Find a formula from a table]\label{rat:prob:findformula}
 	\Crefrange{rat:tab:findformular}{rat:tab:findformulau} show values of rational functions $r$, $q$, $s$,
 	and $t$. Assume that any values marked with an X are undefined.
 	 	 	
 	\begin{table}[!htb]
 		\begin{widepage}
 		\centering
 		\caption{Tables for \cref{rat:prob:findformula}}
 		\label{rat:tab:findformula}
 		\begin{subtable}{.2\textwidth}
 			\centering
 			\caption{$y=r(x)$}
 			\label{rat:tab:findformular}
 			\begin{tabular}{rr}
 				\beforeheading
 				$x$  & $y$                \\ \afterheading
 				$-4$ & $\nicefrac{7}{2}$  \\\normalline
 				$-3$ & $-18$              \\\normalline
 				$-2$ & X                  \\\normalline
 				$-1$ & $-4$               \\\normalline
 				$0$  & $\nicefrac{-3}{2}$ \\\normalline
 				$1$  & $\nicefrac{-2}{3}$ \\\normalline
 				$2$  & $\nicefrac{-1}{4}$ \\\normalline
 				$3$  & $0$                \\\normalline
 				$4$  & $\nicefrac{1}{6}$  \\\lastline
 			\end{tabular}
 		\end{subtable}
 		\hfill
 		\begin{subtable}{.2\textwidth}
 			\centering
 			\caption{$y=s(x)$}
 			\label{rat:tab:findformulas}
 			\begin{tabular}{rr}
 				\beforeheading
 				$x$  & $y$                 \\ \afterheading
 				$-4$ & $\nicefrac{-2}{21}$ \\\normalline
 				$-3$ & $\nicefrac{-1}{12}$ \\\normalline
 				$-2$ & $0$                 \\\normalline
 				$-1$ & X                   \\\normalline
 				$0$  & $\nicefrac{-2}{3}$  \\\normalline
 				$1$  & $\nicefrac{-3}{4}$  \\\normalline
 				$2$  & $\nicefrac{-4}{3}$  \\\normalline
 				$3$  & X                   \\\normalline
 				$4$  & $\nicefrac{6}{5}$   \\\lastline
 			\end{tabular}
 		\end{subtable}
 		\hfill
 		\begin{subtable}{.2\textwidth}
 			\centering
 			\caption{$y=t(x)$}
 			\label{rat:tab:findformulat}
 			\begin{tabular}{rr}
 				\beforeheading
 				$x$  & $y$               \\ \afterheading
 				$-4$ & $\nicefrac{3}{5}$ \\\normalline
 				$-3$ & $0$               \\\normalline
 				$-2$ & X                 \\\normalline
 				$-1$ & $3$               \\\normalline
 				$0$  & $3$               \\\normalline
 				$1$  & X                 \\\normalline
 				$2$  & $0$               \\\normalline
 				$3$  & $\nicefrac{3}{5}$ \\\normalline
 				$4$  & $\nicefrac{7}{9}$ \\\lastline
 			\end{tabular}
 		\end{subtable}
 		\hfill
 		\begin{subtable}{.2\textwidth}
 			\centering
 			\caption{$y=u(x)$}
 			\label{rat:tab:findformulau}
 			\begin{tabular}{rr}
 				\beforeheading
 				$x$  & $y$                \\ \afterheading
 				$-4$ & $\nicefrac{16}{7}$ \\\normalline
 				$-3$ & X                  \\\normalline
 				$-2$ & $-\nicefrac{4}{5}$ \\\normalline
 				$-1$ & $-\nicefrac{1}{8}$ \\\normalline
 				$0$  & $0$                \\\normalline
 				$1$  & $-\nicefrac{1}{8}$ \\\normalline
 				$2$  & $-\nicefrac{4}{5}$ \\\normalline
 				$3$  & X                  \\\normalline
 				$4$  & $\nicefrac{16}{7}$ \\\lastline
 			\end{tabular}
 		\end{subtable}
 		\end{widepage}
 	\end{table}
 	\begin{subproblem}
 		Given that the formula for $r(x)$ has the form $r(x)=\dfrac{x-A}{x-B}$, use \cref{rat:tab:findformular}
 		to find values of $A$ and $B$.
 		\begin{shortsolution}
 			$A=3$ and $B=-2$, so $r(x)=\dfrac{x-3}{x+2}$.
 		\end{shortsolution}
 	\end{subproblem}
 	\begin{subproblem}
 		Check your formula by computing $r(x)$ at the values specified in the table.
 		\begin{shortsolution}
 			$\begin{aligned}[t]
 				r(-4) & = \frac{-4-3}{-4+2} \\
 				      & = \frac{7}{2}       \\
 			\end{aligned}$
 			 			 			
 			$r(-3)=\ldots$ etc
 		\end{shortsolution}
 	\end{subproblem}
 	\begin{subproblem}
 		The function $s$ in \cref{rat:tab:findformulas} has two vertical asymptotes and one zero.
 		Can you find a formula for $s(x)$?
 		\begin{shortsolution}
 			$s(x)=\dfrac{x+2}{(x-3)(x+1)}$
 		\end{shortsolution}
 	\end{subproblem}
 	\begin{subproblem}
 		Check your formula by computing $s(x)$ at the values specified in the table.
 		\begin{shortsolution}
 			$\begin{aligned}[t]
 				s(-4) & =\frac{-4+2}{(-4-3)(-4+1)} \\
 				      & =-\frac{2}{21}             
 			\end{aligned}$
 			 			 			
 			$s(-3)=\ldots$ etc
 		\end{shortsolution}
 	\end{subproblem}
 	\begin{subproblem}
 		Given that the formula for $t(x)$ has  the form $t(x)=\dfrac{(x-A)(x-B)}{(x-C)(x-D)}$, use \cref{rat:tab:findformulat} to find the
 		values of $A$, $B$, $C$, and $D$; hence write a formula for $t(x)$.
 		\begin{shortsolution}
 			$t(x)=\dfrac{(x+3)(x-2)}{(x+2)(x+1)}$
 		\end{shortsolution}
 	\end{subproblem}
 	\begin{subproblem}
 		Given that the formula for $u(x)$ has  the form $u(x)=\dfrac{(x-A)^2}{(x-B)(x-C)}$, use \cref{rat:tab:findformulau} to find the
 		values of $A$, $B$, and $C$; hence write a formula for $u(x)$.
 		\begin{shortsolution}
 			$u(x)=\dfrac{x^2}{(x+3)(x-3)}$
 		\end{shortsolution}
 	\end{subproblem}
 	\end{problem}
 	\end{exercises}
 	 	 	
\section{Graphing rational functions (horizontal asymptotes)}
 \reformatstepslist{R} % the steps list should be R1, R2, \ldots
 We studied rational functions in the previous section, but were
 not asked to graph them; in this section we will demonstrate the
 steps to be followed in order to sketch graphs of the functions.
   
 Remember from \vref{rat:def:function} that rational functions have
 the form
 \[
 	r(x)=\frac{p(x)}{q(x)}
 \]
 In this section we will restrict attention to the case when
 \[
 	\text{degree of }p\leq \text{degree of }q
 \]
 Note that this necessarily means that each function that we consider
 in this section \emph{will have a horizontal asymptote} (see \vref{rat:def:longrun}).
 The cases in which the degree of $p$ is greater than the degree of $q$
 is covered in the next section.
   
 Before we begin, it is important to remember the following:
 \begin{itemize}
 	\item Our sketches will give a good representation of the overall
 	      shape of the graph, but until we have the tools of calculus (from MTH 251)
 	      we can not find local minimums, local maximums, and inflection points algebraically. This
 	      means that we will make our best guess as to where these points are.
 	\item We will not concern ourselves too much with the vertical scale (because of
 	      our previous point)| we will, however, mark the vertical intercept (assuming there is one),
 	      and any horizontal asymptotes.
 \end{itemize}
 \begin{pccspecialcomment}[Steps to follow when sketching rational functions]\label{rat:def:stepsforsketch}
 	\begin{steps}
 		\item \label{rat:step:first} Find all vertical asymptotes and holes, and mark them on the
 		graph using dashed vertical lines and open circles $\circ$ respectively.
 		\item Find any intercepts, and mark them using solid circles $\bullet$;
 		determine if the curve cuts the axis, or bounces off it at each zero.
 		\item Determine the behavior of the function around each asymptote| does
 		it behave like $\frac{1}{x}$ or $\frac{1}{x^2}$?
 		\item \label{rat:step:penultimate} Determine the long-run behavior of the function, and mark the horizontal
 		asymptote using a dashed horizontal line.
 		\item \label{rat:step:last}  Deduce the overall shape of the curve, and sketch it. If there isn't
 		enough information from the previous steps, then construct a table of values
 		including sample points from each branch.
 	\end{steps}
 	Remember that until we have the tools of calculus, we won't be able to
 	find the exact coordinates of local minimums, local maximums, and points
 	of inflection.
 \end{pccspecialcomment}
   
 The examples that follow show how \crefrange{rat:step:first}{rat:step:last} can be
 applied to a variety of different rational functions.
   
 %===================================
 %   Author: Hughes
 %   Date:   May 2012
 %===================================
 \begin{pccexample}\label{rat:ex:1overxminus2p2}
 	Use \crefrange{rat:step:first}{rat:step:last} to sketch a graph of the function $r$
 	that has formula
 	\[
 		r(x)=\frac{1}{x-2}
 	\]
 	\begin{pccsolution}
 		\begin{steps}
 			\item $r$ has a vertical asymptote at $2$; $r$ does not have any holes. The curve of
 			$r$ will have $2$ branches.
 			\item $r$ does not have any zeros since the numerator is never equal to $0$. The
 			vertical intercept of $r$ is $\left( 0,-\frac{1}{2} \right)$.
 			\item $r$ behaves like $\frac{1}{x}$ around its vertical asymptote since $(x-2)$
 			is raised to the power $1$.
 			\item Since the degree of the numerator is less than the degree of the denominator,
 			according to \vref{rat:def:longrun} the horizontal asymptote of $r$ has equation $y=0$.
 			\item We put the details we have obtained so far on \cref{rat:fig:1overxminus2p1}. Notice
 			that there is only one way to complete the graph, which we have done in \cref{rat:fig:1overxminus2p2}.
 		\end{steps}
 	\end{pccsolution}
 \end{pccexample}
   
 \begin{figure}[!htbp]
 	\begin{subfigure}{.45\textwidth}
 		\begin{tikzpicture}
 			\begin{axis}[
 					xmin=-5,xmax=5,
 					ymin=-5,ymax=5,
 					width=\textwidth,
 				]
 				\addplot[asymptote,domain=-5:5]({2},{x});
 				\addplot[asymptote,domain=-5:5]({x},{0});
 				\addplot[soldot] coordinates{(0,-0.5)}node[axisnode,anchor=north east]{$\left( 0,-\frac{1}{2} \right)$};
 			\end{axis}
 		\end{tikzpicture}
 		\caption{}
 		\label{rat:fig:1overxminus2p1}
 	\end{subfigure}%
 	\hfill
 	\begin{subfigure}{.45\textwidth}
 		\begin{tikzpicture}[/pgf/declare function={f=1/(x-2);}]
 			\begin{axis}[
 					xmin=-5,xmax=5,
 					ymin=-5,ymax=5,
 					width=\textwidth,
 				]
 				\addplot[pccplot] expression[domain=-5:1.8,samples=50]{f};
 				\addplot[pccplot] expression[domain=2.2:5]{f};
 				\addplot[asymptote,domain=-5:5]({2},{x});
 				\addplot[asymptote,domain=-5:5]({x},{0});
 				\addplot[soldot] coordinates{(0,-0.5)}node[axisnode,anchor=north east]{$\left( 0,-\frac{1}{2} \right)$};
 			\end{axis}
 		\end{tikzpicture}
 		\caption{}
 		\label{rat:fig:1overxminus2p2}
 	\end{subfigure}%
 	\caption{$y=\dfrac{1}{x-2}$}
 \end{figure}
   
 The function $r$ in \cref{rat:ex:1overxminus2p2} has a horizontal asymptote which has equation $y=0$.
 This asymptote lies on the horizontal axis, and you might (understandably) find it hard
 to distinguish between the two lines (\cref{rat:fig:1overxminus2p2}). When faced
 with such a situation, it is perfectly acceptable to draw the horizontal axis
 as a dashed line| just make sure to label it correctly. We will demonstrate this
 in the next example.
   
 %===================================
 %   Author: Hughes
 %   Date:   May 2012
 %===================================
 \begin{pccexample}\label{rat:ex:1overxp1}
 	Use \crefrange{rat:step:first}{rat:step:last} to sketch a graph of the function $v$
 	that has formula
 	\[
 		v(x)=\frac{10}{x}
 	\]
 	\begin{pccsolution}
 		\begin{steps}
 			\item $v$ has a vertical asymptote at $0$. $v$ does not have
 			any holes. The curve of $v$ will have $2$ branches.
 			\item $v$ does not have any zeros (since $10\ne 0$). Furthermore, $v$
 			does not have a vertical intercept since $v(0)$ is undefined.
 			\item $v$ behaves like $\frac{1}{x}$ around its vertical asymptote.
 			\item $v$ has a horizontal asymptote with equation $y=0$.
 			\item We put the details we have obtained so far in \cref{rat:fig:1overxp1}.
 			We do not have enough information to sketch $v$ yet (because $v$ does
 			not have any intercepts), so let's pick a sample
 			point in either of the $2$ branches| it doesn't matter where our sample point
 			is, because we know what the overall shape will be. Let's compute $v(2)$
 			\begin{align*}
 				v(2) & =\dfrac{10}{2} \\
 				     & = 5            
 			\end{align*}
 			We therefore mark the point $(2,5)$ on \cref{rat:fig:1overxp2}, and then complete the sketch using
 			the details we found in the previous steps.
 		\end{steps}
 		 		 		
 		\begin{figure}[!htbp]
 			\begin{subfigure}{.45\textwidth}
 				\begin{tikzpicture}
 					\begin{axis}[
 							xmin=-10,xmax=10,
 							ymin=-10,ymax=10,
 							xtick={-5,5},
 							ytick={-5,5},
 							axis line style={color=white},
 							width=\textwidth,
 						]
 						\addplot[asymptote,<->,domain=-10:10]({0},{x});
 						\addplot[asymptote,<->,domain=-10:10]({x},{0});
 					\end{axis}
 				\end{tikzpicture}
 				\caption{}
 				\label{rat:fig:1overxp1}
 			\end{subfigure}%
 			\hfill
 			\begin{subfigure}{.45\textwidth}
 				\begin{tikzpicture}[/pgf/declare function={f=10/x;}]
 					\begin{axis}[
 							xmin=-10,xmax=10,
 							ymin=-10,ymax=10,
 							xtick={-5,5},
 							ytick={-5,5},
 							axis line style={color=white},
 							width=\textwidth,
 						]
 						\addplot[pccplot] expression[domain=-10:-1]{f};
 						\addplot[pccplot] expression[domain=1:10]{f};
 						\addplot[soldot] coordinates{(2,5)}node[axisnode,anchor=south west]{$(2,5)$};
 						\addplot[asymptote,<->,domain=-10:10]({0},{x});
 						\addplot[asymptote,<->,domain=-10:10]({x},{0});
 					\end{axis}
 				\end{tikzpicture}
 				\caption{}
 				\label{rat:fig:1overxp2}
 			\end{subfigure}%
 			\caption{$y=\dfrac{10}{x}$}
 		\end{figure}
 	\end{pccsolution}
 \end{pccexample}
   
 %===================================
 %   Author: Hughes
 %   Date:   May 2012
 %===================================
 \begin{pccexample}\label{rat:ex:asympandholep1}
 	Use \crefrange{rat:step:first}{rat:step:last} to sketch a graph of the function $u$
 	that has formula
 	\[
 		u(x)=\frac{-4(x^2-9)}{x^2-8x+15}
 	\]
 	\begin{pccsolution}
 		\begin{steps}
 			\item We begin by factoring both the numerator and denominator of $u$ to help
 			us find any vertical asymptotes or holes
 			\begin{align*}
 				u(x) & =\frac{-4(x^2-9)}{x^2-8x+15}     \\
 				     & =\frac{-4(x+3)(x-3)}{(x-5)(x-3)} \\
 				     & =\frac{-4(x+3)}{x-5}             
 			\end{align*}
 			provided that $x\ne 3$. Therefore $u$ has a vertical asymptote at $5$ and
 			a hole at $3$. The curve of $u$ has $2$ branches.
 			\item $u$ has a simple zero at $-3$. The vertical intercept of $u$ is $\left( 0,\frac{12}{5} \right)$.
 			\item $u$ behaves like $\frac{1}{x}$ around its vertical asymptote at $4$.
 			\item Using \vref{rat:def:longrun} the equation of the horizontal asymptote of $u$ is $y=-4$.
 			\item We put the details we have obtained so far on \cref{rat:fig:1overxminus2p1}. Notice
 			that there is only one way to complete the graph, which we have done in \cref{rat:fig:1overxminus2p2}.
 		\end{steps}
 		 		 		
 		\begin{figure}[!htbp]
 			\begin{subfigure}{.45\textwidth}
 				\begin{tikzpicture}
 					\begin{axis}[
 							xmin=-10,xmax=10,
 							ymin=-20,ymax=20,
 							xtick={-8,-6,...,8},
 							ytick={-10,10},
 							width=\textwidth,
 						]
 						\addplot[asymptote,domain=-20:20]({4},{x});
 						\addplot[asymptote,domain=-10:10]({x},{-4});
 						\addplot[soldot] coordinates{(-3,0)(0,2.4)}node[axisnode,anchor=south east]{$\left( 0,\frac{12}{5} \right)$};
 						\addplot[holdot] coordinates{(3,12)};
 					\end{axis}
 				\end{tikzpicture}
 				\caption{}
 				\label{rat:fig:asympandholep1}
 			\end{subfigure}%
 			\hfill
 			\begin{subfigure}{.45\textwidth}
 				\begin{tikzpicture}[/pgf/declare function={f=-4*(x+3)/(x-5);}]
 					\begin{axis}[
 							xmin=-10,xmax=10,
 							ymin=-20,ymax=20,
 							xtick={-8,-6,...,8},
 							ytick={-10,10},
 							width=\textwidth,
 						]
 						\addplot[pccplot] expression[domain=-10:3.6666,samples=50]{f};
 						\addplot[pccplot] expression[domain=7:10]{f};
 						\addplot[asymptote,domain=-20:20]({5},{x});
 						\addplot[asymptote,domain=-10:10]({x},{-4});
 						\addplot[soldot] coordinates{(-3,0)(0,2.4)}node[axisnode,anchor=south east]{$\left( 0,\frac{12}{5} \right)$};
 						\addplot[holdot] coordinates{(3,12)};
 					\end{axis}
 				\end{tikzpicture}
 				\caption{}
 				\label{rat:fig:asympandholep2}
 			\end{subfigure}%
 			\caption{$y=\dfrac{-4(x+3)}{x-5}$}
 		\end{figure}
 	\end{pccsolution}
 \end{pccexample}
   
 \Cref{rat:ex:1overxminus2p2,rat:ex:1overxp1,rat:ex:asympandholep1} have focused on functions
 that only have one vertical asymptote; the remaining examples in this section
 concern functions that have more than one vertical asymptote. We will demonstrate
 that \crefrange{rat:step:first}{rat:step:last} still apply.
   
 %===================================
 %   Author: Hughes
 %   Date:   May 2012
 %===================================
 \begin{pccexample}\label{rat:ex:sketchtwoasymp}
 	Use \crefrange{rat:step:first}{rat:step:last} to sketch a graph of the function $w$
 	that has formula
 	\[
 		w(x)=\frac{2(x+3)(x-5)}{(x+5)(x-4)}
 	\]
 	\begin{pccsolution}
 		\begin{steps}
 			\item $w$ has vertical asymptotes at $-5$ and $4$. $w$ does not have
 			any holes. The curve of $w$ will have $3$ branches.
 			\item $w$ has simple zeros at $-3$ and $5$. The vertical intercept of $w$
 			is $\left( 0,\frac{3}{2} \right)$.
 			\item $w$ behaves like $\frac{1}{x}$ around both of its vertical
 			asymptotes.
 			\item The degree of the numerator of $w$ is $2$ and the degree of the
 			denominator of $w$ is also $2$. Using the ratio of the leading coefficients
 			of the numerator and denominator, we say that $w$ has a horizontal
 			asymptote with equation $y=\frac{2}{1}=2$.
 			\item We put the details we have obtained so far on \cref{rat:fig:sketchtwoasymptp1}.
 			 			 			
 			The function $w$ is a little more complicated than the functions that
 			we have considered in the previous examples because the curve has $3$
 			branches. When graphing such functions, it is generally a good idea to start with the branch
 			for which you have the most information| in this case, that is the \emph{middle} branch
 			on the interval $(-5,4)$.
 			 			 			
 			Once we have drawn the middle branch, there is only one way to complete the graph
 			(because of our observations about the behavior of $w$ around its vertical asymptotes),
 			which we have done in \cref{rat:fig:sketchtwoasymptp2}.
 		\end{steps}
 	\end{pccsolution}
 \end{pccexample}
   
 \begin{figure}[!htbp]
 	\begin{subfigure}{.45\textwidth}
 		\begin{tikzpicture}
 			\begin{axis}[
 					xmin=-10,xmax=10,
 					ymin=-10,ymax=10,
 					xtick={-8,-6,...,8},
 					ytick={-5,5},
 					width=\textwidth,
 				]
 				\addplot[asymptote,domain=-10:10]({-5},{x});
 				\addplot[asymptote,domain=-10:10]({4},{x});
 				\addplot[asymptote,domain=-10:10]({x},{2});
 				\addplot[soldot] coordinates{(-3,0)(5,0)};
 				\addplot[soldot] coordinates{(0,1.5)}node[axisnode,anchor=north west]{$\left( 0,\frac{3}{2} \right)$};
 			\end{axis}
 		\end{tikzpicture}
 		\caption{}
 		\label{rat:fig:sketchtwoasymptp1}
 	\end{subfigure}%
 	\hfill
 	\begin{subfigure}{.45\textwidth}
 		\begin{tikzpicture}[/pgf/declare function={f=2*(x+3)*(x-5)/( (x+5)*(x-4));}]
 			\begin{axis}[
 					xmin=-10,xmax=10,
 					ymin=-10,ymax=10,
 					xtick={-8,-6,...,8},
 					ytick={-5,5},
 					width=\textwidth,
 				]
 				\addplot[asymptote,domain=-10:10]({-5},{x});
 				\addplot[asymptote,domain=-10:10]({4},{x});
 				\addplot[asymptote,domain=-10:10]({x},{2});
 				\addplot[soldot] coordinates{(-3,0)(5,0)};
 				\addplot[soldot] coordinates{(0,1.5)}node[axisnode,anchor=north west]{$\left( 0,\frac{3}{2} \right)$};
 				\addplot[pccplot] expression[domain=-10:-5.56708]{f};
 				\addplot[pccplot] expression[domain=-4.63511:3.81708]{f};
 				\addplot[pccplot] expression[domain=4.13511:10]{f};
 			\end{axis}
 		\end{tikzpicture}
 		\caption{}
 		\label{rat:fig:sketchtwoasymptp2}
 	\end{subfigure}%
 	\caption{$y=\dfrac{2(x+3)(x-5)}{(x+5)(x-4)}$}
 \end{figure}
   
 The rational functions that we have considered so far have had simple
 factors in the denominator; each function has behaved like $\frac{1}{x}$
 around each of its vertical asymptotes. \Cref{rat:ex:2asympnozeros,rat:ex:2squaredasymp}
 consider functions that have a repeated factor in the denominator.
   
 %===================================
 %   Author: Hughes
 %   Date:   May 2012
 %===================================
 \begin{pccexample}\label{rat:ex:2asympnozeros}
 	Use \crefrange{rat:step:first}{rat:step:last} to sketch a graph of the function $f$
 	that has formula
 	\[
 		f(x)=\frac{100}{(x+5)(x-4)^2}
 	\]
 	\begin{pccsolution}
 		\begin{steps}
 			\item $f$ has vertical asymptotes at $-5$ and $4$. $f$ does not have
 			any holes. The curve of $f$ will have $3$ branches.
 			\item $f$ does not have any zeros (since $100\ne 0$). The vertical intercept of $f$
 			is $\left( 0,\frac{5}{4} \right)$.
 			\item $f$ behaves like $\frac{1}{x}$ around $-5$ and behaves like $\frac{1}{x^2}$
 			around $4$.
 			\item The degree of the numerator of $f$ is $0$ and the degree of the
 			denominator of $f$ is $2$. $f$ has a horizontal asymptote with
 			equation $y=0$.
 			\item We put the details we have obtained so far on \cref{rat:fig:2asympnozerosp1}.
 			 			 			
 			The function $f$ is similar to the function $w$ that we considered in \cref{rat:ex:sketchtwoasymp}|
 			it has two vertical asymptotes and $3$ branches, but in contrast to $w$ it does not have any zeros.
 			 			 			
 			We sketch $f$ in \cref{rat:fig:2asympnozerosp2}, using the middle branch as our guide
 			because we have the most information about the function on the interval $(-5,4)$.
 			 			 			
 			Once we have drawn the middle branch, there is only one way to complete the graph
 			because of our observations about the behavior of $f$ around its vertical asymptotes (it behaves like $\frac{1}{x}$),
 			which we have done in \cref{rat:fig:2asympnozerosp2}.
 			 			 			
 			Note that we are not yet able to find the local minimum of $f$ algebraically on the interval $(-5,4)$,
 			so we make a reasonable guess as to where it is| we can be confident that it is above the horizontal axis
 			since $f$ has no zeros. You may think that this is unsatisfactory, but once we have the tools of calculus, we will
 			be able to find local minimums more precisely.
 		\end{steps}
 	\end{pccsolution}
 \end{pccexample}
   
 \begin{figure}[!htbp]
 	\begin{subfigure}{.45\textwidth}
 		\begin{tikzpicture}
 			\begin{axis}[
 					xmin=-10,xmax=10,
 					ymin=-10,ymax=10,
 					xtick={-8,-6,...,8},
 					ytick={-5,5},
 					width=\textwidth,
 				]
 				\addplot[asymptote,domain=-10:10]({-5},{x});
 				\addplot[asymptote,domain=-10:10]({4},{x});
 				\addplot[asymptote,domain=-10:10]({x},{0});
 				\addplot[soldot] coordinates{(0,1.25)}node[axisnode,anchor=south east]{$\left( 0,\frac{5}{4} \right)$};
 			\end{axis}
 		\end{tikzpicture}
 		\caption{}
 		\label{rat:fig:2asympnozerosp1}
 	\end{subfigure}%
 	\hfill
 	\begin{subfigure}{.45\textwidth}
 		\begin{tikzpicture}[/pgf/declare function={f=100/( (x+5)*(x-4)^2);}]
 			\begin{axis}[
 					xmin=-10,xmax=10,
 					ymin=-10,ymax=10,
 					xtick={-8,-6,...,8},
 					ytick={-5,5},
 					width=\textwidth,
 				]
 				\addplot[asymptote,domain=-10:10]({-5},{x});
 				\addplot[asymptote,domain=-10:10]({4},{x});
 				\addplot[asymptote,domain=-10:10]({x},{0});
 				\addplot[soldot] coordinates{(0,1.25)}node[axisnode,anchor=south east]{$\left( 0,\frac{5}{4} \right)$};
 				\addplot[pccplot] expression[domain=-10:-5.12022]{f};
 				\addplot[pccplot] expression[domain=-4.87298:2.87298,samples=50]{f};
 				\addplot[pccplot] expression[domain=5:10]{f};
 			\end{axis}
 		\end{tikzpicture}
 		\caption{}
 		\label{rat:fig:2asympnozerosp2}
 	\end{subfigure}%
 	\caption{$y=\dfrac{100}{(x+5)(x-4)^2}$}
 \end{figure}
   
 %===================================
 %   Author: Hughes
 %   Date:   May 2012
 %===================================
 \begin{pccexample}\label{rat:ex:2squaredasymp}
 	Use \crefrange{rat:step:first}{rat:step:last} to sketch a graph of the function $g$
 	that has formula
 	\[
 		g(x)=\frac{50(2-x)}{(x+3)^2(x-5)^2}
 	\]
 	\begin{pccsolution}
 		\begin{steps}
 			\item $g$ has vertical asymptotes at $-3$ and $5$. $g$ does
 			not have any holes. The curve of $g$ will have $3$ branches.
 			\item $g$ has a simple zero at $2$. The vertical intercept of $g$ is
 			$\left( 0,\frac{4}{9} \right)$.
 			\item $g$ behaves like $\frac{1}{x^2}$ around both of its
 			vertical asymptotes.
 			\item The degree of the numerator of $g$ is $1$ and the degree of the denominator
 			of $g$ is $4$. Using \vref{rat:def:longrun}, we calculate that
 			the horizontal asymptote of $g$ has equation $y=0$.
 			\item The details that we have found so far have been drawn in
 			\cref{rat:fig:2squaredasymp1}. The function $g$ is similar to the functions
 			we considered in \cref{rat:ex:sketchtwoasymp,rat:ex:2asympnozeros} because
 			it has $2$ vertical asymptotes and $3$ branches.
 			 			 			
 			We sketch $g$ using the middle branch as our guide because we have the most information
 			about $g$ on the interval $(-3,5)$. Note that there is no other way to draw this branch
 			without introducing other zeros which $g$ does not have.
 			 			 			
 			Once we have drawn the middle branch, there is only one way to complete the graph
 			because of our observations about the behavior of $g$ around its vertical asymptotes| it
 			behaves like $\frac{1}{x^2}$.
 			 			 			
 		\end{steps}
 	\end{pccsolution}
 \end{pccexample}
   
 \begin{figure}[!htbp]
 	\begin{subfigure}{.45\textwidth}
 		\begin{tikzpicture}
 			\begin{axis}[
 					xmin=-10,xmax=10,
 					ymin=-10,ymax=10,
 					xtick={-8,-6,...,8},
 					ytick={-5,5},
 					width=\textwidth,
 				]
 				\addplot[asymptote,domain=-10:10]({-3},{x});
 				\addplot[asymptote,domain=-10:10]({5},{x});
 				\addplot[asymptote,domain=-10:10]({x},{0});
 				\addplot[soldot] coordinates{(2,0)(0,4/9)}node[axisnode,anchor=south west]{$\left( 0,\frac{4}{9} \right)$};
 			\end{axis}
 		\end{tikzpicture}
 		\caption{}
 		\label{rat:fig:2squaredasymp1}
 	\end{subfigure}%
 	\hfill
 	\begin{subfigure}{.45\textwidth}
 		\begin{tikzpicture}[/pgf/declare function={f=50*(2-x)/( (x+3)^2*(x-5)^2);}]
 			\begin{axis}[
 					xmin=-10,xmax=10,
 					ymin=-10,ymax=10,
 					xtick={-8,-6,...,8},
 					ytick={-5,5},
 					width=\textwidth,
 				]
 				\addplot[asymptote,domain=-10:10]({-3},{x});
 				\addplot[asymptote,domain=-10:10]({5},{x});
 				\addplot[asymptote,domain=-10:10]({x},{0});
 				\addplot[soldot] coordinates{(2,0)(0,4/9)}node[axisnode,anchor=south west]{$\left( 0,\frac{4}{9} \right)$};
 				\addplot[pccplot] expression[domain=-10:-3.61504]{f};
 				\addplot[pccplot] expression[domain=-2.3657:4.52773]{f};
 				\addplot[pccplot] expression[domain=5.49205:10]{f};
 			\end{axis}
 		\end{tikzpicture}
 		\caption{}
 		\label{rat:fig:2squaredasymp2}
 	\end{subfigure}%
 	\caption{$y=\dfrac{50(2-x)}{(x+3)^2(x-5)^2}$}
 \end{figure}
   
 Each of the rational functions that we have considered so far has had either
 a \emph{simple} zero, or no zeros at all. Remember from our work on polynomial
 functions, and particularly \vref{poly:def:multzero}, that a \emph{repeated} zero
 corresponds to the curve of the function behaving differently at the zero
 when compared to how the curve behaves at a simple zero. \Cref{rat:ex:doublezero} details a
 function that has a non-simple zero.
   
 %===================================
 %   Author: Hughes
 %   Date:   June 2012
 %===================================
 \begin{pccexample}\label{rat:ex:doublezero}
 	Use \crefrange{rat:step:first}{rat:step:last} to sketch a graph of the function $g$
 	that has formula
 	\[
 		h(x)=\frac{(x-3)^2}{(x+4)(x-6)}
 	\]
 	\begin{pccsolution}
 		\begin{steps}
 			\item $h$ has vertical asymptotes at $-4$ and $6$. $h$ does
 			not have any holes. The curve of $h$ will have $3$ branches.
 			\item $h$ has a zero at $3$ that has \emph{multiplicity $2$}.
 			The vertical intercept of $h$ is
 			$\left( 0,-\frac{3}{8} \right)$.
 			\item $h$ behaves like $\frac{1}{x}$ around both of its
 			vertical asymptotes.
 			\item The degree of the numerator of $h$ is $2$ and the degree of the denominator
 			of $h$ is $2$. Using \vref{rat:def:longrun}, we calculate that
 			the horizontal asymptote of $h$ has equation $y=1$.
 			\item The details that we have found so far have been drawn in
 			\cref{rat:fig:doublezerop1}. The function $h$ is different
 			from the functions that we have considered in previous examples because
 			of the multiplicity of the zero at $3$.
 			 			 			
 			We sketch $h$ using the middle branch as our guide because we have the most information
 			about $h$ on the interval $(-4,6)$. Note that there is no other way to draw this branch
 			without introducing other zeros which $h$ does not have| also note how
 			the curve bounces off the horizontal axis at $3$.
 			 			 			
 			Once we have drawn the middle branch, there is only one way to complete the graph
 			because of our observations about the behavior of $h$ around its vertical asymptotes| it
 			behaves like $\frac{1}{x}$.
 			 			 			
 		\end{steps}
 	\end{pccsolution}
 \end{pccexample}
   
 \begin{figure}[!htbp]
 	\begin{subfigure}{.45\textwidth}
 		\begin{tikzpicture}
 			\begin{axis}[
 					xmin=-10,xmax=10,
 					ymin=-5,ymax=5,
 					xtick={-8,-6,...,8},
 					ytick={-3,3},
 					width=\textwidth,
 				]
 				\addplot[asymptote,domain=-10:10]({-4},{x});
 				\addplot[asymptote,domain=-10:10]({6},{x});
 				\addplot[asymptote,domain=-10:10]({x},{1});
 				\addplot[soldot] coordinates{(3,0)(0,-3/8)}node[axisnode,anchor=north west]{$\left( 0,-\frac{3}{8} \right)$};
 			\end{axis}
 		\end{tikzpicture}
 		\caption{}
 		\label{rat:fig:doublezerop1}
 	\end{subfigure}%
 	\hfill
 	\begin{subfigure}{.45\textwidth}
 		\begin{tikzpicture}[/pgf/declare function={f=(x-3)^2/((x+4)*(x-6));}]
 			\begin{axis}[
 					xmin=-10,xmax=10,
 					ymin=-5,ymax=5,
 					xtick={-8,-6,...,8},
 					ytick={-3,3},
 					width=\textwidth,
 				]
 				\addplot[asymptote,domain=-10:10]({-4},{x});
 				\addplot[asymptote,domain=-10:10]({6},{x});
 				\addplot[asymptote,domain=-10:10]({x},{1});
 				\addplot[soldot] coordinates{(3,0)(0,-3/8)}node[axisnode,anchor=north west]{$\left( 0,-\frac{3}{8} \right)$};
 				\addplot[pccplot] expression[domain=-10:-5.20088]{f};
 				\addplot[pccplot] expression[domain=-3.16975:5.83642,samples=50]{f};
 				\addplot[pccplot] expression[domain=6.20088:10]{f};
 			\end{axis}
 		\end{tikzpicture}
 		\caption{}
 		\label{rat:fig:doublezerop2}
 	\end{subfigure}%
 	\caption{$y=\dfrac{(x-3)^2}{(x+4)(x-6)}$}
 \end{figure}
 \begin{exercises}
 %===================================
 %   Author: Hughes
 %   Date:   June 2012
 %===================================
 \begin{problem}[\Cref{rat:step:last}]\label{rat:prob:deduce}
 \pccname{Katie} is working on graphing rational functions. She
 has been concentrating on functions that have the form
 \begin{equation}\label{rat:eq:deducecurve}
 	f(x)=\frac{a(x-b)}{x-c}
 \end{equation}
 Katie notes that functions with this type of formula have a zero
 at $b$, and a vertical asymptote at $c$. Furthermore, these functions
 behave like $\frac{1}{x}$ around their vertical asymptote, and the
 curve of each function will have $2$ branches.
   
 Katie has been working with $3$ functions that have the form given
 in \cref{rat:eq:deducecurve}, and has followed \crefrange{rat:step:first}{rat:step:penultimate};
 her results are shown in \cref{rat:fig:deducecurve}. There is just one
 more thing to do to complete the graphs| follow \cref{rat:step:last}.
 Help Katie finish each graph by deducing the curve of each function.
 \begin{shortsolution}
 	\Vref{rat:fig:deducecurve1}
 	 	 	
 	\begin{tikzpicture}[/pgf/declare function={f=3*(x+4)/(x+5);}]
 		\begin{axis}[
 				xmin=-10,xmax=10,
 				ymin=-10,ymax=10,
 				xtick={-8,-6,...,8},
 				width=\solutionfigurewidth,
 			]
 			\addplot[soldot] coordinates{(-4,0)(0,12/5)};
 			\addplot[asymptote,domain=-10:10]({-5},{x});
 			\addplot[asymptote,domain=-10:10]({x},{3});
 			\addplot[pccplot] expression[domain=-10:-5.42857]{f};
 			\addplot[pccplot] expression[domain=-4.76923:10,samples=50]{f};
 		\end{axis}
 	\end{tikzpicture}
 	 	 	
 	\Vref{rat:fig:deducecurve2}
 	 	 	
 	\begin{tikzpicture}[/pgf/declare function={f=-3*(x-2)/(x-4);}]
 		\begin{axis}[
 				xmin=-10,xmax=10,
 				ymin=-10,ymax=10,
 				xtick={-8,-6,...,8},
 				width=\solutionfigurewidth,
 			]
 			\addplot[soldot] coordinates{(2,0)(0,-3/2)};
 			\addplot[asymptote,domain=-10:10]({4},{x});
 			\addplot[asymptote,domain=-10:10]({x},{-3});
 			\addplot[pccplot] expression[domain=-10:3.53846,samples=50]{f};
 			\addplot[pccplot] expression[domain=4.85714:10]{f};
 		\end{axis}
 	\end{tikzpicture}
 	 	 	
 	\Vref{rat:fig:deducecurve4}
 	 	 	
 	\begin{tikzpicture}[/pgf/declare function={f=2*(x-6)/(x-4);}]
 		\begin{axis}[
 				xmin=-10,xmax=10,
 				ymin=-10,ymax=10,
 				xtick={-8,-6,...,8},
 				width=\solutionfigurewidth,
 			]
 			\addplot[soldot] coordinates{(6,0)(0,3)};
 			\addplot[asymptote,domain=-10:10]({x},{2});
 			\addplot[asymptote,domain=-10:10]({4},{x});
 			\addplot[pccplot] expression[domain=-10:3.5,samples=50]{f};
 			\addplot[pccplot] expression[domain=4.3333:10]{f};
 		\end{axis}
 	\end{tikzpicture}
 \end{shortsolution}
 \end{problem}
   
 \begin{figure}[!htb]
 	\begin{widepage}
 	\setlength{\figurewidth}{0.3\textwidth}
 	\begin{subfigure}{\figurewidth}
 		\begin{tikzpicture}
 			\begin{axis}[
 					xmin=-10,xmax=10,
 					ymin=-10,ymax=10,
 					xtick={-8,-6,...,8},
 					width=\textwidth,
 				]
 				\addplot[soldot] coordinates{(-4,0)(0,12/5)};
 				\addplot[asymptote,domain=-10:10]({-5},{x});
 				\addplot[asymptote,domain=-10:10]({x},{3});
 			\end{axis}
 		\end{tikzpicture}
 		\caption{}
 		\label{rat:fig:deducecurve1}
 	\end{subfigure}%
 	\hfill
 	\begin{subfigure}{\figurewidth}
 		\begin{tikzpicture}
 			\begin{axis}[
 					xmin=-10,xmax=10,
 					ymin=-10,ymax=10,
 					xtick={-8,-6,...,8},
 					width=\textwidth,
 				]
 				\addplot[soldot] coordinates{(2,0)(0,-3/2)};
 				\addplot[asymptote,domain=-10:10]({4},{x});
 				\addplot[asymptote,domain=-10:10]({x},{-3});
 			\end{axis}
 		\end{tikzpicture}
 		\caption{}
 		\label{rat:fig:deducecurve2}
 	\end{subfigure}%
 	\hfill
 	\begin{subfigure}{\figurewidth}
 		\begin{tikzpicture}
 			\begin{axis}[
 					xmin=-10,xmax=10,
 					ymin=-10,ymax=10,
 					xtick={-8,-6,...,8},
 					width=\textwidth,
 				]
 				\addplot[soldot] coordinates{(6,0)(0,3)};
 				\addplot[asymptote,domain=-10:10]({x},{2});
 				\addplot[asymptote,domain=-10:10]({4},{x});
 			\end{axis}
 		\end{tikzpicture}
 		\caption{}
 		\label{rat:fig:deducecurve4}
 	\end{subfigure}
 	\caption{Graphs for \cref{rat:prob:deduce}}
 	\label{rat:fig:deducecurve}
 	\end{widepage}
 \end{figure}
   
 %===================================
 %   Author: Hughes
 %   Date:   June 2012
 %===================================
 \begin{problem}[\Cref{rat:step:last} for  more complicated rational functions]\label{rat:prob:deducehard}
 \pccname{David} is also working on graphing rational functions, and
 has been concentrating on functions that have the form
 \[
 	r(x)=\frac{a(x-b)(x-c)}{(x-d)(x-e)}
 \]
 David notices that functions with this type of formula have simple zeros
 at $b$ and $c$, and vertical asymptotes at $d$ and $e$. Furthermore,
 these functions behave like $\frac{1}{x}$ around both vertical asymptotes,
 and the curve of the function will have $3$ branches.
   
 David has followed \crefrange{rat:step:first}{rat:step:penultimate} for
 $3$ separate functions, and drawn the results in \cref{rat:fig:deducehard}.
 Help David finish each graph by deducing the curve of each function.
 \begin{shortsolution}
 	\Vref{rat:fig:deducehard1}
 	 	 	
 	\begin{tikzpicture}[/pgf/declare function={f=(x-6)*(x+3)/( (x-4)*(x+1));}]
 		\begin{axis}[
 				xmin=-10,xmax=10,
 				ymin=-10,ymax=10,
 				xtick={-8,-6,...,8},
 				width=\solutionfigurewidth,
 			]
 			\addplot[soldot] coordinates{(-3,0)(6,0)(0,9/2)};
 			\addplot[asymptote,domain=-10:10]({-1},{x});
 			\addplot[asymptote,domain=-10:10]({4},{x});
 			\addplot[asymptote,domain=-10:10]({x},{2});
 			\addplot[pccplot] expression[domain=-10:-1.24276]{f};
 			\addplot[pccplot] expression[domain=-0.6666:3.66667]{f};
 			\addplot[pccplot] expression[domain=4.24276:10]{f};
 		\end{axis}
 	\end{tikzpicture}
 	 	 	
 	\Vref{rat:fig:deducehard2}
 	 	 	
 	\begin{tikzpicture}[/pgf/declare function={f=3*(x-2)*(x+3)/( (x-6)*(x+5));}]
 		\begin{axis}[
 				xmin=-10,xmax=10,
 				ymin=-10,ymax=10,
 				xtick={-8,-6,...,8},
 				width=\solutionfigurewidth,
 			]
 			\addplot[soldot] coordinates{(-3,0)(2,0)(0,3/5)};
 			\addplot[asymptote,domain=-10:10]({-5},{x});
 			\addplot[asymptote,domain=-10:10]({6},{x});
 			\addplot[asymptote,domain=-10:10]({x},{3});
 			\addplot[pccplot] expression[domain=-10:-5.4861]{f};
 			\addplot[pccplot] expression[domain=-4.68395:5.22241]{f};
 			\addplot[pccplot] expression[domain=7.34324:10]{f};
 		\end{axis}
 	\end{tikzpicture}
 	 	 	
 	\Vref{rat:fig:deducehard3}
 	 	 	
 	\begin{tikzpicture}[/pgf/declare function={f=2*(x-7)*(x+3)/( (x+6)*(x-5));}]
 		\begin{axis}[
 				xmin=-10,xmax=10,
 				ymin=-10,ymax=10,
 				xtick={-8,-6,...,8},
 				width=\solutionfigurewidth,
 			]
 			\addplot[soldot] coordinates{(-3,0)(7,0)(0,1.4)};
 			\addplot[asymptote,domain=-10:10]({-6},{x});
 			\addplot[asymptote,domain=-10:10]({5},{x});
 			\addplot[asymptote,domain=-10:10]({x},{2});
 			\addplot[pccplot] expression[domain=-10:-6.91427]{f};
 			\addplot[pccplot] expression[domain=-5.42252:4.66427]{f};
 			\addplot[pccplot] expression[domain=5.25586:10]{f};
 		\end{axis}
 	\end{tikzpicture}
 	 	 	
 \end{shortsolution}
 \end{problem}
   
 \begin{figure}[!htb]
 	\begin{widepage}
 	\setlength{\figurewidth}{0.3\textwidth}
 	\begin{subfigure}{\figurewidth}
 		\begin{tikzpicture}
 			\begin{axis}[
 					xmin=-10,xmax=10,
 					ymin=-10,ymax=10,
 					xtick={-8,-6,...,8},
 					width=\textwidth,
 				]
 				\addplot[soldot] coordinates{(-3,0)(6,0)(0,9/2)};
 				\addplot[asymptote,domain=-10:10]({-1},{x});
 				\addplot[asymptote,domain=-10:10]({4},{x});
 				\addplot[asymptote,domain=-10:10]({x},{2});
 			\end{axis}
 		\end{tikzpicture}
 		\caption{}
 		\label{rat:fig:deducehard1}
 	\end{subfigure}%
 	\hfill
 	\begin{subfigure}{\figurewidth}
 		\begin{tikzpicture}
 			\begin{axis}[
 					xmin=-10,xmax=10,
 					ymin=-10,ymax=10,
 					xtick={-8,-6,...,8},
 					width=\textwidth,
 				]
 				\addplot[soldot] coordinates{(-3,0)(2,0)(0,3/5)};
 				\addplot[asymptote,domain=-10:10]({-5},{x});
 				\addplot[asymptote,domain=-10:10]({6},{x});
 				\addplot[asymptote,domain=-10:10]({x},{3});
 			\end{axis}
 		\end{tikzpicture}
 		\caption{}
 		\label{rat:fig:deducehard2}
 	\end{subfigure}%
 	\hfill
 	\begin{subfigure}{\figurewidth}
 		\begin{tikzpicture}
 			\begin{axis}[
 					xmin=-10,xmax=10,
 					ymin=-10,ymax=10,
 					xtick={-8,-6,...,8},
 					width=\textwidth,
 				]
 				\addplot[soldot] coordinates{(-3,0)(7,0)(0,1.4)};
 				\addplot[asymptote,domain=-10:10]({-6},{x});
 				\addplot[asymptote,domain=-10:10]({5},{x});
 				\addplot[asymptote,domain=-10:10]({x},{2});
 			\end{axis}
 		\end{tikzpicture}
 		\caption{}
 		\label{rat:fig:deducehard3}
 	\end{subfigure}%
 	\hfill
 	\caption{Graphs for \cref{rat:prob:deducehard}}
 	\label{rat:fig:deducehard}
 	\end{widepage}
 \end{figure}
 %===================================
 %   Author: Adams (Hughes)
 %   Date:   March 2012
 %===================================
 \begin{problem}[\Crefrange{rat:step:first}{rat:step:last}]
 Use \crefrange{rat:step:first}{rat:step:last} to sketch a graph of
 each of the following functions
 \fixthis{need 2 more subproblems here}
 \begin{multicols}{4}
 	\begin{subproblem}
 		$y=\dfrac{4}{x+2}$
 		\begin{shortsolution}
 			Vertical intercept: $(0,2)$; vertical asymptote: $x=-2$, horizontal asymptote: $y=0$.
 			 			 			
 			\begin{tikzpicture}
 				\begin{axis}[
 						framed,
 						xmin=-5,xmax=5,
 						ymin=-5,ymax=5,
 						grid=both,
 						width=\solutionfigurewidth,
 					]
 					\addplot[pccplot] expression[domain=-5:-2.8]{4/(x+2)};
 					\addplot[pccplot] expression[domain=-1.2:5]{4/(x+2)};
 					\addplot[soldot]coordinates{(0,2)};
 					\addplot[asymptote,domain=-5:5]({-2},{x});
 					\addplot[asymptote,domain=-5:5]({x},{0});
 				\end{axis}
 			\end{tikzpicture}
 		\end{shortsolution}
 	\end{subproblem}
 	\begin{subproblem}
 		$y=\dfrac{2x-1}{x^2-9}$
 		\begin{shortsolution}
 			Vertical intercept:$\left( 0,\frac{1}{9} \right)$;
 			horizontal intercept: $\left( \frac{1}{2},0 \right)$;
 			vertical asymptotes: $x=-3$, $x=3$, horizontal asymptote: $y=0$.
 			 			 			
 			\begin{tikzpicture}
 				\begin{axis}[
 						framed,
 						xmin=-5,xmax=5,
 						ymin=-5,ymax=5,
 						grid=both,
 						width=\solutionfigurewidth,
 					]
 					\addplot[pccplot] expression[domain=-5:-3.23974]{(2*x-1)/(x^2-9)};
 					\addplot[pccplot,samples=50] expression[domain=-2.77321:2.83974]{(2*x-1)/(x^2-9)};
 					\addplot[pccplot] expression[domain=3.17321:5]{(2*x-1)/(x^2-9)};
 					\addplot[soldot]coordinates{(0,1/9)(1/2,0)};
 					\addplot[asymptote,domain=-5:5]({-3},{x});
 					\addplot[asymptote,domain=-5:5]({3},{x});
 					\addplot[asymptote,domain=-5:5]({x},{0});
 				\end{axis}
 			\end{tikzpicture}
 		\end{shortsolution}
 	\end{subproblem}
 	\begin{subproblem}
 		$y=\dfrac{x+3}{x-5}$
 		\begin{shortsolution}
 			Vertical intercept $\left( 0,-\frac{3}{5} \right)$; horizontal
 			intercept: $(-3,0)$; vertical asymptote: $x=5$; horizontal asymptote: $y=1$.
 			 			 			
 			\begin{tikzpicture}
 				\begin{axis}[
 						framed,
 						xmin=-10,xmax=10,
 						ymin=-5,ymax=5,
 						xtick={-8,-6,...,8},
 						minor ytick={-3,-1,...,3},
 						grid=both,
 						width=\solutionfigurewidth,
 					]
 					\addplot[pccplot] expression[domain=-10:3.666]{(x+3)/(x-5)};
 					\addplot[pccplot] expression[domain=7:10]{(x+3)/(x-5)};
 					\addplot[asymptote,domain=-5:5]({5},{x});
 					\addplot[asymptote,domain=-10:10]({x},{1});
 					\addplot[soldot]coordinates{(0,-3/5)(-3,0)};
 				\end{axis}
 			\end{tikzpicture}
 		\end{shortsolution}
 	\end{subproblem}
 	\begin{subproblem}
 		$y=\dfrac{2x+3}{3x-1}$
 		\begin{shortsolution}
 			Vertical intercept: $(0,-3)$; horizontal intercept: $\left( -\frac{3}{2},0 \right)$;
 			vertical asymptote: $x=\frac{1}{3}$, horizontal asymptote: $y=\frac{2}{3}$.
 			 			 			
 			\begin{tikzpicture}[/pgf/declare function={f=(2*x+3)/(3*x-1);}]
 				\begin{axis}[
 						framed,
 						xmin=-5,xmax=5,
 						ymin=-5,ymax=5,
 						grid=both,
 						width=\solutionfigurewidth,
 					]
 					\addplot[pccplot] expression[domain=-5:0.1176]{f};
 					\addplot[pccplot] expression[domain=0.6153:5]{f};
 					\addplot[asymptote,domain=-5:5]({1/3},{x});
 					\addplot[asymptote,domain=-5:5]({x},{2/3});
 					\addplot[soldot]coordinates{(0,-3)(-3/2,0)};
 				\end{axis}
 			\end{tikzpicture}
 		\end{shortsolution}
 	\end{subproblem}
 	\begin{subproblem}
 		$y=\dfrac{4-x^2}{x^2-9}$
 		\begin{shortsolution}
 			Vertical intercept: $\left( 0,-\frac{4}{9} \right)$;
 			horizontal intercepts: $(2,0)$, $(-2,0)$;
 			vertical asymptotes: $x=-3$, $x=3$; horizontal asymptote: $y=-1$.
 			 			 			
 			\begin{tikzpicture}[/pgf/declare function={f=(4-x^2)/(x^2-9);}]
 				\begin{axis}[
 						framed,
 						xmin=-5,xmax=5,
 						ymin=-5,ymax=5,
 						grid=both,
 						width=\solutionfigurewidth,
 					]
 					\addplot[pccplot] expression[domain=-5:-3.20156]{f};
 					\addplot[pccplot,samples=50] expression[domain=-2.85774:2.85774]{f};
 					\addplot[pccplot] expression[domain=3.20156:5]{f};
 					\addplot[asymptote,domain=-5:5]({-3},{x});
 					\addplot[asymptote,domain=-5:5]({3},{x});
 					\addplot[asymptote,domain=-5:5]({x},{-1});
 					\addplot[soldot] coordinates{(-2,0)(2,0)(0,-4/9)};
 				\end{axis}
 			\end{tikzpicture}
 		\end{shortsolution}
 	\end{subproblem}
 	\begin{subproblem}
 		$y=\dfrac{(4x+5)(3x-4)}{(2x+5)(x-5)}$
 		\begin{shortsolution}
 			Vertical intercept: $\left( 0,\frac{4}{5} \right)$;
 			horizontal intercepts: $\left( -\frac{5}{4},0 \right)$, $\left( \frac{4}{3},0 \right)$;
 			vertical asymptotes: $x=-\frac{5}{2}$, $x=5$; horizontal asymptote: $y=6$.
 			 			 			
 			\begin{tikzpicture}[/pgf/declare function={f=(4*x+5)*(3*x-4)/((2*x+5)*(x-5));}]
 				\begin{axis}[
 						framed,
 						xmin=-10,xmax=10,
 						ymin=-20,ymax=20,
 						xtick={-8,-6,...,8},
 						ytick={-10,0,...,10},
 						minor ytick={-15,-5,...,15},
 						grid=both,
 						width=\solutionfigurewidth,
 					]
 					\addplot[pccplot] expression[domain=-10:-2.73416]{f};
 					\addplot[pccplot] expression[domain=-2.33689:4.2792]{f};
 					\addplot[pccplot] expression[domain=6.26988:10]{f};
 					\addplot[asymptote,domain=-20:20]({-5/2},{x});
 					\addplot[asymptote,domain=-20:20]({5},{x});
 					\addplot[asymptote,domain=-10:10]({x},{6});
 					\addplot[soldot]coordinates{(0,4/5)(-5/4,0)(4/3,0)};
 				\end{axis}
 			\end{tikzpicture}
 		\end{shortsolution}
 	\end{subproblem}
 \end{multicols}
 \end{problem}
 %===================================
 %   Author: Hughes
 %   Date:   March 2012
 %===================================
 \begin{problem}[Inverse functions]
 Each of the following rational functions are invertible
 \[
 	F(x)=\frac{2x+1}{x-3}, \qquad G(x)= \frac{1-4x}{x+3}
 \]
 \begin{subproblem}
 	State the domain of each function.
 	\begin{shortsolution}
 		\begin{itemize}
 			\item The domain of $F$ is $(-\infty,3)\cup(3,\infty)$.
 			\item The domain of $G$ is $(-\infty,-3)\cup(-3,\infty)$.
 		\end{itemize}
 	\end{shortsolution}
 \end{subproblem}
 \begin{subproblem}
 	Find the inverse of each function, and state its domain.
 	\begin{shortsolution}
 		\begin{itemize}
 			\item $F^{-1}(x)=\frac{3x+1}{x-2}$; the domain of $F^{-1}$ is $(-\infty,2)\cup(2,\infty)$.
 			\item $G^{-1}(x)=\frac{3x+1}{x+4}$; the domain of $G^{-1}$ is $(-\infty,-4)\cup(-4,\infty)$.
 		\end{itemize}
 	\end{shortsolution}
 \end{subproblem}
 \begin{subproblem}
 	Hence state the range of the original functions.
 	\begin{shortsolution}
 		\begin{itemize}
 			\item The range of $F$ is the domain of $F^{-1}$, which is $(-\infty,2)\cup(2,\infty)$.
 			\item The range of $G$ is the domain of $G^{-1}$, which is $(-\infty,-4)\cup(-4,\infty)$.
 		\end{itemize}
 	\end{shortsolution}
 \end{subproblem}
 \begin{subproblem}
 	State the range of each inverse function.
 	\begin{shortsolution}
 		\begin{itemize}
 			\item The range of $F^{-1}$ is the domain of $F$, which is $(-\infty,3)\cup(3,\infty)$.
 			\item The range of $G^{-1}$ is the domain of $G$, which is $(-\infty,-3)\cup(-3,\infty)$.
 		\end{itemize}<++>
 	\end{shortsolution}
 \end{subproblem}
 \end{problem}
 %===================================
 %   Author: Hughes
 %   Date:   March 2012
 %===================================
 \begin{problem}[Composition]
 Let $r$ and $s$ be the rational functions that have formulas
 \[
 	r(x)=\frac{3}{x^2},\qquad s(x)=\frac{4-x}{x+5}
 \]
 Evaluate each of the following.
 \begin{multicols}{3}
 	\begin{subproblem}
 		$(r\circ s)(0)$
 		\begin{shortsolution}
 			$\frac{75}{16}$
 		\end{shortsolution}
 	\end{subproblem}
 	\begin{subproblem}
 		$(s\circ r)(0)$
 		\begin{shortsolution}
 			$(s\circ r)(0)$ is undefined.
 		\end{shortsolution}
 	\end{subproblem}
 	\begin{subproblem}
 		$(r\circ s)(2)$
 		\begin{shortsolution}
 			$\frac{147}{4}$
 		\end{shortsolution}
 	\end{subproblem}
 	\begin{subproblem}
 		$(s\circ r)(3)$
 		\begin{shortsolution}
 			$192$
 		\end{shortsolution}
 	\end{subproblem}
 	\begin{subproblem}
 		$(s\circ r)(4)$
 		\begin{shortsolution}
 			$(s\circ r)(4)$ is undefined.
 		\end{shortsolution}
 	\end{subproblem}
 	\begin{subproblem}
 		$(s\circ r)(x)$
 		\begin{shortsolution}
 			$\dfrac{4x^2-3}{1+5x^2}$
 		\end{shortsolution}
 	\end{subproblem}
 \end{multicols}
 \end{problem}
 %===================================
 %   Author: Hughes
 %   Date:   March 2012
 %===================================
 \begin{problem}[Piecewise rational functions]
 The function $R$ has formula
 \[
 	R(x)=
 	\begin{dcases}
 		\frac{2}{x+3},    & x<-5     \\
 		\frac{x-4}{x-10}, & x\geq -5 
 	\end{dcases}
 \]
 Evaluate each of the following.
 \begin{multicols}{4}
 	\begin{subproblem}
 		$R(-6)$
 		\begin{shortsolution}
 			$-\frac{2}{3}$
 		\end{shortsolution}
 	\end{subproblem}
 	\begin{subproblem}
 		$R(-5)$
 		\begin{shortsolution}
 			$\frac{3}{5}$
 		\end{shortsolution}
 	\end{subproblem}
 	\begin{subproblem}
 		$R(-3)$
 		\begin{shortsolution}
 			$\frac{7}{13}$
 		\end{shortsolution}
 	\end{subproblem}
 	\begin{subproblem}
 		$R(5)$
 		\begin{shortsolution}
 			$-\frac{1}{5}$
 		\end{shortsolution}
 	\end{subproblem}
 \end{multicols}
 \begin{subproblem}
 	What is the domain of $R$?
 	\begin{shortsolution}
 		$(-\infty,10)\cup(10,\infty)$
 	\end{shortsolution}
 \end{subproblem}
 \end{problem}
 \end{exercises}
   
\section{Graphing rational functions (oblique asymptotes)}\label{rat:sec:oblique}
 \begin{subproblem}
 	$y=\dfrac{x^2+1}{x-4}$
 	\begin{shortsolution}
 		\begin{enumerate}
 			\item $\left( 0,-\frac{1}{4} \right)$
 			\item Vertical asymptote: $x=4$.
 			\item A graph of the function is shown below
 			       			       			      
 			      \begin{tikzpicture}[/pgf/declare function={f=(x^2+1)/(x-4);}]
 			      	\begin{axis}[
 			      			framed,
 			      			xmin=-20,xmax=20,
 			      			ymin=-30,ymax=30,
 			      			xtick={-10,10},
 			      			minor xtick={-15,-5,...,15},
 			      			minor ytick={-10,10},
 			      			grid=both,
 			      			width=\solutionfigurewidth,
 			      		]
 			      		\addplot[pccplot,samples=50] expression[domain=-20:3.54724]{f};
 			      		\addplot[pccplot,samples=50] expression[domain=4.80196:20]{f};
 			      		\addplot[asymptote,domain=-30:30]({4},{x});
 			      	\end{axis}
 			      \end{tikzpicture}
 		\end{enumerate}
 	\end{shortsolution}
 \end{subproblem}
 \begin{subproblem}
 	$y=\dfrac{x^3(x+3)}{x-5}$
 	\begin{shortsolution}
 		\begin{enumerate}
 			\item $(0,0)$, $(-3,0)$
 			\item Vertical asymptote: $x=5$, horizontal asymptote: none.
 			\item A graph of the function is shown below
 			       			       			      
 			      \begin{tikzpicture}[/pgf/declare function={f=x^3*(x+3)/(x-5);}]
 			      	\begin{axis}[
 			      			framed,
 			      			xmin=-10,xmax=10,
 			      			ymin=-500,ymax=2500,
 			      			xtick={-8,-6,...,8},
 			      			ytick={500,1000,1500,2000},
 			      			grid=both,
 			      			width=\solutionfigurewidth,
 			      		]
 			      		\addplot[pccplot,samples=50] expression[domain=-10:4]{f};
 			      		\addplot[pccplot] expression[domain=5.6068:9.777]{f};
 			      		\addplot[asymptote,domain=-500:2500]({5},{x});
 			      	\end{axis}
 			      \end{tikzpicture}
 		\end{enumerate}
 	\end{shortsolution}
 \end{subproblem}